GI Kaplan Qs

Ace your homework & exams now with Quizwiz!

A 61-year-old man presents with colicky abdominal pain and vomiting of 3 days duration. On physical examination there is moderate distention, high-pitched hyperactive bowel sounds, and a 5-cm tender groin mass. On direct questioning he explains that he has "had that bulge for many years." He has always been able to "push it back in" when he lies down but for the past 3 days he has been unable to do so. His temperature is 38.9ºC (102ºF) and white blood cell count 12,500/mm3 (normal 5,000-10,000/mm3). Which of the following is the most appropriate management at this time? A. Sonogram of the mass B. Trial of nasogastric suction and IV fluids for a few days C. Insertion of a long rectal tube via sigmoidoscopy D. Manual reduction of the hernia, followed by a period of observation E. Urgent surgical intervention

E. The clinical picture is that of a strangulated inguinal hernia. If he only had the tender mass without signs of intestinal obstruction, he might have omentum trapped. If he had no fever with a normal white blood cell count but a tender mass, he could be obstructed but without strangulation. The combination that he has, however, is clearly that of obstruction with strangulation. He needs urgent surgery. A sonogram to make a diagnosis (choice A) might be appropriate for a mass without signs of obstruction if we could not clinically document that it was a hernia. Nasogastric suction and IV fluids (choice B) are the standard approach for obstruction caused by adhesions when there are no signs suggestive of strangulation. We do not operate only for adhesions (they form again), but do so only to rescue the bowel that is trapped. In hernias, on the other hand, we want not only to rescue the bowel but also to repair the hernia. A long rectal tube (choice C) may be used in Ogilvie syndrome or volvulus but is not effective in treating strangulated hernias. Manual reduction (choice D) would actually be dangerous in this case, as it might force a dead segment of bowel into the abdomen, increasing morbidity and delaying definitive treatment. If the patient had no fever, no leukocytosis, and no tenderness, such an approach might be justified to gain time for an elective, non-rushed hernia repair. Review A hernia that cannot be reduced any further might be strangulated. Typical signs are abdominal pain, fever, a mechanical ileus (entrapped bowel), and leukocytosis. Strangulated hernia is a surgical emergency. Open repair is preferred, because the patient will need surgical resection of the gangrenous section of the bowel. Mesh is typically not part of the repair because of the risk for bacterial invasion caused by the compromised bowel.

A 24-year-old man comes to the clinic for evaluation of a 4-month history of postprandial diarrhea, weight loss of 9 pounds, and lower abdominal pain. He denies recent travel or antibiotic use. On physical examination his temperature is 38.0ºC (100.4ºF), and he has several oral aphthous ulcers. On abdominal examination there is tenderness and mild voluntary guarding in the right lower quadrant. Rectal examination reveals brown stool that is strongly positive for occult blood. Which of the following is most likely causing this patient's symptoms? A. Gram-negative organism B. Folate deficiency C. Mucosal ulceration with no transmural involvement in the ascending colon D. Toxin-producing organism E. Transmural inflammation in the region of the terminal ileum

E. This patient, who has postprandial diarrhea, weight loss, low-grade fever, and right lower quadrant findings on physical examination, has the typical presentation of Crohn's disease, which most commonly involves the terminal ileum. Inflammation in this disease is transmural, as opposed to the inflammation in ulcerative colitis that is limited to the mucosa of the large intestine. Although the etiology of Crohn's disease is not known, gram-negative organisms (choice A) have not been demonstrated to be causative. Folate deficiency (choice B) does not cause a diarrheal illness. It can, however, result infrequently from Crohn's disease if there is severe proximal small bowel malabsorption. Mucosal ulceration in the ascending colon (choice C) may be seen during a colonoscopy in patients who have Crohn's disease, but the inflammation in fact is transmural. Classic findings on colonoscopy will show involved inflamed bowel with normal bowel mucosa interspaced with the affected portions of the bowel and involvement of the entire GI tract anywhere from the mouth to the anus. Toxin-producing organisms (choice D) such as enterotoxigenic Escherichia coli and Vibrio cholerae produce a watery diarrheal syndrome without any of the other findings described in this patient. Review Crohn's disease patients can present with chronic diarrhea, weight loss, and right lower quadrant abdominal pain. The diarrhea is caused by decreased absorption of bile acids with resultant secretory diarrhea. This depletion of the bile salt pool leads to malabsorption of fat with resultant steatorrhea and increased risk for gallstone formation. Fat soluble vitamins then become depleted. Diagnosis is confirmed with colonoscopy with biopsy. Because transmural inflammation of the bowel is the rule with Crohn's disease, bowel complications include obstruction, abscess formation, sinuses, and fistulae formation.

Six days after an uncomplicated left nephrectomy, a 42-year-old man complains of moderate abdominal pain and diarrhea. He is stooling approximately 6 times a day with profuse watery diarrhea. The patient also experiences painful cramps during these episodes. In other medical history, the patient has a history of hypertension and elevated cholesterol. His medications include metoprolol (Lopressor) and simvastatin (Zocor). He denies any recent cardiopulmonary history and any history of chronic gastrointestinal disorders. His chart indicates that he received a 2-day regimen of cefazolin (Ancef) for perioperative skin prophylaxis. His temperature is 38.3ºC (101.0ºF). Stool is positive for fecal leukocytes and occult blood. The diagnosis is established with additional studies and he is given treatment. Unfortunately he fails to respond to therapy and requires surgery. An image of a segment of his colon is shown. Which of the following is the most likely pathogen?

B. This patient is experiencing C. difficile colitis, also known as pseudomembranous colitis. This type of diarrhea is common in hospitalized patients; at least 30% of hospitalized patients are colonized with the organism. Certain antibiotics have a predilection for causing C. difficile colitis: clindamycin, ampicillin, amoxicillin, and cephalosporins have a high incidence of causing the diarrhea. These antibiotics suppress normal gut flora, allowing Clostridium to proliferate. Two endotoxins (A and B) cause the symptoms and the colonic pathology noted in the vignette. Diagnosis is made by ELISA test for endotoxin A and B; sigmoidoscopy also provides additional diagnostic information, particularly the presence of plaques (also called pseudomembranes). Treatment is oral metronidazole or vancomycin. Surgery is sometimes indicated for recurrences that are unresponsive to therapy or when complications such as toxic megacolon occur. B. cereus (choice A) is a spore-forming gram-negative rod that causes diarrhea similar to that seen with C. difficile colitis. It is usually associated with food poisoning, however, particularly on reheated grain foodstuffs (such as fried rice). Treatment is symptomatic. Patients who have this pathogen develop nausea and vomiting within 6 hour of ingestion of this pathogen, and this is followed by diarrhea. There is no fever associated with this. E. coli 0157:H7 (choice C) causes severe abdominal pain and severe bloody diarrhea. It is usually obtained by consumption of raw beef or unprocessed milk. This organism also has been implicated in hemolytic uremic syndrome, a potentially deadly syndrome involving hemolysis and kidney failure. For hemolytic uremic syndrome to develop, the patient typically must be treated with antibiotics after ingestion of this pathogen. Shiga-like toxins cause direct damage to renal glomerular endothelial cells, which lead to the kidney damage. Treatment usually involves parenteral antibiotics. Shigella (choice D) species cause fever, cramps, and diarrhea that can be watery but later develop into bloody, mucus-containing stools. It is usually obtained by poor hygiene, infected food, and transmission by flies. Children attending daycare are the most likely to acquire this condition. Treatment is symptomatic if mild, oral fluoroquinolones or trimethoprim-sulfamethoxazole if more severe. The symptoms of Y. enterocolitica (choice E) infection are almost identical to those of Salmonella. Yersinia is typically acquired by ingestion of contaminated pork, beef, milk, or cheese. If it is associated with abdominal pain, the pain is classically located in the right lower quadrant, as this pathogen has a predilection for the cecum and terminal ileum. It is also known to cause mesenteric adenitis. Treatment is symptomatic. Review The 2 main risk factors for Clostridium difficile colitis are recent antibiotic use (which disrupts the normal bowel flora and allows this native pathogen to replicate unchecked) and recent hospitalization (which increases risk for acquiring this pathogen, primarily because of poor handwashing techniques). Patients who have this pathogen typically present with diarrhea, abdominal pain, and leukocytosis. There may also be associated fever, abdominal tenderness, and abdominal distention. Identification of the stool cytotoxin tissue culture assay is diagnostic. Identification is usually done by stool enzyme immunoassay for toxins A and B. Diagnosis can also be made via sigmoidoscopy or colonoscopy because of its classic appearance with pseudomembranes. Treatment involves discontinuing the antibiotic and administering oral metronidazole (Flagyl) or vancomycin.

A 28-year-old man with end-stage renal disease (ESRD) on continuous ambulatory peritoneal dialysis (CAPD) for 2 months is brought to the emergency department with fever, abdominal pain, and cloudy dialysis fluid. The pain has been present for 12 hours. There is no diarrhea or vomiting. The patient has ESRD secondary to chronic glomerulonephritis; there is no history of diabetes, urinary infections, or antibiotic use. Examination reveals a temperature of 38.9ºC (102ºF) and blood pressure 110/70 mm Hg. The throat is clear, as are the lungs. Cardiac examination reveals a grade 2/6 systolic murmur. Abdominal examination reveals decreased bowel sounds with diffuse tenderness. There is mild rebound. There is no edema or skin rash. A complete blood count shows a leukocyte count of 14,200/mm3 (normal 5,000-10,000/mm3) and hemoglobin 12.5 g/dL (normal 13.8-17.0 g/dL). Peritoneal fluid is cloudy with 1,000 white blood cells, 85% of which are polymorphonuclear leukocytes. Gram stain of the peritoneal fluid is negative. Cultures of blood and peritoneal dialysis fluid are taken. Which of the following is the most appropriate initial step in management? A. Fluconazole B. Immediate removal of dialysis catheter C. Intravenous ceftriaxone and vancomycin D. Intravenous gentamicin E. Oral ciprofloxacin

C. Gram-negative organisms classically cause peritonitis in a patient on CAPD, but there is a recent tendency for this condition to be caused by gram-positive pathogens such as Staphylococcus aureus or Epidermidis. It is usually characterized by abdominal pain and over 100 white blood cells (typically polymorphonuclear leukocytes) in a sample of peritoneal dialysis fluid. Intravenous ceftriaxone and vancomycin would be a reasonable treatment to cover both gram-negative and gram-positive pathogens. Fluconazole (choice A) would be indicated for a fungal infection. Fungal peritonitis is not usually seen until patients have been treated with multiple antibiotics or are further immunosuppressed. Immediate removal of the dialysis catheter (choice B) is usually not needed unless the patient has a peritonitis that has not improved with a trial of antibiotics. Intravenous gentamicin (choice D) has good gram-negative coverage but would not be an ideal drug to cover Staphylococcus. Ciprofloxacin (choice E) would be a very broad-spectrum antibiotic that would not be a first choice as a single antibiotic to treat staphylococcal peritonitis. Furthermore, the oral route may not be adequate, as patients who have peritonitis may have nausea and vomiting. Review Patients on chronic ambulatory peritoneal dialysis are at increased risk for peritonitis. When peritonitis occurs, it is likely to be caused by gram-negative enteric organisms or Staphylococcus aureus or Staphylococcus epidermidis, which will respond to ceftriaxone plus vancomycin. Other causative organisms include enterobacteriacaea and enterococcus, which are part of the gut flora.

A 36-year-old woman without significant past medical history is complaining of 8 weeks of intermittent diarrhea. She reports the diarrhea is nonbloody and without mucus. There is mild abdominal cramping but no severe pain. There is no associated constipation, and symptoms are not worse during stressful periods. The diarrhea occurs shortly after meals and improves with fasting. She denies any weight loss. She denies sick contacts, travel history, camping history, or eating at any unusual locations. She is unsure if there is an association with milk products. Her temperature is 36.7ºC (98ºF), blood pressure is 100/70 mm Hg, pulse is 73/min, and respirations are 13/min. Her abdomen is soft, nontender, and nondistended with normal bowel sounds. Which of the following is the most likely etiology of her diarrhea? A. Enteroinvasive E. coli B. Irritable bowel syndrome C. Lactose intolerance D. Pseudomembranous colitis E. Zollinger-Ellison syndrome

C. This patient is describing lactose intolerance. There are several clues to the diagnosis and reasons the other choices are incorrect. Lactose intolerance is very common. It is an osmotic diarrhea, which means that undigested food causes the symptoms. Resolution with fasting is another sign that the diarrhea is osmotic. The only answer choice listed that is osmotic is lactose intolerance. In addition, the lack of red flag symptoms such as significant abdominal pain, weight changes, or bloody stools leads us to believe that this is not a more serious condition. Because ingestion of lactose-containing food is the precipitating cause, fasting will improve these symptoms. Enteroinvasive diarrhea (choice A) presents with fevers, chills, abdominal pain, and diarrhea. Patients typically have a bloody diarrhea. In addition, bacterial diarrhea is acute. Our patient describes chronic diarrhea (longer than 2-3 weeks). Irritable bowel syndrome (choice B) is a very common form of chronic diarrhea. Patients will describe intermittent diarrhea and constipation together with abdominal pain that is relieved with defection. It is often exacerbated by stressful situations or fatty foods. Patients who have irritable bowel syndrome will classically have abdominal pain or discomfort along with a change in bowel frequency or form. This pain may be relieved by defecation. Symptoms may persist with fasting rather than improve with fasting. Pseudomembranous colitis (choice D) is associated with antibiotic use and is caused by the bacterium Clostridium difficile. The severity of disease can range from a small amount of diarrhea to fulminant infection. Again, this would present with acute diarrhea, not chronic diarrhea. Recent hospitalization and/or antibiotic use are the main risk factors for the development of this condition. Zollinger-Ellison syndrome (choice E) causes a large volume diarrhea. It can also present with steatorrhea. Refractory peptic ulcer disease and ulcers located in atypical locations are classically associated with this condition. It is associated with increased gastrin levels caused by a gastrin-producing neuroendocrine tumor that causes gastric acid hypersecretion, and peptic ulcer disease is found in many of these patients. Review Lactose intolerance is characterized by reduced lactase concentration in the mucosal brush border in the small intestine. Diagnosis is confirmed with lactose hydrogen breath test. Treatment includes reduction or elimination of dietary lactose or lactase preparations. Prevalence studies reveal that up to 70% of the population has lactase deficiency, with the highest rates being in the Asian countries.

A 22-year-old man is diagnosed with Crohn's disease limited to the terminal ileum. His symptoms of mild right lower quadrant pain and postprandial diarrhea resolve after the initiation of treatment with mesalamine. Two years later, he develops recurrent episodes of abdominal distention, nausea, and vomiting after large meals. On two occasions, these symptoms are accompanied by inability to pass flatus or bowel movements. Which of the following has this patient most likely developed? A. A fistula from the ileum to the sigmoid B. An obstructing cecal carcinoma C. An obstructing ileal carcinoid D. Fibrosis and a stricture in the terminal ileum E. Gastric outlet obstruction

D. This patient who has Crohn's disease has developed symptoms of a small bowel obstruction, a common and important complication of Crohn's disease which occurs as a result of chronic transmural inflammation. It both partially destroys the normal bowel wall and constricts it with thick bands of fibrosis. Other important intestinal complications can include fistula formation and chronic abscesses. In addition, a wide variety of extraintestinal complications can include autoimmune diseases (arthritis, aphthous ulcers, erythema nodosum, pyoderma gangrenosum, eye involvement, ankylosing spondylitis, primary sclerosing cholangitis) and complications related to disrupted bowel physiology (renal complications, including kidney stones and urinary tract obstruction, malabsorption, and amyloidosis secondary to long-standing inflammation). A fistula from the ileum to the sigmoid (choice A) can develop in patients who have Crohn's disease, but it will present with symptoms of diarrhea (because of the bypass of a large portion of the colon) and not obstruction. Cecal carcinoma (choice B) can cause obstruction, but it would be extraordinarily rare in a 22-year-old man. Carcinoma is more likely to complicate Crohn's disease after many years of active disease, but this remains a rare diagnosis. There is nothing in his history to suggest the development of carcinoid syndrome (choice C) in this young patient. These tumors when they do occur rarely present with a bowel obstruction, but they may present with the carcinoid syndrome (i.e., facial flushing, diarrhea, wheezing, and tricuspid regurgitation). Gastric outlet obstruction (choice E) may produce vomiting, usually of only partially digested foods, and does not usually result in significant abdominal distention. Furthermore, there is no impairment of passage of flatus or bowel movements, because the obstruction is much more proximal. Review Chronic transmural inflammation may result in stricture formation within the small bowel, leading to small bowel obstruction. Signs and symptoms of small bowel obstruction include obstipation, abdominal pain/bloating, nausea, and vomiting.

A 54-year-old man comes to the health care provider for a periodic health examination. His family history is significant for his mother who died of a cerebrovascular accident at age 72, his father who died of a myocardial infarction at age 68, and his brother who developed sigmoid cancer at age 60. The patient is on no medications except for aspirin, 81 mg daily. Physical examination is unremarkable. The patient asks for a recommendation regarding current cancer screening. Which of the following is the most appropriate screening test for this patient? A. Annual digital rectal examination and fecal occult blood testing B. Flexible sigmoidoscopy C. Flexible sigmoidoscopy and barium enema D. Colonoscopy E. Genetic testing for the p53 gene

D. Any patient who has a first-degree relative who has developed an adenoma or colorectal cancer should undergo colonoscopy for screening at age 50, or 10 years before the relative developed the adenoma or carcinoma, whichever comes first. This patient has a brother who has a colon cancer at age 60; therefore, a full colonoscopy is warranted. Screening recommendations also recommend screening for colon cancer beginning at age 50. Although there are various opinions regarding appropriate screening in the "average risk individual," there is a consensus that full colonoscopy is required in patients who have an increased risk (e.g., first-degree relative with a positive history). Annual digital rectal examination and fecal occult blood testing (choice A) are no longer considered as reliable methods of screening for colon cancer, because a shift in the demographics of colon cancer with more than half of colon cancer being identified in the first half of the colon. Fecal occult blood testing with sigmoidoscopy can be used as a screening method but is not nearly as effective as colonoscopy, because a smaller portion of the bowel gets examined. Digital rectal examination also often fails to identify premalignant colonic polyps. Flexible sigmoidoscopy along with annual fecal occult blood testing (choice B) is an acceptable initial screening technique for patients older than age 50 who have no specific known risk factors. If polyps are identified, they can be biopsied, their type established, and subsequent complete colonoscopy performed if adenomas were identified microscopically. Flexible sigmoidoscopy and barium enema (choice C) offers an alternative way of screening the entire colon in patients in whom a complete colonoscopy cannot be performed. The lack of ability to biopsy polyps during these procedures makes flexible sigmoidoscopy and barium enema a second-line choice following colonoscopy. Genetic testing for the p53 gene (choice E) is not currently used for colon cancer screening. Review Screening for colorectal cancer is recommended from age 50 for normal risk individuals and earlier for those who have a family risk for colon cancer or are experiencing colon cancer-type symptoms. Preventive strategies include aspirin (at least 300 mg daily for at least 5 years), other NSAIDs, and possibly COX II inhibitors. Some trials have identified calcium-channel blockers and estrogen as potential preventive strategies against the formation of colonic polyps, which are the precursor lesions of colon cancer. Aspirin and adequate calcium intake with limitation of red beef intake may be preventive strategies against the formation of colonic polyps, which are the precursor lesions for the development of colon cancer.

A 39-year-old chronic alcoholic comes to the emergency department with complaints of severe epigastric pain, nausea, and vomiting. He admits to actively drinking and reports that the pain is severe and radiates to his back. He reports that he had 3 similar episodes in the past year that required hospitalization and resolved after several days of not eating. His temperature is 38.4ºC (101ºF). Serum amylase and lipase levels are elevated. Which of the following may occur as a complication of this episode? A. Adult respiratory distress syndrome due to circulating phospholipase B. Development of a carcinoma as a result of pancreatic duct injury C. Hypercalcemia as a result of stimulation of parathyroid secretion D. Hypertension due to decreased intravascular volume E. Hypoglycemia due to impaired insulin production

A. Acute pancreatitis is suggested by this patient's severe epigastric pain, which radiates to his back and is accompanied by nausea and vomiting. The diagnosis is confirmed by demonstrating serum elevation of the pancreatic enzymes amylase and lipase. More than 80% of the hospital admissions for acute pancreatitis are related to either biliary tract disease (typically with a stone lodging in the duct system below the entry of the pancreatic duct) or alcoholism, and this patient admits to recent active drinking. Acute pancreatitis is a very dangerous disease, not only because it potentially destroys large areas of pancreatic tissue, but also because the destruction of the exocrine gland tissues potentially releases a great many enzymatically or physiologically active substances into the bloodstream. Among the released substances is phospholipase, which circulates through the bloodstream and damages the alveolar capillary membranes in the lungs, predisposing for adult respiratory distress syndrome (ARDS). Pancreatic cancer (choice B) does not develop as a complication of acute pancreatitis. Hypercalcemia (choice C) does not occur as a result of pancreatitis, although it may be an infrequent cause of pancreatitis. Hypocalcemia occurs in severe pancreatitis by saponification of calcium by free fatty acids and undigested fats. Decreased intravascular volume (choice D) does occur, but it leads to hypotension and not hypertension. In patients who have severe pancreatitis, insulin production and release is impaired (choice E), leading to hyperglycemia rather than hypoglycemia. Review Acute pancreatitis leads to production of inflammatory mediators that can lead to damage of the alveolocapillary membrane of the lung, which can lead to destruction of the pneumocytes and decrease in the production of surfactant. This leads to increased surface tension in the lung, along with inadequate oxygenation. Bilateral lung infiltrates and potentially ARDS can develop. These patients may require mechanical ventilation if ARDS develops.

A 7-month-old girl is brought to the office by her grandmother who is concerned that her granddaughter isn't gaining any weight. The grandmother lives out of state and occasionally visits her daughter. She was last here 3 months ago, when the girl was 4 months old; at the time the girl weighed 8 pounds. She was smiling and waving her arms and legs joyously when she was attended to. However, when the grandmother arrived a few days ago, she noticed at once that the child was nervous and often crying, and her weight was only 8.5 pounds. She had also noticed that the infant was somewhat unkempt, with long fingernails and a diaper rash that was very sore and looked infected in places. Her daughter, the infant's mother, had started working again and was having a hard time coordinating the infant's care, so she often had to leave her unattended for a couple of hours until her boyfriend could come and babysit a while. On physical examination, the infant indeed weighs 8.5 pounds. There is diffuse erythema and edema of the diaper area with areas of honey-colored crust-scale that spares the inguinal folds and does not involve skin outside of the occluded area. Her occipital skull is flattened and there is a circular patch of alopecia at the center of the flattened zone. She has poor muscle tone and cannot sit up appropriately. Which of the following is the most appropriate next step in management? A. Hospitalization for documentation of caloric intake and weight gain B. Prescription for oral antibiotics C. Reassurance D. Skin-prick testing E. A sweat chloride test

A. The patient shows failure to thrive, along with signs and symptoms of abuse by deprivation: inadequate weight gain, unkempt look, impetigo-like irritant contact dermatitis in the diaper area, flattened occiput with pressure alopecia secondary to long hours of lying on the back, irritability, and poor muscle tone. Failure to thrive is failure to gain weight or deceleration of weight growth. Patients may show little subcutaneous fat, muscle wasting, rashes, poor tone, and weak cry. Hospitalization for documentation of caloric intake is sometimes necessary, especially in situations in which abuse by starvation or deprivation is suspected. Complete blood counts, urinalysis, liver function tests, and serum protein should be drawn. A sweat chloride test should be considered. Oral antibiotics (choice B) are not indicated as the first most important step, because normal care for this child with adequate feeding and regular diaper changes and cleansing of the irritated and impetigo-like area with clean water and mild soap, followed by application of topical anti-inflammatory and antibiotic preparations, will most certainly lead to resolution. This is not the child's main area of concern at this time. Reassurance (choice C) would not be adequate management, because the mother is obviously neglecting her child and has caused her failure to thrive by food and care deprivation. Even though the grandmother may improve the child's disposition during her stay, she cannot guarantee that similar conditions would not recur once she leaves again. Skin-prick testing (choice D) is used in the diagnosis of allergic diseases. It is not indicated as the first most important step in the management of an infant who has failure to thrive, but it may be considered at a later time if other tests are negative and there is indication that the infant is suffering from an allergic predisposition. A sweat chloride test (choice E) may be indicated later in the evaluation of this child, but the first most appropriate step is to improve the child's environment and record caloric intake and weight gain in controlled surroundings. This child does not have the classic presentation for cystic fibrosis, which typically is marked with recurrent cycles of mucus retention, lung infection, and recurrent bouts of inflammation. Protein calorie malabsorption is part of the cystic fibrosis presentation, but this condition is less likely, given that all newborns are screened for this condition, which allows for early detection and treatment. Review A health care provider has a duty to protect vulnerable populations. If child abuse/neglect is suspected, it is the duty of the health care provider to admit the child to the hospital, feed the child, and monitor response to therapy. If the child thrives in the hospital setting when given appropriate calories, it is much more likely that child neglect/abuse has occurred. Failure of the child to respond to this appropriate nutrition should prompt the health care provider to search for nutritional deficiency/malabsorption states or other pathophysiologic processes.

A 68-year-old woman reports that she had an operation 40 years earlier for intractable ulcer symptoms. She recalls that a large portion of her stomach was removed. Review of her prior medical records indicates that she underwent an antrectomy and a gastrojejunostomy. Which of the following complications is most likely in this scenario? A. Bacterial overgrowth B. Bleeding from the lower gastrointestinal tract C. Chronic pancreatitis D. Constipation E. Pancreatic insufficiency

A. This patient had a Billroth II gastrojejunostomy, which is a commonly used operation for both intractable peptic ulcer disease and gastric carcinoma. In this operation, the distal approximately two thirds of the stomach is resected, and the stomach is then attached along a longitudinal plane to the jejunum. The technique usually leaves a foot or more of duodenum in a "blind loop" configuration through which food does not pass. The operation does, however, allow preservation of the extrahepatic duct system, so that biliary and pancreatic secretions can still enter the gut. Patients are at risk for bacterial overgrowth in the blind loop. Blind loop syndrome can be a cause of steatorrhea related to bacterial destruction of bile salts, macrocytic anemia related to malabsorption of vitamin B12, and diarrhea and malnutrition related to generalized malabsorption of nutrients secondary to bacterial destruction of pancreatic enzymes. This blind loop may allow the retained bacteria to make more vitamin K, which is important to remember in the setting of a patient taking warfarin (Coumadin). Patients can bleed after a Billroth II by the formation of an ulcer at the gastrojejunostomy (i.e., a marginal ulcer), but there is no cause for lower gastrointestinal tract bleeding (choice B). Chronic pancreatitis (choice C) and pancreatic insufficiency (choice E) are not complications of gastrojejunostomy and Billroth II operation. Constipation (choice D) is not a feature of having had a gastrojejunostomy. Instead, some of these patients are prone to dumping syndrome, in which diarrhea is common. Review The management of peptic ulcer disease has been transformed with the advent of proton pump inhibitors and H2 blockers. Surgery for patients who have peptic ulcer disease is limited to those who have upper GI bleeding that is not responsive to endoscopy therapy or those who have gastric outlet obstruction that is not responsive to conservative management. Surgical complications include blind loop syndrome from the Billroth II procedure, which can lead to malabsorption caused by less absorption capability, and blind loop syndrome, which can lead to bacterial overgrowth, vitamin B12 deficiency, and diarrhea.

A 31-year-old man describes a burning sensation in the substernal area after eating chocolate, caffeine, or alcohol. The symptoms are exacerbated at night, and he has woken on several occasions from sleep because of coughing. Over-the-counter antacids and H2 receptor antagonists provide little relief. He often takes antacids before dinner. Which of the following is the most likely underlying cause for his coughing? A. Acid-induced bronchoconstriction B. Aspiration of antacids taken before bedtime C. Aspiration of solid food ingested with dinner D. H2-receptor antagonist-induced bronchoconstriction E. Tracheoesophageal fistula

A. This patient has the typical symptoms of gastroesophageal reflux disease (GERD) and has now developed nocturnal aspiration of acid. This is one of the extra-esophageal complications of acid reflux. It typically occurs at night, when the patient is supine and has a relatively poor gag reflex because of sleep. The other common extra-esophageal symptoms of GERD are laryngitis and hoarseness, as acid reflux inflames the vocal cords. Antacids taken before bedtime (choice B) should tend to decrease rather than exacerbate his symptoms, because they will lower the stomach acidity. There is no reason for this otherwise healthy man to have a nocturnal aspiration of food eaten hours earlier (choice C). H2-receptor antagonist-induced bronchoconstriction (choice D) is incorrect, because these drugs do not cause bronchoconstriction. Tracheoesophageal fistula (choice E) in adults is usually the result of an underlying bronchogenic carcinoma, which would be very unusual in a 31-year-old man, particularly with no history of smoking. Review GERD symptoms may be exacerbated at night when the patient assumes a recumbent position. This reflux of gastric contents may cause atypical symptoms, which include cough, laryngitis, asthma, or dental erosion. An exacerbation of previously controlled asthma may also be a sign of GERD. Treatment consists of lifestyle modifications and acid suppression therapy.

An 18-year-old college student without a significant past medical history comes to the health care provider complaining of a 2-week history of bloody diarrhea, vague intermittent abdominal pain, and a 15-pound weight loss. He denies any sick contacts, travel, or eating any foods that were "new or unusual." His temperature is 38.3ºC (100.9ºF), blood pressure 100/70 mm Hg, pulse 93/min, and respirations 13/min. Physical examination reveals a thin young man in no distress. His abdomen is mildly diffusely tender without rebound or guarding. Rectal examination is hemoccult positive. Laboratory studies show a mild iron-deficiency anemia and a mildly elevated sedimentation rate. Colonoscopy reveals continuous and extensive inflammation of the colon without skip lesions. Which of the following is associated with this patient's condition? A. Curative colectomy B. Fistula formation C. Noncaseating granulomas on biopsy D. Primary biliary cirrhosis E. Rectal sparing

A. This patient has ulcerative colitis. This question highlights the differences between ulcerative colitis and Crohn's disease. The presence of continuous inflammation as opposed to "skip lesions," bloody diarrhea, and rectal involvement all suggest ulcerative colitis. Colectomy is curative but is usually reserved for treatment-resistant cases or patients who have active ulcerative colitis for longer than 10 years, because those patients are at high risk for colon cancer. Fistula formation (choice B), noncaseating granulomas on biopsy specimen (choice C), transmural lesions, and skip lesions are all characteristic of Crohn's disease. They help to differentiate Crohn's disease from ulcerative colitis. Ulcerative colitis is associated with primary sclerosing cholangitis. It has no strong association with primary biliary cirrhosis (choice D), which is a condition that is primarily seen in women and those older than age 45 years. Bloody diarrhea is very common in ulcerative colitis because of the high incidence of rectal involvement. Ninety-five percent of ulcerative colitis patients have rectal involvement, as opposed to 40% of patients who have Crohn's disease. The disease classically manifests in the rectum and then spreads continuously proximally. We therefore would not expect rectal sparing (choice E) in our patient. Review Ulcerative colitis is characterized by diffuse inflammation of the colonic mucosa, which begins in the rectum and progresses in a stepwise fashion proximally. It is characterized by a relapsing, remitting course. Treatment is intended to induce and maintain remission. Complications of ulcerative colitis include toxic megacolon (risk for rupture) and adenocarcinoma of the colon, which is seen in 3 to 5% of the patients.

A male patient has a sudden onset of fretfulness and pain. He curls up with his legs located in a fetal position. Over the next few hours he alternates between episodes of pain/crying with tears and acting normally. The patient's mother fears something is terribly wrong and brings him to the emergency department. His past medical history is unremarkable. The previous week he had had a cold with a runny nose. Stools had been normal that day but at the hospital he has a semi-soft stool with some blood and mucus. On examination the patient is quiet. The abdomen is surprisingly soft and normal. This classic presentation of intussusception is most likely to occur in which of the following age groups? A. Birth to 4 weeks of age B. 3 to 12 months of age C. 3 to 5 years of age D. Early adolescence E. Late adolescence

B. Intussusception usually occurs between ages 3 and 12 months, with the peak incidence at 6 months of age. It characteristically presents with episodes of distress and crying interspersed with quiet periods of normal behavior and playing. More than 50% of children will pass stool mixed with mucus and blood, classically known as the "currant jelly" stool. Usually the abdomen is soft and nontender, but with advanced intussusception there may be signs of peritonitis. Sometimes a sausage-like mass may be palpable in the upper abdomen. A barium enema is performed for the diagnosis and treatment of the intussusception. Abdominal conditions that may occur more frequently in the neonatal period, birth to 4 weeks of age (choice A), include disorders such as volvulus, pyloric stenosis, and necrotizing enterocolitis. Volvulus would present with bilious vomiting and abdominal distention. Pyloric stenosis usually presents at about 4-6 weeks of age with nonbilious vomiting. Necrotizing enterocolitis is more common among premature infants and may present with grossly bloody stools. In the 3- to 5-year age group (choice C) abdominal pain is frequently caused by constipation. Other common abdominal conditions at this age include gastroenteritis, whereas abdominal tumors such as Wilms tumor and neuroblastoma are less common. Appendicitis is not common before the age of 5 years. Intussusception is less likely to occur during either early adolescence (choice D) or late adolescence (choice E). Appendicitis usually occurs between 10 and 19 years of age, but it may occur throughout a person's lifespan. Gynecologic disorders including pelvic inflammatory disease, ovarian cysts, and pregnancy-related conditions should be considered in adolescent girls who present with acute abdominal complaints. Review Intussusception is a condition in the abdomen in which there is a prolapse of one part of the intestine into the lumen of an immediately distal adjoining part of the intestine. The ileocecal region is the most commonly affected portion of the intestine, but it could affect any portion of the colon. This condition may lead to bowel wall edema from venous obstruction and it can progress to bowel necrosis if left untreated. The most commonly affected population is at 3-12 months of age, at which time it is typically an idiopathic phenomenon. When intussusception occurs in an older individual, it is typically secondary to a pathologic lead point, such as polyp, lymphoma, cancer, or Meckel diverticulum.

A 40-year-old woman presents with intense pruritus and fatigue. Her medical history is significant for hypothyroidism. Physical examination reveals a slightly enlarged liver and xanthomas. Laboratory studies show a cholesterol level of 538 mg/dL and alkaline phosphatase 571 IU/L. Which of the following serum values would most likely be elevated? A. Alpha-fetoprotein B. Antimitochondrial antibody C. HBs antigen D. Prothrombin time E. Serum transaminases

B. Primary biliary cirrhosis is an autoimmune disease associated with elevated antimitochondrial antibodies. Hashimoto thyroiditis, an autoimmune disease that is the most common cause of hypothyroidism, is also associated with increased levels of this antibody. Elevated alpha-fetoprotein (choice A) is associated with hepatocellular carcinoma. HBs antigen (choice C) is usually elevated in active hepatitis B (unless the patient is tested during the window period). Elevated prothrombin time (choice D) and increased serum transaminases (choice E) are indicative of liver failure. These values are usually normal in primary biliary cirrhosis unless the patient has reached end-stage liver disease. Review Primary biliary cirrhosis is a condition seen primarily in women in their fifties. Presentation is consistent with cholestatic liver disease, with pruritis and fatigue being common presenting symptoms. Disease progression is controlled with the use of corticosteroids and ursodiol. Liver transplantation is an effective therapy for end-stage liver disease. This is an autoimmune disease of the small intrahepatic bile ducts that results in portal tract inflammation. Alkaline phosphatase and bilirubin, markers of cholestasis, are elevated. Ninety percent of affected patients will have a positive antimitochondrial antibody, which is considered the serum marker for this condition.

A 4-year-old girl was diagnosed with left-sided otitis media 10 days ago and prescribed oral amoxicillin 40 mg/kg/day for 7 days. She has since developed watery, bloody, diarrhea with mucus, crampy abdominal pain, and fever. On physical examination her temperature is 39.4ºC (102.9ºF), pulse 88/min, and respirations 16/min. She has normal bowel sounds and is diffusely tender to palpation. Which of the following is the most appropriate initial step in diagnosis? A. Barium enema B. Evaluation of stool for Clostridium difficile toxins C. Evaluation of stool for rotavirus D. Stool examination for ova and parasites E. Stool hemoccult test

B. The child has pseudomembranous colitis, caused by the toxins produced by Clostridium difficile. The colon is colonized by C. difficile after eradication of the normal microflora by a course of antibiotics. Virtually all antibiotics have been implicated in the pathogenesis of pseudomembranous colitis. Patients usually present with watery diarrhea, tenesmus, fever, and abdominal pain. Symptoms typically occur 7-10 days after initiation of the antibiotic; in rare cases, pseudomembranous colitis can occur up to 6 weeks after antibiotic initiation. Toxic megacolon, resulting from progressive loss of muscular tone in the colon, is a rare but serious complication that could progress to perforation of the colon and peritonitis. On sigmoidoscopy or colonoscopy, the disease is characterized by an accumulation of an inflammatory exudate, the "pseudomembrane" over the mucosa. Assay for C. difficile toxins (enterotoxin A or cytotoxin B) is useful in diagnosing pseudomembranous colitis. . Oral metronidazole is the treatment of choice but another option is oral vancomycin. Barium enema (choice A) can image gross structural abnormalities of the colon but is not helpful in diagnosing pseudomembranous colitis. Evaluation of stool for rotavirus (choice C) might not be the best test with the history of recent antibiotic use. Rotavirus is a common pathogen, however, especially during wintertime, causing viral gastroenteritis. Rotavirus infection usually causes fever, abdominal pain, vomiting, and watery diarrhea. It is typically not associated with recent antibiotic use. Evaluation of stool for ova and parasites (choice D) is not a good initial test for a syndrome that is most consistent with C. difficile infection. Patients who have recent travel to endemic areas or patients who have prolonged diarrhea without another explanation should have this test ordered. The hemoccult test (choice E) is useless because the diarrhea is grossly bloody. Review Recent hospitalization and antibiotic use are the most common causes of C. difficile colitis. Patients typically present with diarrhea, abdominal pain, and leukocytosis. Patients may also have fever, abdominal distention and tenderness. Stool antigen tests for toxins A and B are useful when they are positive. There may also be evidence of pseudomembranes seen on colonoscopy or sigmoidoscopy. Treatment is discontinuation of antibiotic (if possible), oral metronidazole or vancomycin.

A 56-year-old alcoholic man gives a history of several years of constant epigastric pain, radiating straight through to the back. The pain is severe, present at all times, and exacerbated by eating. He also has steatorrhea and diabetes. He relates that he began to have episodes of acute alcoholic pancreatitis in his mid-thirties. At first these occurred every few years, but eventually they grew in number to several attacks per year. Eventually the pain became constant. Although he made numerous attempts to quit drinking, he was not successful until about 1 year ago; still, his current abstinence has not alleviated the pain. Which of the following is the most appropriate initial step in evaluation? A. Arteriogram B. CT scan of the upper abdomen C. Endoscopic retrograde cholangiopancreatogram (ERCP) D. Sonogram of the upper abdomen E. Upper gastrointestinal series with barium

B. The clinical diagnosis is chronic pancreatitis, and CT scan will give useful information about potentially correctable features of the disease, such as pseudocysts, dilated ducts, calculi, or areas amenable to resection. CT scan of the abdomen is considered to be the most sensitive test for picking up chronic pancreatitis. If surgery is contemplated, ERCP will then be needed. Arteriogram (choice A) adds little to the diagnosis of chronic pancreatitis, and it is invasive and expensive. It does not have a role in the evaluation of a patient suspected of having chronic pancreatitis. ERCP (choice C) is needed before surgery, but it would not precede CT scan in the initial evaluation because of its invasiveness. It would not be necessary unless the CT scan of the abdomen picked up a correctable cause for the chronic pancreatitis, such as retained stone in the common duct. Sonogram (choice D) lacks the necessary detail for this evaluation. It is an excellent way to follow the size of pseudocysts that are being observed. Abdominal ultrasound lacks the sensitivity of the CT scan of the abdomen. Upper gastrointestinal series (choice E) would give minimal information about the pancreas. It is not typically used in the assessment of a patient suspected of having chronic pancreatitis. Review CT scan of the abdomen in a patient who is suspected of having chronic pancreatitis will show pancreatic calcifications, focal or diffuse enlargement of the pancreas, ductal dilation, and/or vascular complications. Although it is more expensive and has radiation exposure associated with it, the CT scan of the abdomen is 10-20% more sensitive for picking up chronic pancreatitis than abdominal ultrasound. Abdominal ultrasound is used in the assessment for possible chronic pancreatitis only if CT scan of the abdomen is not available. The plain abdominal film showing pancreatic calcifications is not very sensitive for mild to moderate chronic pancreatitis.

On postoperative day 5 after elective left hemicolectomy for chronic diverticular disease, a patient is draining copious amounts of clear pink fluid from his midline laparotomy wound. A medical student removes the dressing, confirms that it is soaked, and sees a normal-appearing fresh wound with a row of skin staples in place. The student asks the patient to sit up so he can get out of bed and be helped to the treatment room for a more thorough examination. When the patient complies, the wound opens widely, and a handful of small bowel suddenly rushes out. Which of the following is the most appropriate management at this time? A. Cover bowel with dry sterile dressings and schedule urgent surgical closure B. Cover bowel with sterile dressings soaked in warm saline and rush patient to the operating room C. Irrigate bowel with cold antiseptic solutions while awaiting urgent surgical closure D. Take patient to the treatment room and suture the skin edges together E. Wearing sterile gloves, push bowel back in and tape the wound securely

B. Until the patient attempted to get out of bed, he had a wound dehiscence that could have been handled by taping the wound securely. Once the bowel came out, the problem became an evisceration. Immediate surgical repair is mandatory. While setting it up, the bowel must be protected from drying out, and the patient must be protected from significant heat loss. The key is application of warm and moist dressings. Dry dressings (choice A) would prevent further contamination but would fail in the key elements of "warm and moist." Cold antiseptic solutions (choice C) would irritate the bowel and contribute to hypothermia. Once an evisceration has occurred, the entire abdominal wall has to be surgically closed. Suturing the skin edges outside of the operating room (choice D) or resorting to tape (choice E) would not suffice. Review Postoperative wound evisceration needs to be treated urgently with operative surgical wound repair. With the wound open after evisceration, the patient will lose fluid and heat and will need to have moist, warm saline packs applied to the exposed bowel until the patient can return to the operating room for emergent repair.

A patient presents to the emergency department with acute abdominal pain. Which of the following descriptions of the patient's pain would be consistent with a diagnosis of diverticulitis? A. Burning substernal pain after meals B. Severe, diffuse ache in the periumbilical region C. Steady ache in the left lower quadrant with referral to the back D. Steady, boring epigastric pain with referral to the back E. Sudden, severe pain in the lower quadrant with referral to the flank

C. Diverticulitis commonly produces a steady, aching pain, localized to the left lower quadrant of the abdomen, with referral to the back in some cases. In addition, a tubular mass (caused by inflammation) may be appreciated on abdominal examination, and the patient may be febrile with an increased white count. Remember, if a patient presents with symptoms and signs similar to appendicitis, but the complaints concern the left rather than the right side of the abdomen, think of diverticulitis as a likely diagnosis. Also, take note that diverticulosis, not diverticulitis, is the most common cause of massive lower gastrointestinal bleeding in adults. Burning substernal pain after meals (choice A) is suggestive of reflux esophagitis. The pain may be referred to the left arm. Severe, diffuse ache in the periumbilical region (choice B) is suggestive of bowel infarction, although it must be distinguished from the early pain of appendicitis. The pain of bowel infarction is often aching and severe, whereas appendicitis can (but doesn't always) produce crampy pain. Steady, boring epigastric pain with referral to the back (choice D) and little relation to meals may indicate pancreatitis. Nausea and vomiting frequently accompany the pain. Sudden, severe, left lower quadrant pain described as tearing or ripping with referral to the flank (choice E) may indicate a distal dissecting aortic aneurysm. The pain may also be periumbilical. Review Symptomatic diverticulitis presents with fever, leukocytosis, and left lower quadrant pain. It is caused by overactivity of colonic smooth muscle that results in herniation of mucosa and submucosa through the muscle layers of the colon, which form a diverticulum. Diverticulitis is inflammation of the diverticulum, which may be caused by infection.

A 63-year-old man in apparent good health is convinced by magazine articles and TV programs that he will live longer if he takes one aspirin tablet every day. After about 3 weeks of doing so (325 mg/day), he begins to notice bright red blood on the toilet paper when he wipes after a bowel movement. This does not occur every time, but only when, for whatever reason, he has to strain more than usual. He has never had any discomfort referable to hemorrhoids or any bowel pathology that he is aware of. Anoscopy and digital rectal examination show external and internal hemorrhoids, none of which are bleeding at the time of the examination. Which of the following is the most appropriate next step in management? A. Discontinue the aspirin B. Perform rubber-band ligation of the internal hemorrhoids C. Perform 60-cm flexible proctosigmoidoscopy D. Recommend stool softeners E. Reduce the aspirin dosage

C. Normal colonic mucosa does not bleed in response to anticoagulation, but lesions with a predisposition to do so may be more likely to become symptomatic at such time. The bleeding lesion in this case may indeed be the internal hemorrhoids that were diagnosed, but a malignant source has to be ruled out before making such assumption. At the very least, the patient needs a 60-cm examination with a flexible endoscope. If one of the answer choices was colonoscopy, that would be the next best step, because this allows visualization of the entire colon. Because the patient is older than age 50 years, a colonoscopy is warranted as part of the evaluation for potential colon cancer. Discontinuing the aspirin (choice A) or reducing the dosage (choice E) might indeed get rid of the bleeding, but in doing so we may miss the opportunity of diagnosing a potential malignancy that had been asymptomatic until now. There is little evidence that this bleeding is related to aspirin, especially in the setting of the patient not having melena caused by stomach irritation or gastritis. Rubber-band ligation (choice B) may indeed be done eventually if there are no other lesions and one wishes to eliminate the internal hemorrhoids. Bleeding caused by hemorrhoids should be a rule out diagnosis when another potentially serious lesion has been successfully eliminated. Stool softeners (choice D) could also help if the hemorrhoids are not removed and the patient wishes to continue taking aspirin, but as pointed out, all these decisions have to follow after cancer has been ruled out. Review After the age of 50, screening recommendations include a yearly fecal occult blood test and flexible sigmoidoscopy every 5 years or colonoscopy every 10 years. Colon cancer is a common malignancy in this country and this condition often has a long latency period for detection and early intervention.

A 9-year-old boy is brought to the emergency department by his grandmother because he suddenly developed a fever with shallow breathing, vomiting, and mental confusion. She is beside herself because the child has come to visit over the holidays and had been perfectly well before today. He has never had any problems with his health except for a skin condition present from birth that she describes as "alligator skin." When you ask if she gave her grandson any unusual medication or remedy, she admits to having applied a homemade paste to his skin that was supposed to get rid of the heavy scale. She states that there were several affected family members in previous generations and this remedy has been used to treat them without ever having caused problems. She also insists that it could not have been the cause of any illness, because it was really just a bunch of aspirin tablets crushed and mixed with petrolatum. The boy's temperature is 39ºC (102.2ºF), pulse 120/min, and respirations 35/min. He has vomited twice while in the emergency department and appears to be dehydrated and lethargic. Blood glucose is 45 mg/dL and serum pH is 6.9. Which of the following is the most appropriate next step? A. Activated charcoal by mouth B. Gastric lavage C. Intravenous bicarbonate D. Intravenous 5% dextrose in saline E. Oral N-acetylcysteine

C. Salicylates have analgesic and antipyretic properties when used systemically, and keratolytic properties when used topically. This patient has developed salicylate intoxication secondary to absorption of large amounts of salicylate through the skin. Many skin conditions that are characterized by hyperkeratosis are treated with topical preparations that contain a certain amount of salicylic acid (ranging from 1 to 40%, depending on the indication). They should never be used in children, however, because the surface-to-volume ratio is very large and even a relatively small amount absorbed through a large surface area will result in significant systemic levels. Even in adults, preparations with a significant amount of salicylic acid should be used cautiously, and either for a very limited time or only on certain body surface areas at a time. The clinical presentation and physical findings will vary with the dose of salicylates ingested. Vomiting, hyperpnea, fever, lethargy, and mental confusion are seen in mild salicylate poisoning. Convulsions, coma, and respiratory and cardiovascular collapse are seen in severe salicylate poisoning. Hyperventilation, dehydration, bleeding disorders, seizures, and coma are seen in chronic salicylate ingestion. Arterial blood gas analysis will show metabolic acidosis with respiratory compensation in children and a respiratory alkalosis alone in adolescents. To enhance the excretion of salicylate, the intravenous route should be used to administer bicarbonate and raise the urine pH to 7.0-7.5. If the salicylate level is greater than 100 mg/dL, hemodialysis may be indicated. Activated charcoal by mouth (choice A) and gastric lavage (choice B) would be indicated if the poisoning had occurred after oral ingestion of salicylates. Because the intoxication followed absorption through the skin, however, these options would not be of help in this patient. Intravenous 5% dextrose in saline (choice D) is not the intravenous fluid of choice in patients who have salicylate intoxication. It would not alkalize the urine to promote excretion of the salicylate. Oral N-acetylcysteine (choice E) is used in the treatment of acetaminophen poisoning and is not indicated in this patient. Review Salicylate poisoning is managed with: Gastric lavage with suitable airway protection should be considered up to 24 hours after ingestion. Give activated charcoal. Administer aggressive IV fluids with IV bicarbonate to enhance the excretion of salicylate (increase the urine pH to 7.0-7.5).

A 69-year-old woman comes to the health care provider for a periodic health maintenance examination. She has no complaints. She takes 1,500 mg of calcium daily, a multivitamin, and daily aspirin. She exercises regularly, eats a fairly healthy diet, and does not smoke cigarettes. She drinks a couple of glasses of wine with dinner each evening and smokes marijuana occasionally with a friend who has glaucoma. She is concerned about her risks for breast, ovarian, and colon cancer because she has friends with each of these diseases. Breast and pelvic examination are unremarkable. A mammogram is normal. Findings from a barium enema are shown. Which of the following is the most common presenting symptom of this condition? A. Anemia B. Blumer shelf C. Constipation D. Virchow node E. Weight loss

C. The barium enema demonstrates an irregular lesion that causes obstruction in the sigmoid colon. This finding on barium enema is highly suspicious for adenocarcinoma. Cancers arising in the left and sigmoid colon are characterized by rectal bleeding, obstructive symptoms, and constipation possibly alternating with diarrhea. Stools may develop a narrow caliber as the tumor encroaches onto the lumen. Although anemia (choice A), Blumer shelf (metastasis to the pelvic cul-de-sac) (choice B), Virchow node (palpable lymph node in the left supraclavicular space) (choice D), and weight loss (choice E) can be associated with cancers of the left and sigmoid colon, they are more commonly associated with highly advanced colon cancer. Review Remember the most common signs for colon masses; some of them will depend on the location of the mass. A growing mass in the left colon is most likely to produce obstruction to the passage of feces, and thus, likely to present with constipation alternating with diarrhea or bloody bowel movements. A mass in the right side of the colon is most likely to present with chronic blood loss and anemia.

A 32-year-old woman has had a 15-year history of heartburn. Over the past 4 months, she has had difficulty swallowing large bites of solid food. She has no difficulty with soft foods or liquids, and she has not lost weight. Which of the following is the most likely explanation for her symptoms? A. Adenocarcinoma in the lower third of the esophagus B. Barrett's esophagus in the distal esophagus C. Fibrosis and narrowing at the distal esophagus D. Schatzki ring in the distal esophagus E. Squamous carcinoma in the mid-third of the esophagus

C. This patient has classic symptoms of mechanical dysphagia, as she has difficulty with large solid food but not softer foods or liquids. Mechanical dysphagia frequently follows many years of heartburn and is often indicative of a peptic stricture that has developed as a result of fibrosis after a long period of chronic inflammation caused by gastroesophageal reflux disease (GERD). These benign strictures can usually be dilated endoscopically. An intensive regimen of proton pump inhibitors should then be instituted to reduce the frequency of recurrence. This patient should be evaluated endoscopically to rule out Barrett's esophagus, which may occur with long-term untreated GERD. Although chronic acid reflux can predispose for Barrett's esophagus (choice B) and subsequently adenocarcinoma (choice A), Barrett's esophagus is a mucosal change only that would not cause lumenal narrowing. Furthermore, adenocarcinoma would be very unusual in a patient this young. Schatzki ring (choice D) is unlikely, because it typically produces episodic mechanical dysphagia rather than the progressive mechanical dysphagia described in this question. Squamous carcinoma (choice E) in the mid-third of the esophagus can produce mechanical dysphagia. This patient is far younger than the usual patient who has squamous carcinoma, however, and she has no risk factors, such as smoking, drinking, lye ingestion, or upper esophageal web that may be associated with iron deficiency anemia (Plummer-Vinson syndrome). Review Peptic stricture formation caused by fibrosis and narrowing of the distal esophagus should be suspected in those with a history of chronic GERD and new-onset dysphagia for solids but not liquids. Endoscopy will diagnose and simultaneously treat the condition through dilation.

A 20-year-old man is brought to the emergency department 3 hours after ingesting 50 tablets of 325-mg acetaminophen in a suicide attempt. He has nausea and vomiting, but no other abnormalities on physical examination. His temperature is 37ºC (98.6ºF), blood pressure 135/80 mm Hg, pulse 100/min, and respirations 20/min. Serum acetaminophen concentration is within the range of probable toxicity but serum transaminases and other hepatic markers are normal. Which of the following is the most appropriate next step in management? A. Activated charcoal treatment B. Acetylcysteine treatment C. Penicillamine treatment D. Activated charcoal and acetylcysteine treatment E. Gastric lavage

D. Acetaminophen overdose may result in massive hepatocellular necrosis and fulminant hepatic failure. Liver metabolism of acetaminophen by P-450 cytochrome produces a highly toxic intermediate, which depletes glutathione stores, resulting in accumulation of free radicals. In the first few hours after ingestion, the patient may have nausea and vomiting, but no signs of liver failure. Biochemical evidence of hepatic damage, namely elevation of transaminases, begins 24-48 hours after ingestion. Therapy, therefore, should be started before hepatic transaminases begin to increase. The likelihood of hepatic toxicity is based on a combination of serum acetaminophen levels and time elapsed from ingestion. Nomograms have been developed to determine the likelihood that a specific level of acetaminophen at a given time following ingestion will produce hepatotoxicity. Within 4 hours following ingestion, the initial treatment should include activated charcoal (to adsorb residual drug in the intestine) and N-acetylcysteine. Treatment with N-acetylcysteine is most effective if started within 10 hours after ingestion. The FDA-approved recommendation is to continue oral treatment for 72 hours. Activated charcoal treatment (choice A) alone is not sufficient to prevent the toxic effects of acetaminophen that has already been absorbed. Charcoal adsorbs almost all drugs and poisons in the gut. It is effective in the first few hours after acetaminophen ingestion and does not interfere with administration of acetylcysteine. Acetylcysteine treatment (choice B) is used to counteract hepatic toxicity of acetaminophen. In the first few hours after ingestion, it should be combined with administration of activated charcoal to prevent further intestinal absorption of the drug. Penicillamine (choice C) is a chelator used for treatment of acute arsenic poisoning. Penicillamine should be administered after dimercaprol (BAL). Gastric lavage (choice E) is effective within approximately 1 hour of ingestion of poisons or drugs. It is indicated when induced emesis is refused, unsuccessful, or contraindicated. Review Treatment of acetaminophen overdose is determined by the time since ingestion and the serum levels of acetaminophen, which are drawn serially. If the patient presents within 4 hours of ingestion of acetaminophen, treatment consists of N-acetylcysteine with or without an antiemetic, activated charcoal, and evaluation for possible liver transplantation.

A patient with a history of hypertension calls his health care provider's office for advice. He has had long-standing heartburn and recently consulted with a gastroenterologist. He underwent an endoscopy and was told that "Barrett mucosa" was found by biopsy. The patient has read in the newspaper that people with this condition will probably develop esophageal cancer. Which of the following is the most appropriate response to this concern? A. "Your concerns are ungrounded." B. "It is foolish to worry because this type of cancer is unlikely to develop and would occur many years later." C. "You should chew food very carefully to prevent the possibility of a mechanical obstruction." D. "Only a small minority of patients with Barrett's esophagus will develop cancer, but you should undergo endoscopic surveillance." E. "You should consult with an oncologist regarding esophageal cancer prevention strategies."

D. Barrett's esophagus may occur in a small number of patients who have gastroesophageal reflux disease (GERD). This condition is a metaplasia of the normal squamous mucosa of the esophagus to a columnar (glandular) type of epithelium, and is usually seen after repeated acid exposure to the distal esophagus. Tobacco and alcohol use are also thought to contribute to the process but not to the same extent that they contribute to squamous cell esophageal cancer. This condition is most commonly seen in middle-aged Caucasian men who have a long-standing history of reflux. The significance of Barrett's esophagus is that it may lead to the development of low- or high-grade dysplasia, or esophageal adenocarcinoma. This is a very infrequent occurrence when considering the large number of patients who have GERD, however, and even those who have Barrett's esophagus. Barrett's esophagus usually does not resolve with either medical or surgical therapy. Endoscopic surveillance (with multiple small biopsies, because dysplasia cannot be reliably evaluated by endoscopic appearance alone) every 1-2 years has been often recommended, but some studies suggest that it may not be cost effective. It is not factually true to inform the patient that his concerns are "ungrounded" (choice A), because there is in fact a small risk for adenocarcinoma. It is never appropriate to belittle a patient's concerns and inform him that his worries are "foolish" (choice B). Barrett's esophagus is a histologic change and, unless accompanied by a stricture, does not produce symptoms of mechanical dysphagia (choice C). It is inappropriate to refer the patient to a cancer specialist (choice E) for the prevention of a very unlikely development of cancer; furthermore, preventive strategies should include instructions to avoid factors that exacerbate GERD and should be delivered to the patient by the health care provider. Review Barrett's esophagus may occur after long-standing damage to the distal esophagus, typically caused by ongoing gastroesophageal reflux disease. Complications include esophageal stricture and a potential for adenocarcinoma. Treatment involves acid suppression with a proton pump inhibitor plus surveillance about every 2 years. If acid suppression is not effective, radiofrequency ablation can be performed along with antireflux surgery with or without radiofrequency. Treatment has not been proven to prevent progression of the disease but is typically performed to help with symptoms.

A 42-year-old alcoholic man comes to the emergency department with severe abdominal pain that began 3 hours ago. He is well known to the medical staff because of 3 prior hospitalizations for acute pancreatitis. He says that this time his pain began suddenly and in short order became constant, generalized, and extremely severe. He lies motionless on the stretcher, and on physical examination he has tenderness, rigidity, and rebound in all four abdominal quadrants. Bowel sounds are absent. Initial studies are significant for a serum amylase of 1,200 U/L and free air under both diaphragms on an upright plain film of the abdomen. Which of the following is the most likely diagnosis? A. Acute edematous pancreatitis B. Acute hemorrhagic pancreatitis C. Gangrenous acute cholecystitis D. Perforated viscus E. Ruptured pancreatic pseudocyst

D. The clinical picture is highly suggestive and the radiograph findings virtually pathognomonic for a perforated viscus. Alcoholics can easily get peptic ulcer disease (which can perforate), and perforated ulcers are well known for their propensity to produce elevations of the serum amylase. Patients who have a perforated viscus will classically lie motionless and have a rigid, quiet abdomen on physical examination. The next step in any patient with a perforated viscus is emergency laparotomy. Pancreatitis, whether edematous (choice A) or hemorrhagic (choice B), could indeed account for abdominal pain, but it would have had gradual onset, be mostly in the upper abdomen with radiation straight through to the back, and be accompanied by rather modest physical findings. The high amylase would have fit, but not the free air under the diaphragms, because the pancreas is a solid organ that would not release air if perforated. Gangrenous cholecystitis (choice C) is not correct. Although it is true that the gallbladder could have perforated and led to generalized peritonitis, it would have been far more likely to have findings confined to the right upper abdomen. Besides, a gangrenous gallbladder is full of pus and stones but not air, and this condition would therefore not produce the radiograph described. The same is true of pancreatic pseudocyst (choice E). It could produce severe generalized chemical peritonitis when it ruptures, but no air would have escaped from it. Review Suspect perforated viscus when there is sudden onset of abdominal pain and acute abdomen. Free air under the diaphragm seen on upright film makes the diagnosis. Immediate laparotomy is required for treatment.

A 5-month-old boy is brought to the office because of frequent watery stools and vomiting present for the past 2 days. The child has been in daycare since age 3 months and has had several episodes of fever and runny nose in the meantime, but this is the first time he has had diarrhea. It started approximately 48 hours earlier with the passage of six to seven loose, watery stools with a foul smell. He had also vomited a couple of times and has had fever since the beginning of the symptoms. The parents had called the daycare to see if other children were affected, but learned that no one else had similar symptoms. They had not traveled anywhere since the child was born. On physical examination, the child is in moderate distress, with a temperature of 38.1ºC (100.6ºF), a pulse of 120/min, and respirations of 26/min. He has dry mucosa and his anterior fontanelle is sunken. Skin turgor is also diminished. Palpation of the abdomen does not reveal tenderness. On auscultation, increased peristalsis can be heard. The stool is guaiac-positive for occult blood but shows no visible gross blood on examination. He is given intravenous rehydration in the office and sent home on supportive treatment with lots of fluids and antipyretics as needed. Two days later, the mother calls the office to say that all the symptoms have completely resolved and the child is feeling well and behaving normally. Which of the following was the most likely cause of this patient's illness? A. Clostridium difficile B. Clostridium perfringens C. Enterotoxigenic Escherichia coli D. Rotavirus E. Staphylococcus aureus

D. Diarrhea is defined as an increased stool output, with excess loss of fluid and electrolytes. It can be classified as acute or chronic. Several mechanisms exist that cause diarrhea, of which more than one may be present: Causes of acute and chronic diarrhea are age-dependent. Acute diarrhea is almost always infectious, with gastroenteritis the most common cause in any age group. Food poisoning, systemic infections, parasitic infections, and antibiotics are other causes. Viral agents are the most common cause of acute diarrhea in children, and rotavirus is the most common viral cause of diarrhea in the winter. Enteric adenovirus and Norwalk virus are also common causes of diarrhea. Bacterial causes include E. coli, Salmonella, Shigella, Campylobacter, Yersinia, and Clostridium. Parasitic causes include Entamoebahistolytica, Giardia, and Cryptosporidium. Clinically, rotavirus infection presents with watery diarrhea that can last up to 7 to 10 days. It may be accompanied by 3 to 4 days of vomiting. Fever may be present. The stool may be positive for occult blood. There is no abdominal tenderness associated with rotavirus infection. C. difficile (choice A) enterocolitis is most commonly seen in hospitalized patients on antibiotics. The incubation period and duration of symptoms are variable. It may be associated with fever and abdominal pain, but there is no vomiting. C. perfringens (choice B) infection has a minimal incubation period and duration of up to 24 hours. It may be associated with mild vomiting and abdominal pain, but no fever. This condition is typically caused by ingestion of pre-formed spores that are not destroyed with reheating. The classic food that is associated with this pathogen is reheated roast beef that was inadequately refrigerated after cooking. Enterotoxigenic E. coli (choice C) is the cause of traveler's diarrhea. The incubation period is 1 to 3 days and the duration is 3 to 5 days. The patient often has vomiting, low-grade fever, and abdominal pain. S. aureus (choice E) is characterized by onset within 12 hours of ingestion of contaminated food and a common source for the outbreak. Review Viruses account for about 70 to 80% of episodes of acute gastroenteritis in children, with rotavirus being the most common identifiable cause of gastroenteritis in children. Rotavirus is commonly spread in the daycare setting. Rotavirus is ubiquitous and almost all children are affected by 3 years of age. Spread is via fecal-oral route, but recent studies have suggested that rotavirus may be spread by respiratory droplets. Treatment is conservative, with maintenance of hydration being the most important element of care. Supporting the patient's hydration status until the virus is spontaneously cleared is the management of choice. Rotavirus infection can cause simultaneous nausea, vomiting, and diarrhea along with a low grade fever that may be caused by the dehydration that rapidly occurs. Strict hand washing techniques may help to limit the spread of this virulent pathogen.

A 24-year-old graduate student is found to have acute myelogenous leukemia (AML). Before undergoing a planned bone marrow transplant, he begins aggressive multi-drug chemotherapy. His tolerance of this regimen is severely limited, however, by intractable vomiting with each chemotherapy cycle. Which of the following will most likely reduce vomiting in this patient? A. Dopamine B. Erythromycin C. Omeprazole (Prilosec) D. Ondansetron (Zofran) E. Opiates

D. Ondansetron (Zofran), a 5-hydroxytryptamine3 antagonist, is the most potent antiemetic available for chemotherapy-induced vomiting. It is safe and is available as an oral (including dissolving tablet) or IV medication. Ondansetron has greatly improved the symptoms of many patients who were previously unable to tolerate full-dose chemotherapy because of severe vomiting. It has side effects only infrequently, the most common being constipation. Dopamine (choice A) is actually a pro-emetic compound. The common mediators of vomiting in the CNS are 5-hydroxytryptamine, dopamine, and acetylcholine. Erythromycin (choice B) also tends to cause, not relieve, nausea and vomiting. It has motilin-stimulating properties in the stomach, leading to increased gastric motility. It is of therapeutic value in patients who have diabetic gastroparesis. Omeprazole (choice C) is of no value in inhibiting vomiting. It is a proton pump inhibitor, which is effective in reducing or eliminating acid secretion. Opiates (choice E) are commonly used in patients receiving chemotherapy but are actually pro-emetic, because they stimulate the chemoreceptor trigger zone, which lies within the brain but outside the blood-brain barrier. Review 5-HT3 receptor antagonists are extremely effective anti-emetic agents used for prevention and treatment of chemotherapy-induced emesis. Their mechanism of action is that they selectively antagonize serotonin 5-HT3 receptors. Examples include dolasetron (Anzemet), granisetron (Kytril), and ondansetron (Zofran).

A 24-year-old man comes to the student health clinic with complaints of chronic epigastric pain 1 week before taking final exams. The patient has been awakened from sleep by epigastric pain at approximately 3 AM on several occasions. Ranitidine (Zantac) has improved his symptoms transiently. Eating generally improves his symptoms for about 30 minutes, but it is then followed by increased pain. Which of the following is the most likely cause of his symptoms? A. Diffuse gastric mucosal vasoconstriction B. Gastrin-producing tumor C. Infection or organism invading the antral mucosa D. Organism colonizing the gastric antrum E. Reduced gastrin release

D. Peptic ulcer disease is strongly suggested by this patient's chronic epigastric pain that is severe enough to awaken him at night and is temporarily relieved with ranitidine and eating. In the absence of nonsteroidal anti-inflammatory drug (NSAID) use, the most likely etiology is antral colonization with Helicobacter pylori. This gram-negative organism lives in the mucous layer that lines the stomach and apparently uses the mucus as its food source. The result is focal disruption of the mucosal barrier, predisposing for both chronic gastritis and gastric peptic ulcer formation. In the past, medical personnel tended to blame ulcer formation on "stress," with the implication that correcting the problem involved teaching the patient to be more calm. Now that we more fully understand peptic ulcer disease, we treat the bacterial colonization as part of our primary therapy. Diffuse gastric mucosal vasoconstriction (choice A) describes the pathogenesis of stress-induced gastritis. This may be a preventable condition; all patients admitted to the intensive care unit of a hospital should have preventive therapy with acid suppression to prevent this condition. Gastrin-producing tumors (choice B) can cause Zollinger-Ellison syndrome, in which the increased circulating gastrin markedly increases gastric acid production. These tumors are rare, and patients who have them typically have a history of multiple peptic ulcers or ulcers located in atypical locations, often with complications such as perforation or massive bleeding. This patient's history of symptoms that have worsened during stress is more suggestive of the more common form of peptic ulcer disease related to stress and H. pylori colonization. The H. pylori organisms that are associated with peptic ulcer disease do not actually invade the antral mucosa (choice C). Increased, but not decreased (choice E), gastrin release, as described above with respect to Zollinger-Ellison syndrome, may cause peptic ulcers. Review The most common etiology for duodenal ulceration is infection with H. pylori. Patients who have duodenal ulcer tend to have worsening of their symptoms with fasting and improvement of their symptoms with eating. Treatment of this condition involves eradication of H. pylori.

An 8-month-old boy is brought to the office for a regular checkup. He is developing well and does not seem to have any particular problems. He is still breastfeeding but has started solids at 5 months of age. He enjoys strained vegetables and fruits, cottage cheese, and soft meat. The mother thinks that he will soon wean off the breast milk because he likes "real" food very much. She is interested in recommendations as to introducing other new foods in the months to come, and expects him to be eating almost everything by his first birthday. On physical examination, the boy is sitting contently in his mother's lap and responds appropriately to visual and auditory stimuli. He is in the ninetieth and ninety-fifth percentiles for height and weight, respectively. The immunizations are up to date. Which of the following foods should be avoided before the boy is 1 year old? A. Beans B. Egg yolk C. Fish D. Honey E. Iron-fortified cereals

D. Solid foods can be introduced at 4 to 6 months of age in infants, whether breast- or formula-fed. Before this time the risk for atopy is increased and infants are not developmentally ready to take solids. New foods should be introduced one at a time, with a several-day interval in between, and in small amounts so that food intolerances can be identified. Honey is to be avoided in the first year of life, because there is an increased risk for infantile botulism. Food introduction according to age is approximated in the following table. Beans (choice A) can be slowly introduced when the infant is 6 to 8 months old. Egg yolk (choice B) may be introduced at the age of 6 to 8 months. Egg whites should be avoided during the first year of life because of their propensity to cause an allergic reaction. Fish (choice C) is introduced when the infant is 7 to 9 months old. Extra care should be taken to remove even the smallest fish bones before feeding the infant. Iron-fortified cereals (choice E) are introduced at age 4 to 6 months and are the first solids added to mother's milk or formula. In poor families, iron supplementation should be started at 6 weeks of age. Review The intake of honey should be avoided in all infants before 1 year of age. Infants younger than 12 months do not have the capability of handling the spores that may cause clostridial botulism infection because of their immaturely functioning gut. Cow's milk should also be avoided before 1 year of age because of its tendency to cause gastritis and irritation in the stomach that can lead to GI blood loss.

Three months after a needle-stick exposure to blood from a patient with hepatitis B, a nurse is evaluated for infection with the virus. Laboratory results reveal: HBsAG - absent anti-HBs antibody - absent IgM anti-HBc - present IgG anti-HBc - absent HBeAg - absent On the basis of these results, which of the following most accurately describes the nurse's hepatitis B status? A. She had been effectively vaccinated against hepatitis B before the needle-stick exposure occurred. B. She has mounted an inappropriate antibody response to hepatitis B as a result of an immunocompromised state. C. She is a carrier of hepatitis B. D. She is actively infected with hepatitis B. E. She was not infected with hepatitis B.

D. The nurse's elevated IgM anti-HBc indicates she was infected with hepatitis B. Formerly HBsAg (surface antigen) and anti-HBsAg (antibody to surface antigen) were used exclusively to determine this. Typically HBsAg is positive for up to 6 months, and anti-HBsAg is positive for years after that. Unfortunately this simple scheme has the disadvantage that many patients have a 2-week to 4-month window period, when the surface antigen (HBsAg) and the antibody (anti-HBs) are not detectable. Presumably, for a relatively brief period, HBsAg production exactly matches antibody production, and the two co-precipitate such that neither free species is present in adequate concentration to be detectable. This problem can be circumvented either by serial measurements of HBsAg and anti-HBs, or by concurrent measurement of other antigens and antibodies, including HBeAg, anti-HBe, and anti-HBc (HBcAg is not reliable). During the core window period, IgM anti-HBc may be the only marker of recent HBV infection, as it is in this nurse. If the nurse had been effectively vaccinated for hepatitis B (choice A), she would have had an elevated anti-HBs antibody level and no HBsAg present in the serum. Anti-HBc antibody would have been absent as well. The negative HBeAg suggests that the patient does not have active replication of the virus at this time, as this "envelope" level parallels the active replication of the virus in the body. The antibody response to hepatitis B infection was appropriate in this person, which argues against immunocompromise (choice B). Hepatitis B carriers (choice C) have elevated HBsAg and may have persistently elevated HBeAg (in approximately 10% of cases). IgG anti-HBc (not IgM) predominates in these chronic patients. Had she not been infected with hepatitis B at all (choice E), IgM anti-HBc would be absent. Review Patients who have hepatitis B will have measurable markers in their blood, which determine the status with regard to infectivity. Patients who are successfully immunized against hepatitis B will have +HBsAb. Patients who have recent infection will develop HBsAg. If the patient becomes infected with HBV and goes on to clear the infection (spontaneously or with treatment), the patient will undergo a change from +HBsAg to negative status with development of +HBsAb. True infection will cause +HBcAb formation, first with IgM (recent), followed by IgG becoming positive. The IgM core antibody may be the only marker while the patient is recovering from the hepatitis B infection, and this is known as the core window. Vaccination will not cause any reaction to the core. HBeAg is a marker of active viral replication and a marker for how potentially infectious the person is if someone had a blood exposure to that patient.

A previously healthy, intoxicated 19-year-old man is driving a car without using a seat belt. He crashes the car into the back of a parked truck. In the process he slams his abdomen into the steering wheel and ruptures his spleen. Which of the following is the most important problem associated with this type of injury? A. Bacteremia B. Electrolyte abnormalities C. External blood loss D. Internal blood loss E. Peritonitis

D. The spleen is a highly vascularized organ and is vulnerable to traumatic rupture. This can occur "spontaneously" (i.e., with minimal trauma, such as falling against a table or even overly vigorous palpation during a physical examination) in patients who have an enlarged spleen caused by disease (e.g., leukemia, mononucleosis, autoimmune diseases with red blood cell sequestration in the spleen, or as a complication of portal hypertension). Alternatively, splenic rupture can occur in previously normal individuals who have severe trauma to the abdomen. In either case, the heavily vascularized spleen is usually unable to stop (often massively) bleeding internally. Emergency splenectomy is indicated to control the bleeding. Bacteremia (choice A) and peritonitis (choice E) are much less of a risk in splenic rupture than in rupture of a hollow viscus, such as the colon, because the spleen is usually sterile. If the spleen is removed, the patient will need to have immunization against the gram-negative enteric organisms of H flu, strep pneumo, and meningitis. Electrolyte abnormalities (choice B) can develop secondarily to the ischemia produced by severe blood loss; these are much less critical than the blood loss itself and will often correct spontaneously with adequate replacement of blood. External blood loss (choice C) is often insignificant in injuries such as this. The primary problem that must be addressed is the internal injuries that can occur in the setting of ruptured spleen. Review Patients who sustain abdominal trauma may have injury to the spleen and other viscera as a result of the trauma. Patients will have left upper quadrant abdominal pain or pain in the left shoulder because of the common innervations between the spleen and left shoulder area. This is a highly vascular organ that may cause significant blood loss if insulted. Patients will have tachycardia, hypotension, and tachypnea as a result of the blood loss, which may be massive. Abdominal CT scan is the test of choice to evaluate the extent of trauma. Surgical intervention with splenectomy may be needed in emergent situations. For slow bleeds that do not resolve on their own, endovascular embolization may be attempted to interrupt blood to the splenic artery.

A 58-year-old man with known hepatitis C and cirrhosis complains of worsening fatigue and confusion over the past 5 days. He has been admitted 3 times in the past 4 months for variceal bleeding and has had ascites that has been refractory to high-dose oral diuretic use. He also reports that over the past 48 hours he has had a declining urinary output. On physical examination, he is gaunt and jaundiced. He has tense ascites and a liver span of 7 cm in the midclavicular line. Laboratory results reveal a white blood cell count of 4,600/mm3 (normal 5,000-10,000/mm3), hemoglobin 9.4 g/dL (normal 13.8-17.2 g/dL), and hematocrit 29% (normal 41-50%). His electrolytes reveal BUN 34 mg/dL (normal 7-20 mg/dL) and creatinine 3.1 mg/dL (normal in men 0.8-1.4 mg/dL). Urinary sodium is less than 10 mEq/L (normal spot sodium >20 mEq/L). Which of the following is the most appropriate treatment for his elevated BUN and creatinine? A. Hemodialysis B. Kidney transplantation C. Large volume paracentesis D. Liver transplantation E. Mesocaval shunt

D. This patient who has well advanced cirrhosis and portal hypertension has developed the onset of renal insufficiency consistent with hepatorenal syndrome. This occurs during the end stages of cirrhosis and is characterized by diminished urine output and low urinary sodium. In the setting of end-stage liver disease, renal vasoconstriction occurs, and the distal convoluted tubule responds by conserving sodium. Unless the renal function is allowed to deteriorate further, liver transplantation will reverse this vasoconstriction and kidney function will return to normal. There are no indications in this question to suggest that the patient requires acute hemodialysis (choice A). Renal transplantation (choice B) is of no value in this patient, because the underlying lesion is in the liver; the kidneys will return to normal function if there is improvement in hepatic function. Large volume paracentesis (choice C) may relieve the ascites but will have no significant benefit on the impaired renal function. A mesocaval shunt (choice E) is a surgical procedure which may decompress the portal pressure but will not affect renal function. Review Hepatorenal syndrome refers to unexplained renal failure in patients who have liver disease or those who have biliary tract obstruction. The kidneys are morphologically normal. It is often irreversible and is rapidly fatal. The only effective management is liver transplantation. Dialysis and transjugular intrahepatic portosystemic shunts are used temporarily until a liver transplant is performed.

A 53-year-old woman comes to the health care provider complaining of fatigue over the past 6 months. During this time, she has also developed pruritus and lost 4 pounds. She is not sexually active and her past medical history is significant only for Sjögren syndrome. On physical examination she is afebrile and has mildly icteric sclera. There are excoriations noted on all 4 extremities and trunk and back. The liver edge is smooth and non-tender and measures 9 cm at the midclavicular line. There is no ascites, splenomegaly, or peripheral edema. Laboratory results reveal a normal complete blood count, normal electrolytes, liver function tests revealing an alkaline phosphatase of 260 U/L (normal <110 U/L), total bilirubin of 3.1 mg/dL (normal <1.9 mg/dL), and normal transaminase levels. Which of the following is the most likely diagnosis? A. Acute cholecystitis B. Acute hepatitis A infection C. Bacterial cholangitis D. Primary biliary cirrhosis E. Primary sclerosing cholangitis

D. This woman has a classic presentation of primary biliary cirrhosis. It typically affects middle-aged women and will progress gradually to the point of end-stage liver disease over a number of years. The disease is caused by an autoimmune destruction of intrahepatic bile ducts, and the diagnosis is made by liver biopsy. The serology that should be checked is the antimitochondrial antibody. Primary biliary cirrhosis is often seen in individuals who have other autoimmune diseases, such as Sjögren syndrome, pernicious anemia, and Hashimoto thyroiditis. Acute cholecystitis (choice A) presents acutely with right upper quadrant pain and fever and not with chronic fatigue and pruritus. Acute hepatitis A (choice B) may cause jaundice and fatigue, but it is a self-limiting infection and does not last 6 months. Cholangitis (choice C) is caused by acute obstruction of the common bile duct and presents urgently with fever, right upper quadrant pain, and jaundice (Charcot triad). Primary sclerosing cholangitis (choice E) is a sclerosing process of the extra- and intrahepatic ducts, which usually presents in young males who have underlying inflammatory bowel disease. Review Primary biliary cirrhosis most commonly occurs in women over age 45. It is a progressive disease that classically presents with pruritus and fatigue. The pruritus occurs as a result of a toxic accumulation of bile salts retained in the liver, which also leads to progressive liver damage and cirrhosis. Autoantibodies in the form of antimitochondrial antibodies are seen in this autoimmune disease. Treatment is bile salt therapy (ursodiol or ursodeoxycolic acid), which can slow or stop the progression to advanced liver disease, and cholestyramine which controls the itching. End-stage liver disease can be treated only with liver transplantation.

A 41-year-old gas station attendant is shot once with a .38-caliber revolver. The entry wound is in the left midclavicular line, 2 inches below the nipple. There is no exit wound. He is hemodynamically stable. Chest radiograph shows a small pneumothorax on the left and demonstrates the bullet to be lodged in the left paraspinal muscles. In addition to the appropriate treatment for the pneumothorax, which of the following will this patient most likely need? A. Barium swallow B. Bronchoscopy C. Extraction of the bullet via local back exploration D. Extraction of the bullet via left thoracotomy E. Exploratory laparotomy

E. Although this vignette describes a gunshot wound of the chest, we must remember that the chest and the abdomen are not stacked up like pancakes. There is a dome - the diaphragm - that separates them, and thus an area where chest and abdomen overlap. Any gunshot wound below the nipples involves the abdomen, and such is the case here. The management of all gunshot wounds of the abdomen requires exploratory laparotomy. The surgical exploration will identify vascular sites of injury along with any visceral damage, with correction occurring at the time of the exploration. Barium swallow and bronchoscopy (choices A and B) are indicated if there are signs suggestive of injury to those organs (coughing up blood, spitting up blood), or if the anatomic trajectory of the bullet puts the track in their vicinity. Here, we have an entry wound on the left and a bullet lodged on the left: the midline has not been crossed. Taking out the bullet (choices C and D) is unnecessary if the missile is not pressing on some vital structure. A bullet embedded in a muscle can be left there. Review Gunshot wounds can have significant impact on the anatomic structures that are traumatized by them. When a gunshot wound affects the abdomen, surgical exploration is needed (laparoscopically or through an open procedure) to identify the sites of injury and to correct the injured areas. Disruption of the bowel will cause massive contamination. Disruption of the vascular structures and spleen can result in massive hemorrhage. The patient should be stabilized and taken to the operating room for subsequent exploration and correction of the injured structures.

A 53-year-old woman comes to the health care provider for an annual examination. She has no complaints. She has hypertension, for which she takes a thiazide diuretic, but no other medical problems. Her past gynecologic history is significant for normal annual Pap tests for many years, her last being 2 months ago. A recent mammogram was negative. Heart, lung, breast, abdomen, and pelvic examinations are unremarkable. Which of the following procedures or tests should most likely be performed on this patient? A. Chest radiograph B. Pap test C. Pelvic ultrasound D. Cardiac stress test E. Colonoscopy

E. Cancer screening should be an essential part of an annual examination. Colorectal cancer is a major cause of serious morbidity and mortality for women in the United States: greater than 50,000 new cases are diagnosed each year, and there are more than 25,000 deaths from colorectal cancer. Screening should begin at age 50 in asymptomatic women who have no significant family history. Screening options for colorectal cancer include home fecal occult blood test (FOBT), flexible sigmoidoscopy, the combination of home FOBT and flexible sigmoidoscopy, colonoscopy, and double-contrast barium enema. This patient has no indication for a chest radiograph (choice A). Lung cancer has surpassed breast cancer as the major cause of cancer death in women. Chest radiographs are not used for lung cancer screening, however. Selected patients with long-standing history of smoking may qualify for low-dose CT of the chest screening for lung cancer according to the most recent guidelines. The Pap test (choice B) is an important method of screening for cervical cancer and should be performed according to the patient's risk and previous history of screening.This patient had a normal Pap test 2 months ago, however. Pelvic ultrasound (choice C) is not indicated in this patient. She has nothing abnormal in her history or physical that would warrant pelvic ultrasound. Pelvic ultrasound is not used for ovarian cancer screening. Cardiac stress test (choice D) is not a useful screening test for cardiovascular disease in asymptomatic patients. The results of this test are even more spurious in the asymptomatic female population. Review Colorectal cancer is the third leading cause of cancer-related death in men and women in the United States. For average-risk patients who are asymptomatic, screening should begin at age 50. Screening can be performed via colonoscopy as a way to detect and remove polyps, which are the precursor lesions.

A newborn examination of an infant reveals a scaphoid abdomen and a palpable fullness of the epigastrium. An abdominal radiograph of the infant shows gaseous distention of the stomach and proximal duodenum. A nasogastric tube is placed and suction produces bilious fluid from the stomach. Which of the following prenatal studies might have revealed this abnormality? A. Alpha-fetoprotein level B. Antibody screens for common fetal infections C. Hemoglobin electrophoresis D. Human choriogonadotropin levels E. Ultrasonography

E. This patient most likely has duodenal atresia, an obstruction resulting from a failure of recanalization of the duodenal lumen, resulting in complete intrinsic obstruction. It can be detected with prenatal ultrasonography and it occurs in 1/10,000 live births. Twenty to 30% of patients with duodenal atresia have trisomy 21. There is no predilection for sex or race. Duodenal atresia presents early, usually in the first hours or day of life. The neonate has bilious vomiting without abdominal distention. In approximately 15% of neonates who have this anomaly, vomiting is nonbilious because the obstruction is located proximal to the ampulla of Vater. There may be a history of polyhydramnios (which is commonly associated with diabetes and twin pregnancy). Prenatal ultrasonography may reveal a dilated and fluid-filled stomach and duodenum in addition to other possible anomalies. On physical examination, the newborn may have a scaphoid abdomen and epigastric fullness from a dilated stomach and proximal duodenum. Nasogastric suction produces bilious fluid. Abdominal films reveal the characteristic "double bubble" of the stomach and proximal duodenum. Therapy involves surgical correction. The patient should be evaluated for associated anomalies. Elevated alpha-fetoprotein levels (choice A) have been reported in fetomaternal hemorrhage, umbilical cord hemangioma, polycystic kidneys, cytomegalovirus, and parvovirus infections, as well as central nervous system anomalies. Antibody screens for common fetal infections (choice B) are used to detect infection with toxoplasmosis, other infections, rubella, cytomegalovirus infection, and herpes simplex virus infection (TORCH). Hemoglobin electrophoresis (choice C) is used to detect hemoglobinopathies, such as sickle cell disease or alpha-thalassemia heterozygosity in at-risk populations. Human choriogonadotropin levels (choice D) may be significantly elevated with sacrococcygeal teratoma, choriocarcinoma, parvovirus infection, Turner syndrome, and Down syndrome. Review Duodenal atresia may present with a history of polyhydramnios. The neonate may have a bilious or nonbilious vomiting, depending on whether the obstruction is distal to or proximal to the ampulla of Vater. Physical examination of the neonate may reveal a scaphoid abdomen with epigastric fullness. An ultrasound can detect this abnormality if done during the prenatal period. Surgical treatment is indicated in this setting.

A 43-year-old man develops excruciating abdominal pain at 8:23 PM (he looked at his watch when the pain "hit him"). When seen in the emergency department about 30 minutes later, he has a rigid abdomen, lies motionless on the examination table, has no bowel sounds, and is obviously in great pain which he describes as constant and encompassing the entire abdomen. There is very severe pain when deep palpation of the abdomen is attempted in any of the 4 quadrants. The examining hand cannot make much of an indentation, however, because of the impressive muscle guarding. When the attempt is aborted, he manifests severe rebound tenderness. Radiographs show free air under both diaphragms. Which of the following does this man most likely have? A. Acute abdomen, the nature of which cannot yet be defined B. Acute inflammatory process affecting an intra-abdominal viscera C. Acute obstruction of an intra-abdominal viscera D. Ischemic process affecting intra-abdominal organs E. Perforation of the gastrointestinal tract

E. There is no doubt that this patient has an acute abdomen, but we can tell more than that. The sudden onset, generalized extent, and silent abdomen in a man who does not want to move suggest a perforation. In addition, the presence of free air in the peritoneal cavity pinpoints the gastrointestinal tract as the source. We cannot tell whether he perforated a peptic ulcer, blew out a sigmoid diverticulum, or had his bowel perforated by a chicken bone, but there is a hole in his gastrointestinal tract. Choice A identifies that the patient has an acute abdomen, but the free air under the diaphragm is consistent with perforation of a hollow organ such as the stomach or bowel. An inflammatory process (choice B) would have gradual onset and would be localized to the area of the inflamed viscera. Obstruction (choice C) has sudden onset, but it leads to colicky pain. The patient would be thrashing about looking for a position of comfort, and would still have bowel sounds and not have free air. Ischemia (choice D) is a more likely scenario in an older person, especially if that person has a history of atherosclerosis. If it were to lead to necrosis and perforation, the latter would not be evident 30 minutes after the trouble began. Review When there is a perforation in the GI tract hollow viscus, the radiographic manifestation will be free air under the diaphragm. Once a patient is shown to have this manifestation, surgical intervention is needed to identify the site of the perforation so that correction can be performed. The two most common GI sites for perforation are the stomach and duodenum from peptic ulcer disease and the rectosigmoid colon from acute diverticulitis.

A 19-year-old college freshman undergoes sigmoidoscopy because of a family history of multiple polyps in his young siblings. His brother underwent total proctocolectomy at age 23, and his sister underwent a total proctocolectomy at age 29, after both were found to have hundreds of colonic adenomas on colonoscopy. Both siblings are alive and well 5 years later and without any other findings of neoplasms. The patient undergoes sigmoidoscopy and is found to have several dozen small colonic polyps within the rectosigmoid. Which of the following is the most appropriate next step in management? A. Schedule a repeat sigmoidoscopy in 1 year B. Schedule a colonoscopy in 1 year C. Evaluate the more proximal colon with a barium enema D. Schedule a full colonoscopy now E. Schedule a total proctocolectomy

E. This patient has 2 first-degree relatives who have undergone total proctocolectomies for a polyposis syndrome. This most likely represents the familial polyposis coli syndrome in that both siblings had high numbers of colonic adenomas and underwent an appropriate total proctocolectomy. If this patient is found to have several dozen polyps during the flexible sigmoidoscopy, it is evidence that he is expressing the phenotypic trait of the familial adenomatous polyposis (FAP) gene. Rather than wait for any of these polyps to undergo malignant degeneration, he should have a total proctocolectomy in the immediate near future. Once many polyps have been demonstrated, the likelihood of one of these becoming malignant is high. Scheduling repeat sigmoidoscopy (choice A) or colonoscopy (choice B) in 1 year is wrong, because this patient, who already has extensive polyposis, has passed the point of needing only periodic monitoring. Evaluating the complete colon with either a barium enema (choice C) or full colonoscopy (choice D) is not needed, because there is already sufficient evidence to warrant total proctocolectomy. Review Familial adenomatous polyposis is an inherited condition caused by a single gene defect. Patients who have this condition develop multiple colonic polyps that have an extremely high rate of development of colon cancer. Surgical removal of the colon is curative and is used as a preventive strategy for patients who carry this gene.

A 54-year-old man, who 5 years ago underwent laparotomy for a gunshot wound to the abdomen, is admitted to the hospital because of protracted vomiting and progressive abdominal distention. The symptoms began 5 days ago, and since then he has not had a bowel movement or passed any gas. At the time of hospitalization, he has hyperactive bowel sounds and some abdominal discomfort but he does not have an acute abdomen. Abdominal radiographs show dilated loops of small bowel, multiple air-fluid levels, and no free air under the diaphragms. He is placed on nasogastric suction and intravenous fluids. After 6 hours he develops fever, leukocytosis, and abdominal tenderness. There is severe tenderness when external pressure is applied to his abdomen and then suddenly released. There are no audible bowel sounds. Which of the following is the most appropriate next step in management? A. Add antibiotics B. Barium tag and serial abdominal radiographs C. CT scan of the abdomen D. Upper gastrointestinal endoscopy and introduction of a long intestinal tube E. Emergency exploratory laparotomy

E. This patient presents with mechanical intestinal obstruction caused by adhesions which developed as a result of previous exploratory laparotomy, and he now has signs of bowel strangulation. If the strangulated loop is still viable, it has to be freed immediately. If it is necrotic, it has to be resected with equal urgency to prevent continued peritoneal soiling. This patient has now developed an acute abdomen, as manifested by rebound tenderness, fever, and leukocytosis. Antibiotics (choice A) will not provide viability to a compromised loop of bowel or prevent peritonitis if the loop is dead. Barium tag (choice B) is done in the postoperative period after abdominal surgery when we cannot decide whether a sluggish bowel has paralytic ileus or early mechanical obstruction. This is not the situation here. CT scan (choice C) is our universal answer when we do not know what is happening inside the belly. Here we do. Had he shown up with obstruction and no reasonable etiology for it (no prior surgery, no hernias), we might have done a CT scan. Endoscopy and a long tube (choice D) will not take care of dead or dying bowel. Review The clinical picture is typical of bowel obstruction; abdominal distention, vomiting, and hyperactive bowel sounds are common manifestations. The most common cause of bowel obstruction is adhesions from a previous surgery. Regardless of the cause, it is always a surgical emergency.

A 36-year-old schoolteacher comes to the clinic for a new-patient evaluation. She teaches second grade at a local school and just recently moved to the area. She has no current health complaints but reports a history of hepatitis. She apparently contracted "chronic hepatitis" from a blood transfusion received shortly after the birth of her first child 20 years ago, although she is unsure of the details of the disease. She was told she was infected, but she has always felt well and she doubts the diagnosis. Since then she has not suffered any symptomatic liver disease, and her liver function tests have always been normal. Review of symptoms fails to reveal any evidence of liver disease, and physical examination is unremarkable. A review of medical records that she has brought with her reveals the following: Hep A Ab - neg HBsAg - neg HBsAb - POS Hep C ab - POS In addition to disease education and counseling, which of the following is an appropriate intervention for this patient at this time? A. Hepatitis A vaccination B. Hepatitis B vaccination C. Liver biopsy for risk stratification D. Referral for interferon/ribavirin treatment E. Right upper abdominal ultrasound

The correct answer is A. This patient is extremely likely to have chronic hepatitis C but is currently asymptomatic. Though asymptomatic patients without any elevation of liver enzymes do not require treatment, they do need protection against other forms of hepatitis; that is because, if infected, they are at a much higher risk for fulminant liver disease. The current recommendations for hepatitis A vaccination are as follows: Hepatitis B vaccination (choice B) is not necessary, because this patient is antibody-positive, indicating immunity. A liver biopsy (choice C) is an invasive procedure and is not necessary, given the normal LFTs and the lack of evidence of liver disease. If this patient were symptomatic and had elevated LFTs, a liver biopsy could be considered, although much of the same information could be obtained from an ultrasound (choice E). Asymptomatic patients without any evidence of transaminitis are not appropriate candidates for interferon/ribavirin treatment (choice D). The treatment protocol is long and expensive and has many side effects. Additionally, many patients do not respond to this therapy despite the appropriate treatment being given. Therapy is less successful for chronic disease, and therapy is based on viral load numbers. Once evidence of liver dysfunction is present, referral for treatment is appropriate. Review Hepatitis C infection usually occurs from parenteral drug use but could also be acquired as a result of a blood transfusion given many years ago, before the mandatory testing of blood products against hepatitis C. Almost 85% of patients who are infected with hepatitis C will develop a chronic carrier state without spontaneously clearing the virus. The majority of hepatitis C patients who live in the United States have genotype 1, and the genotypes that are most responsive to therapy are genotypes 2 and 3. Patients who have hepatitis B or C should have vaccination against all of the hepatitis viruses (if an immunization is possible) to prevent fulminate liver disease in case the patient gets infected with one of the hepatitis viruses.

A 34-year-old bus driver comes to the health care provider with complaints of difficulty swallowing. The symptoms began gradually, approximately 9 months ago, and have prevented him from swallowing solids or drinking liquids comfortably. He awakens at night with cough and occasional morning regurgitation of recognizable food from the night before. He has learned to reduce his oral intake and has lost 6 pounds over the past 2 months. He does not smoke or drink, and has no family history of esophageal or other gastrointestinal illnesses. Physical examination is unremarkable. Which of the following is the most likely diagnosis? A. Achalasia B. Esophageal adenocarcinoma C. Lower esophageal web D. Peptic stricture E. Progressive systemic sclerosis

A. The diagnosis of a motility disturbance rather than a mechanical cause for dysphagia is suggested by the history of difficulty with swallowing both solids and liquids at the onset of his symptoms. Achalasia is characterized by markedly reduced or absent peristaltic waves in the body of the esophagus and an increased resting pressure at the lower esophageal sphincter (LES) that fails to relax with swallowing. This causes stasis of food within the esophagus, which can be regurgitated, even aspirated, and can lead to aspiration pneumonia. The underlying disease process in achalasia appears to be a malfunction of the myenteric plexus of the esophagus, which in turn leads to denervation of the esophageal musculature. Forceful dilation of the LES is often initially helpful in relieving the obstruction and pressure within the esophagus, but may have to be repeated periodically as obstruction often recurs. The addition of sublingual nitroglycerine or calcium-channel blockers that tend to relax the LES may increase the time between dilations. Some patients are eventually treated with either a Heller myotomy or injection of botulinum toxin into the LES. Rare cases of achalasia are complicated by esophageal rupture with secondary mediastinitis. Esophageal carcinoma (choice B) may occur in patients who have a history of Barrett's esophagus secondary to long-standing GERD. It, too, produces a mechanical-type dysphagia. In any case, this patient is far younger than the typical patient who develops esophageal adenocarcinoma seen later in life. A lower esophageal web (choice C), also referred to as a Schatzki ring, produces episodic dysphagia to large solids that are greater in diameter than the size of the ring. A peptic stricture (choice D) presents with a mechanical-type dysphagia. The dysphagia initially affects large solids, and then gradually worsens to smaller, softer foods, and eventually liquids. It usually occurs in patients who have a long-standing history of GERD. Progressive systemic sclerosis (PSS) (choice E) is far more likely to occur in women. It, too, causes a motility disturbance of the esophagus and, like achalasia, has reduced or absent motility in the body of the esophagus. In PSS, however, there is reduced LES pressure at rest that predisposes to severe GERD and its possible sequelae. Unless there are symptoms of a mechanical obstruction secondary to a stricture, esophageal retention of food does not occur. Review Achalasia is a failure of the LES to relax and poor peristaltic stripping waves, producing a motor disorder signified by initial complaints of dysphagia for solids and liquids. Treatment is esophageal dilation with calcium-channel blockers or nitrates used prior to clinical intervention.

A 43-year-old man reports a 9-pound weight loss over the past 9 months, accompanied by difficulty swallowing solids and liquids. He has woken on several occasions at 4 AM and regurgitated partially digested dinner contents. An upper gastrointestinal series reveals a widely dilated esophagus with a smoothly tapering distal esophagus. There appears to be partially digested food present in the esophagus. Which of the following is the most likely diagnosis? A. Achalasia B. Diffuse esophageal spasm C. Esophageal squamous cancer D. Peptic stricture E. Scleroderma

A. This patient has the symptoms of a motility-type dysphagia in that he has difficulty with both solids and liquids from the onset of his symptoms. The nocturnal aspiration of food occurs because the esophagus remains filled for hours or even days after completing a meal. The radiograph reveals the typical dilated esophagus of achalasia, which is termed a bird's-beak esophagus, with distal esophageal tapering. Diffuse esophageal spasm (choice B) will typically produce "non-cardiac chest pain" in association with a motility-type dysphagia. This patient has increased force of peristalsis, which is the opposite problem that occurs with achalasia. Esophageal squamous cancer (choice C) would produce a mechanical type of dysphagia and is unlikely in a 42-year-old man without any specific risk factors (i.e., smoking, drinking, lye ingestion, or Plummer-Vinson syndrome). Peptic stricture (choice D) is incorrect because it would produce a mechanical, not a motility or motor, type dysphagia. Scleroderma (choice E) will also produce a motility-type dysphagia, but it is very uncommon for men to develop this disease. Furthermore, aspiration in scleroderma occurs only after a peptic stricture has developed. Before this, the lower esophageal sphincter is typically wide open and foods do not accumulate in the esophagus. Review Achalasia is a motor disorder affecting the esophagus. There are two simultaneous problems that occur with this disorder: failure of the lower esophageal sphincter to relax and poor peristalsis in the distal third of the esophagus. The end result of this condition is that food is retained in the lower esophagus with dilation of the distal esophagus and poor transfer of food into the stomach. Patients present with dysphagia for solids and liquids along with regurgitation and retrosternal chest pain. Definitive therapy is surgery with pneumatic dilation versus laparoscopic cardiomyotomy. Conservative therapy may also include nitrates or calcium-channel blockers that help to relax the lower esophageal sphincter, which helps to empty the food from the esophagus.

A 57-year-old alcoholic man is being treated for acute hemorrhagic pancreatitis. He was in intensive care for 1 week, where he required chest tubes for pleural effusions and a ventilator for several days. Eventually he improved sufficiently to be transferred to the floor but 3 days later, and about 2 weeks after the onset of the disease, he spiked a fever and developed leukocytosis. Which of the following developments do these recent findings most likely suggest? A. Chronic pancreatitis B. Pancreatic abscess C. Pancreatic pseudocyst D. Pelvic abscess E. Subphrenic abscess

B. A very common complication of hemorrhagic pancreatitis, and often the reason for the demise of the patient, is the development of a pancreatic abscess. The timetable is usually about 10-14 days from the onset of the disease, and the initial manifestations are fever and leukocytosis. Chronic pancreatitis (choice A) develops after several years of recurrent attacks of pancreatitis and is characterized by steatorrhea, diabetes, and constant pain. Pancreatic pseudocyst (choice C) is another potential complication of pancreatitis, but the manifestations are related to pressure symptoms from the fluid collection, there is no fever or leukocytosis, and the timetable for development is about 6 weeks from the onset of the disease. Pelvic abscess (choice D) and subphrenic abscess (choice E) are indeed in the differential diagnosis, as they also show up with fever and leukocytosis some 10-14 days from the original problem. The original problem for these patients, however, is usually an infectious process in the abdomen (e.g., a ruptured appendix or a perforated viscus). If the problem began with pancreatitis and then there are signs of sepsis, the pancreas is the logical place to harbor the pus. Review Pancreatitis has several complications that can occur following the acute inflammation. When the patient has acute pancreatitis, both endocrine and exocrine function of the gland can be affected. Pancreatic abscess should be suspected following acute pancreatitis when the patient develops fever, abdominal tenderness, and leukocytosis. Pancreatic abscess occurs when the peripancreatic fluid collections become colonized and infected. Although not common, it is fatal if not treated surgically. CT scan is diagnostic, showing a ring-enhancing fluid collection with gas. Prophylactic antibiotics may be used as a preventive measure in patients who have acute pancreatitis, but it is unknown whether this will prevent a pancreatic abscess.

Ten days after having an exploratory laparotomy for blunt abdominal trauma, a 53-year-old man develops daily fever spikes to 38.9ºC (102.0ºF) and 39.5ºC (103.1ºF), as well as leukocytosis. At the time of surgery he was found to have a seat-belt injury to the small bowel and required resection and re-anastomosis of the affected area. He then had an uneventful postoperative course until the fever became apparent on day 10 of postop. He has been tolerating oral intake since day 5 of postop, and at this time his abdominal examination is unremarkable except for the fresh midline wound. Rectal examination is also unremarkable. The abnormality responsible for the fever will most likely be demonstrated by which of the following studies? A. Chest radiograph B. CT scan of the abdomen C. Plain films of the abdomen D. Repeated blood cultures E. Upper and lower gastrointestinal radiographs with water-soluble contrast

B. Fever that starts about 10 days after a contaminated abdominal surgical procedure is most likely caused by a deep abscess, either pelvic or subphrenic. The former has been ruled less likely by the patient having a normal rectal examination; the latter can best be demonstrated by CT scan. A chest radiograph (choice A) could be helpful if an air-fluid level were to show up beneath the right diaphragm, but such a finding is rare. An elevated hemidiaphragm with pleural effusion would also be suggestive but not diagnostic. The lung itself should not be the source of this late postoperative fever but is a common source of fever sooner in the postoperative period. Plain films of the abdomen (choice C) might show the same findings detailed above, but the ability to image this abscess is much less likely than with the more sensitive CT scan of the abdomen. Pus alone will not be seen on a plain film, but a localized abscess may be visualized. If an abdominal abscess is suspected, a CT scan of the abdomen is a much more sensitive test to perform. A plain film may demonstrate perforation but by its ability to identify free air in the peritoneal cavity, but this patient does not have the manifestation of perforation in this setting. Blood cultures (choice D) might identify a type of flora, which would suggest the source of the infection (in this case the gastrointestinal tract flora), but would not pinpoint the location of the abscess. Additionally, blood culture results may not be as sensitive, because this patient was most likely previously given antibiotics as part of the treatment regimen with his previous trauma and surgery. Radiograph studies of the bowel (choice E) might identify a leak in an anastomosis and might be plausible if a gastric or a colonic suture line breakdown were suspected, but this would not be the case in this scenario, as the patient was tolerating his diet for the previous 5 days. Review Several causes of postoperative fever are well known. These causes are somewhat time-dependent, with early causes being atelectasis, wound infections, urinary tract infections, pulmonary emboli, and pneumonia. If a fever is suspected 10 days postoperative following an abdominal surgery, peritoneal abscess should be the primary consideration.

A 56-year-old alcoholic comes to the health care provider complaining of 6 months of worsening midepigastric pain radiating to his back. The pain is exacerbated by eating and continued alcohol use. One month ago he was told that he had developed diabetes. He has also noted that, over the past 2 months, he has had greasy, foul-smelling, large-volume stools. Which of the following is the most likely cause of this man's steatorrhea? A. Acid inactivation of pancreatic enzymes B. Bacterial overgrowth C. Biliary insufficiency D. Excess secretion of glucagon E. Insufficient lipase available for normal fat digestion

E. Midepigastric pain radiating to the back and exacerbated by eating and continued alcohol use should suggest the diagnosis of pancreatitis. Significant risk factors for pancreatitis include alcoholism (as this patient has) and biliary tract disease (for which there is no evidence in this patient). It is a little unclear from the history whether the patient is experiencing recurrent episodes of acute pancreatitis or is developing the onset of chronic pancreatitis. Whichever is the case, both processes are capable of destroying enough exocrine pancreatic tissue to predispose for exocrine pancreatic insufficiency with resultant insufficient production of pancreatic enzymes, such as lipase, which normally facilitates fat usage, to allow for normal digestion. Steatorrhea, or excess fat in stool, is seen clinically as greasy, foul-smelling (because of bacterial action on the fat), large-volume (in part because of gas production by bacteria and in part because of failure to digest most of the food) stools. There is no evidence that there is hyperacidity to account for inactivation of pancreatic enzymes (choice A). This phenomenon can occur, but it would be more likely to be seen in settings such as Zollinger-Ellison syndrome, in which a gastrin-producing tumor of the pancreas or small intestine stimulates excess acid production by the stomach. Bacterial overgrowth (choice B) can produce steatorrhea by deconjugating bile salts and making them unavailable to form micelles. This patient has no underlying etiology for bacterial overgrowth, however, such as blind loop syndrome or hypomotility. Biliary insufficiency (choice C) can cause steatorrhea; however, although biliary tract disease can predispose for pancreatitis, pancreatitis does not predispose for biliary insufficiency. The pain of biliary tract disease is usually localized to the right upper quadrant and may radiate to the right shoulder. Excess glucagon (choice D) is rarely noted in the absence of the very rare neuroendocrine tumor known as glucagonoma. It does not produce fat maldigestion or malabsorption. Review Recurrent acute or chronic pancreatitis may result in inefficient production of normal pancreatic enzymes. A history of alcoholism is an important clue to the diagnosis. Lipase deficiency results in large-volume, greasy, foul-smelling stools secondary to fat malabsorption.

A 1-month-old boy is brought to the emergency department by his mother, who states that he has been having forceful vomiting for the past several days. She states that he vomits every time she feeds him, and the situation seems to be getting worse, though he seems in no pain. She describes the vomitus as non-bilious and his stools as normal with no blood. On examination the infant appears to be mildly dehydrated, with soft abdomen and palpable 2-cm, firm, mobile mass in the right upper quadrant. Which of the following is the most likely diagnosis? A. Duodenal atresia B. Intussusception C. Hirschsprung's disease D. Midgut volvulus E. Pyloric stenosis

E. Pyloric stenosis typically develops between 3 and 6 weeks of life. It is caused by hypertrophy of the pyloric muscle, which obstructs gastric outflow. The incidence is higher in males and first-born infants. The symptoms include progressively worsening vomiting, which becomes projectile and is non-bilious. On examination peristaltic waves may be seen, and an olive-sized mass is usually palpated in the right upper quadrant. The vomiting continues despite multiple formula changes. Duodenal atresia (choice A) is usually associated with other congenital anomalies and presents with bilious vomiting. Upright abdominal radiographs show the classic "double bubble sign." Intussusception (choice B) presents with vomiting, bloody stool, and colicky abdominal pain. As the obstruction progresses, the vomitus becomes bile-stained. On examination there is usually a sausage-shaped mass in the distribution of the colon. Hirschsprung's disease (choice C), or aganglionic megacolon, is associated with failure to pass meconium or constipation and abdominal distention. Diagnosis is made by rectal biopsy. Midgut volvulus (choice D) can occur at any age but is common in infancy. Symptoms include bilious vomiting, abdominal distention, pain, and bloody stools. An upper gastrointestinal series is diagnostic for volvulus showing a "corkscrew" narrowing of the distal duodenum. Review Pyloric stenosis causes recurrent projectile, non-bilious vomiting between 3 and 6 weeks of life. Patients often have a history of multiple formula changes because of presumed intolerances to these feedings. Failure to thrive and weight loss may progress to volume depletion. An olive-shaped mass may be palpable in the right upper abdomen. Ultrasound is typically used as a diagnostic evaluation tool. Treatment is with IV fluid and electrolyte replacement, followed by pyloromyotomy done via an open or laparoscopic approach.

A 45-year-old man with alcoholic cirrhosis is bleeding from a duodenal ulcer. He has required 6 units of blood over the past 8 hours, and all conservative measures to stop the bleeding, including irrigation with cold saline, intravenous vasopressin, and endoscopic use of the laser have failed. He is being considered for surgical intervention. Laboratory studies done at the time of admission, when he had received only 1 unit of blood, showed a bilirubin 4.5 mg/dL (normal <1.9 mg/dL), prothrombin time 22 seconds (normal 11-15 seconds), and serum albumin 1.8 g/dL (normal 3.5-5.5 g/dL). He was mentally clear when he came in but has since developed encephalopathy and is comatose. Which of the following best describes his operative risk? A. Acceptable B. Amenable to improvement if he receives vitamin K C. Amenable to improvement if he is given albumin D. Prohibitive unless he is dialyzed to normalize his bilirubin E. Prohibitive regardless of attempts to improve his condition

E. The studies show extremely marginal liver function, which would be tipped into overt liver failure by an anesthetic and an operation. He is not a surgical candidate. The patient has signs of end-stage liver disease with hepatic encephalopathy along with poor manufacturing of clotting factors. Because the patient would need to have anesthetic agents metabolized through the liver, he is incapable of surviving the surgery at this point in time. Choice A obviously misses the gravity of his situation. Vitamin K (choice B) works only when there is a functioning liver that can use it. In the absence of adequate liver function, it will not correct the prothrombin time. Albumin (choice C) can be given, but it will have a short life span and will not correct the liver dysfunction. The low albumin is not the main problem per se, it is a symptom of how bad his liver is. The same is true of bilirubin (choice D). It is a symptom, not the problem. We can operate on patients who have much higher bilirubin if it is not caused by intrinsic liver disease. Review Patients who have cirrhosis may go on to develop end-stage liver disease. Liver failure is manifested by elevation in total bilirubin, increased AST and ALT, decreased serum albumin (poor manufacturing), and hyponatremia as a result of ascites. As the liver fails, the last substance that a liver stops making is clotting factors, which means that prolongation of the clotting time will be a late and serious manifestation of liver failure. Patients will also develop thrombocytopenia caused by portal hypertension with hypersplenism and platelet sequestration.

A 34-year-old tax lawyer comes to the clinic complaining of difficulty swallowing. On several occasions over the past few months, he has been aware of meat becoming stuck in the mid-chest immediately after eating. After each episode, he has several hours of chest pain that resolves gradually. On 2 occasions he induced vomiting to obtain relief. Over the past 10 days the difficulty swallowing has become worse, and he now has trouble with even soft foods. He has been taking ranitidine (Zantac), magnesium hydroxide, and omeprazole (Prilosec) for 4 years but has remained symptomatic despite these measures. He has smoked 1 pack of cigarettes daily for 15 years and denies any alcohol use. Physical examination is normal. Which of the following is the most likely cause for these symptoms? A. Diffuse esophageal spasm B. Esophageal squamous carcinoma C. Lower esophageal web D. Peptic esophageal stricture E. Scleroderma

D. This patient presents with symptoms consistent with gradual luminal narrowing of the esophagus after many years of gastroesophageal reflux disease (GERD). These symptoms suggest the development of a benign peptic stricture. This may occur even if the patient is on medical therapy to reduce acid secretion, because many patients will continue to produce acid despite standard medical regimens and may require very high doses of proton pump inhibitors. Treatment consists of endoscopic dilation of this stricture and continued aggressive anti-reflux therapy. Diffuse esophageal spasm (choice A) presents with "noncardiac chest pain," usually described as a squeezing chest pain. It typically presents with a motility-type dysphagia. Although GERD is a common underlying factor, the type of dysphagia in esophageal spasm would be consistent with a motility-type dysphagia (i.e., patient has difficulty with liquids as well as solids from the onset of symptoms). Squamous carcinoma (choice B) is not a sequelae of long-standing GERD. Furthermore, his age makes it extremely unlikely that he would develop esophageal carcinoma (even with his history of smoking as a risk factor). Squamous cell carcinoma is closely related to long-term use of alcohol and cigarettes. A lower esophageal web (choice C) or Schatzki ring produces dysphagia in an episodic pattern as foods that are greater in size than the diameter of the web become lodged in the distal esophagus. Intermittent dysphagia is typically seen with this condition. Scleroderma (choice E) could cause dysphagia, but this would commonly be accompanied by Raynaud phenomenon and characteristic changes in the skin. Additionally, scleroderma is about 3 times as common in women as in men. Review Long-standing, chronic GERD predisposes to development of peptic esophageal strictures, even during medical therapy with proton pump inhibitors. Carcinoma is less likely in young patients with no history of constitutional symptoms or weight loss.

Two bewildered parents rush into the emergency department carrying a 3-year-old girl. They tell the health care provider at the door that she ingested multiple acetaminophen pills less than an hour ago. The mother was experiencing a terrible headache over the past 2 days and was taking acetaminophen as a pain reliever. She left the open bottle on her desk to get a glass of water, and when she returned her daughter was putting the tablets in her mouth. Immediately the mother removed whatever pills were still in the child's mouth and checked to insure that all the other medications were safely stored away; when she saw that they were untouched in the medicine cabinet, the family got in the car to rush to the hospital. On route there the girl started vomiting. The mother does not know how many pills her daughter took because it wasn't a full bottle to begin with. At the hospital she takes out the half-empty bottle and spills the contents on the counter. A total of 36 pills is present from an original 100 count, 325 mg of acetaminophen each. The child, crying on the examining table, appears frightened and diaphoretic, and repeats several times that she wants to throw up. Her pulse is 120/min and respirations are 24/min. Which of the following is the most appropriate next step? A. Administration of dextrose and naloxone (Narcan) B. Administration of N-acetylcysteine (Mucomyst, Acetadote) intravenously C. Administration of N-acetylcysteine orally D. Immediate measurement of plasma concentration of acetaminophen E. Supportive care and measurement of plasma concentration of acetaminophen 3 hours later

E. Acetaminophen is an analgesic and antipyretic that is metabolized in the liver. Acetaminophen poisoning causes an N-acetyl-p-benzoquinoneimine metabolite that produces hepatotoxicity in the absence or depletion of glutathione. In children younger than age 12 the toxic dose is 150 mg/kg. A single ingestion of 7.5 g is considered to be a minimum toxic dose in adolescents and adults. There are 4 stages of acetaminophen poisoning if the patient is left untreated. Absorption may be delayed 4 hours in an overdose. A plasma level of acetaminophen therefore should be measured when it has been at least 4 hours since the ingestion. Levels drawn before that may not be accurate and may give a false low result. The Rumack-Matthew nomogram is used to plot the level of acetaminophen based on the time elapsed since ingestion and the acetaminophen plasma concentration. N-acetylcysteine is the antidote for acetaminophen poisoning, and it should be administered if the acetaminophen level is equal to or greater than 150 µg/ml. N-acetylcysteine works best if started within 8 hours of ingestion, but it may be given up to 16 hours after ingestion. In cases of severe poisoning, Poison Control may advise N-acetylcysteine be given as late as 24 to 36 hours after ingestion has occurred. In the United States, N-acetylcysteine is given orally. Liver enzymes, bilirubin levels, and prothrombin time should be monitored in patients whose plasma levels of acetaminophen are within the toxic range in the nomogram. In patients who have fulminant hepatic necrosis, death may occur. Severe hepatic damage may necessitate a liver transplant in some patients. Dextrose and naloxone administration (choice A) is indicated in patients who present with an altered mental status or respiratory depression following narcotic and opioid ingestions. Because there are no such signs in this patient and because it is suspected that the child ingested only one drug, acetaminophen, dextrose and nalaxone (Narcan) is not indicated at this time. It is not indicated to administer N-acetylcysteine intravenously (choice B), because the acetaminophen level has not yet been determined and so there are no data as yet indicating that it is necessary. N-acetylcysteine is typically given orally, but there is an alternative intravenous delivery system that can also be given via protocol. Administration of N-acetylcysteine orally (choice C) may be indicated once a toxic plasma level of acetaminophen is confirmed. That is not the case here, however. Also, the symptoms that the child is exhibiting do not necessarily have to be indicative of acetaminophen poisoning, but may be the result of a sympathicomimetic response to a frightening situation. The administration of N-acetylcysteine therefore should be delayed until data are available to support its necessity. Absorption of acetaminophen may be delayed 4 hours in an overdose. A plasma level of acetaminophen therefore should be measured when it has been at least 4 hours since the ingestion. Levels drawn before that time may not be accurate and may give a false low result. Immediately measuring plasma concentration of acetaminophen (choice D) is therefore not indicated.

A 4-year-old girl is found drinking liquid drain cleaner by her parents and is immediately brought to the emergency department. She appears very irritable and is unwilling to swallow any liquid medication. Examination of the oral cavity reveals no evidence of burns or ulceration. Which of the following is the most appropriate management of this patient? A. Barium swallow B. CT scan of the abdomen C. NPO for 12 hours, then clear liquid diet for 3 days D. Indirect laryngoscopy E. Esophagoscopy

E. Ingestion of corrosive material is a rather common problem in pediatrics. The most common corrosive substances ingested by children include household cleaners, detergents, bleaches, disk batteries, and coins. Liquid drain cleaner is a highly alkaline substance, and ingestion can cause severe esophageal necrosis of the liquefaction type. Full-thickness injury is common. In severe cases, it can cause esophageal perforation and mediastinitis. Long-term complications can include stricture and squamous cell carcinoma. Acid ingestion causes coagulation necrosis and eschar formation. The eschar tends to protect the esophagus from full-thickness injury and corrosive perforation. When a child has a definitive history of ingestion of corrosive substance, he or she needs to be evaluated emergently. Especially in this case, dysphagia further suggests the presence of esophageal injury. Even though there are no ulcerations or burns in the mouth, esophageal ulceration is very likely, because oral lesions correlate poorly with esophageal injuries. The child should be stabilized, and flexible esophagoscopy is indicated to directly visualize the esophagus. It might be prudent to perform a chest radiograph first to rule out overt perforation and mediastinitis. If esophagoscopy shows no esophageal injury, no treatment is necessary. For mild to moderate burns, therapy should include IV hydration, analgesics, and antibiotics. Complications include esophageal stricture. Prevention is better than treatment. When there is a young child at home, corrosive substances should always be put in a safe place, out of the child's reach. Pediatricians should also address this issue with the parents and give appropriate anticipatory guidance on routine office visits. Barium swallow (choice A) is not appropriate in the setting of acute corrosive injury of the esophagus. CT scan of the abdomen (choice B) is not indicated unless there are peritoneal signs that suggest stomach perforation and peritonitis. NPO for 12 hours, then liquid diet for 3 days (choice C) without further evaluation by esophagoscopy is inappropriate. Indirect laryngoscopy (choice D) is not indicated unless there is a burn of the larynx that might result in laryngeal edema. Review Oral ingestion of caustic substances will cause major damage to the esophagus. Priority evaluation of the esophagus is urgently needed because damage may lead to esophageal perforation and mediastinitis. There is a poor correlation between mouth injuries and esophageal injuries, so lack of oral involvement should not be taken as reassurance of no significant esophageal injury. Treatment is based on the extent of injury, with an attempt to save as much esophageal mucosa as possible. Esophageal strictures commonly result from these types of injuries because of significant scarring that occurs from these types of conditions.

A 35-year-old clergyman with 19-year history of type 1 diabetes is referred to a gastroenterologist by his endocrinologist because of symptoms of early satiety, abdominal bloating, nausea, and recurrent episodes of vomiting after a large meal. A physical examination reveals mild retinopathy and 2 healed ulcers on the ventral surface of his left foot. Abdominal examination is normal. Which of the following is the most likely cause of his symptoms? A. Autonomic neuropathy B. Inappropriate inhibition of gastrin C. Increased resting pressure at the lower esophageal sphincter D. Rapid gastric emptying in the antrum of the stomach E. Scarring at the lower esophageal sphincter

A. This patient has gastroparesis as a result of autonomic neuropathy caused by long-standing diabetes. Autonomic neuropathy is usually seen in diabetic patients who have polyneuropathy, but it can also be associated with postural hypotension, disordered sweating, impotence in men, bladder dysfunction, esophageal dysfunction, and constipation or diarrhea. As with other forms of diabetic peripheral neuropathy, it has been speculated that damage to the tiny blood vessels that accompany and feed nerves may contribute to the nerve dysfunction. If autonomic dysfunction is suspected in a diabetic patient, a useful maneuver on physical examination is to look for a decrease in heart rate in response to the Valsalva maneuver (forced expiratory effort against a closed airway). Autonomic neuropathy is also detected with the lack of pulse variation in naturally occurring cardiac sinus arrhythmia. Diabetic gastroparesis is difficult to manage; patients who cannot tolerate meals of ordinary size and type may tolerate many tiny meals with the use of fluid nutritional supplements to ensure adequate nutrition. These patients may also have retained food in their stomach hours after a meal because of the gastroparesis, which can lead to complications in the setting of emergency surgery with intubation. Inappropriate inhibition of gastrin (choice B) is unrelated to the gastroparesis in a patient who has diabetes. Increased resting pressure at the lower esophageal sphincter (LES) (choice C) is seen in patients who have achalasia; it produces a motility-type dysphagia. It is not associated with diabetes. Gastric emptying (choice D) in diabetic patients tends to be slowed rather than rapid, secondary to the autonomic disease. After the food passes through the stomach, however, a dumping syndrome can result, which is manifested by diarrhea. Scarring at the LES (choice E) produces a mechanical-type dysphagia. There is no causal association between diabetes and scarring at the lower esophageal sphincter. Review Patients who have long-standing diabetes develop both microvascular and macrovascular complications. Microvascular complications include retinopathy, neuropathy, and nephropathy. Macrovascular complications include coronary artery disease, cerebrovascular disease, and peripheral vascular disease. Diabetic neuropathy includes both autonomic neuropathy and peripheral neuropathy. Autonomic neuropathy may affect the GI system with gastroparesis, diabetic diarrhea, and cardiac neuropathy that includes impaired pulse response to stress and impaired response to pain, which can lead to the well-known condition of silent myocardial infarction that is known in patients who have long-standing diabetes.

A 63-year-old man has had mid-abdominal colicky pain with nausea, anorexia, and vomiting for the past 48 hours. The patient has a history of deep venous thrombosis and had a pulmonary embolus in the distant past, but he is not currently on anticoagulants. His temperature is 37ºC (98.6ºF), blood pressure 95/60 mm Hg, and pulse 102/min. On physical examination he has a distended and tympanitic abdomen, but he does not have an "acute abdomen." Abdominal radiographs show multiple distended loops of small bowel and distention of the right colon up to the middle of the transverse colon. Stool is positive for occult blood. Because of his history, a diagnosis of possible mesenteric venous thrombosis is entertained. Which of the following is the most appropriate next diagnostic step? A. CT scan of the abdomen B. Doppler ultrasound C. Exploratory laparotomy D. Laparoscopy E. Upper gastrointestinal endoscopy

A. A CT scan of the abdomen (or MRI scan, which was not offered as a choice) has excellent diagnostic yield when looking for mesenteric venous thrombosis. CT scan with contrast or CT angiogram will show bowel wall thickening, bowel dilation, pneumatosis intestinalis, portal venous gas pattern, and occlusion of the mesenteric vasculature. The CT scan will also provide evidence showing the extent of bowel compromise from ischemia. Mesenteric angiography also remains a viable option in the evaluation of a patient suspected of having mesenteric thrombosis. The clinical picture is indeed very suggestive for that condition. Color Doppler ultrasound (choice B) also has excellent diagnostic capability if done by an experienced sonographer under ideal circumstances. But gas is the "enemy" of sonograms. Given the gas distention in this man, the circumstances are no longer ideal for the ultrasound. Exploratory laparotomy (choice C) is in order if there are signs of peritonitis (which this man does not have). Surgical treatment is not necessarily imperative at this time. He may be able to be managed with anticoagulants or thrombolytic agents. Laparoscopy (choice D) is less invasive than open laparotomy, but the necessary insufflation of the abdomen in an already distended individual who has marginal vital signs is not a good idea, particularly when the noninvasive CT scan can provide the diagnosis. Upper gastrointestinal endoscopy (choice E) will not see the affected area of the bowel (between the ligament of Treitz and the mid-transverse colon, the territory drained by the superior mesenteric vein). Lower gastrointestinal endoscopy would have been better, inasmuch as the right colon could have been seen (and incidentally rule out a cancer as the potential cause of the cutoff of the colonic distention). Although there is usefulness associated with this test, it is not primarily used in the setting of suspected mesenteric thrombosis. Review Patients with mesenteric thrombosis have decreased blood flow into the gastrointestinal tract. This condition can occur acutely or chronically, and can result in ischemic bowel disease. Risk factors include aging, atherosclerosis, history of previous thrombosis or embolic phenomena, smoking history, known hypercoagulable state, atrial fibrillation, myocardial infarction, recent surgery, shock, and chronic heart failure. Patients suspected of having mesenteric thrombosis should undergo a mesenteric angiogram, CT scan with contrast, or CT scan with angiography. Treatment can be surgical to eliminate the obstruction, papaverine infusion and embolectomy, or arterial bypass with or without bowel resection (depending on the level of involvement with the bowel).

A 33-year-old man complains that his chest hurts when he eats, especially when he eats meat. A radiograph shows a dilated esophagus, and achalasia is suspected. Esophageal manometry is used to confirm the diagnosis. Swallowing induced relaxation is reduced at which anatomic location in this man? A. Lower esophageal sphincter (LES) B. Proximal esophagus C. Middle esophagus D. Pharynx E. Upper esophageal sphincter (UES)

A. Achalasia is a disorder of esophageal motility that affects the LES and lower 2/3 of the esophageal body. The LES remains tonically contracted and does not relax as food moves down the esophagus. Food therefore cannot move easily from the esophagus into the stomach. The distal esophagus often becomes greatly dilated. Patients who have achalasia most commonly complain of dysphagia (difficulty swallowing), chest pain, and regurgitation. Since this is a motor disorder, dysphagia occurs for both solids and liquids. During swallowing, the bolus of food is propelled through the pharynx (choice D) by peristaltic contractions. These contractions, along with relaxation of the UES (choice E), propel the bolus of food into the esophagus. Relaxation of the upper esophageal sphincter occurs normally in patients who have achalasia. The upper, middle, and lower esophagus (choices B and C) propel the bolus toward the stomach by coordinated contractions of the muscle layers. Review Achalasia is a condition in which there is failure of the lower 2/3 of the esophagus to relax along with poor peristaltic stripping waves along the body of the esophagus. When food is swallowed, the LES does not contract, trapping food in the lower esophagus and which then dilates proximal to the area of increased contraction. This results in dysphagia for both solids and liquids (consistent with a motor disorder), regurgitation, and retrosternal chest pain.

A 63-year-old woman comes to the emergency department with severe abdominal pain. She reports the pain is epigastric and worse with eating. There is some radiation to the back. There is associated diarrhea that is described as "fatty." Symptoms have been present for 3 to 4 months. Her medical history is significant for 2 episodes of acute pancreatitis, hypertension, and hyperlipidemia, but she takes no medications for these disorders. Her temperature is 38.3ºC (100.9ºF), blood pressure 160/90 mm Hg, pulse 143/min, and respirations 16/min. Physical examination shows epigastric tenderness, mild distention, and decreased bowel sounds. She appears uncomfortable and in moderate distress. An obstruction series reveals only scattered pancreatic calcifications. Laboratory studies show: Amylase 45 U/L (normal 23-140 U/L) Lipase 10 U/L (normal <160 U/L) Albumin 2.8 g/dL (normal 3.4-5.4 g/dL) AST 98 IU/L (normal <40 IU/L) ALT 45 IU/L (normal <40 IU/L) Total bilirubin 1.1 mg/dL (normal <1.9 mg/dL) Which of the following is the most likely underlying etiology for this patient's symptoms? A. Alcohol B. Elevated triglycerides C. Gallstones D. Hypercalcemia E. Idiopathic

A. This patient has chronic pancreatitis. The finding of calcifications in the pancreas is a good clue to this patient's diagnosis. Amylase and lipase can be normal or elevated, depending on how much intact pancreas is left. Seventy percent of chronic pancreatitis is caused by alcohol ingestion. Our patient's malnutrition, as evidenced by her low albumin and her AST to ALT ratio of 2:1, is consistent with alcoholic hepatitis. The treatment is pain relief, a low-fat diet with medium chain triglycerides, vitamins, and pancreatic enzyme replacement. The remaining 30% of chronic pancreatitis is idiopathic (choice E). Elevated triglycerides (choice B) are a rare cause of acute pancreatitis, not chronic pancreatitis. Triglyceride levels in these patients are usually >1,000 mg/dL. Gallstones (choice C) are an important cause of acute pancreatitis but are not typically associated with chronic disease. Patients who have gallstone pancreatitis are usually overweight, fertile, and in their forties. Correction is by ERCP removal or laparoscopic cholecystectomy with common duct exploration and removal of the choledocholithiasis. Hypercalcemia (choice D) is a rare cause of acute pancreatitis. Pancreatic calcifications seen on obstruction series do not represent elevated serum calcium levels. Hypocalcemia can complicate acute pancreatitis as a result of saponification of fats, which takes place due to pancreatic insufficiency because of acute pancreatitis. Review Chronic pancreatitis is most often diagnosed when calcifications of the pancreas are noted on abdominal CT scan. The most common cause of chronic pancreatitis in the United States is alcoholism. AST to ALT ratio of at least 2:1 is consistent with alcoholic hepatitis. Other causes of chronic pancreatitis include untreated gallstone disease, medications, celiac disease, and family history.

A 23-year-old professional basketball player presents to his health care provider 3 hours before game time complaining of abdominal pain. The symptoms began approximately 8 hours ago in a diffuse fashion. Two hours later, he began feeling nauseated and vomited twice. Over the past 4 hours the abdominal pain has become more severe and well-localized in the right lower quadrant. His examination now reveals well-localized pain in the right lower quadrant inferolateral to the umbilicus. Which of the following is the most likely diagnosis? A. Acute obstruction of the appendiceal lumen by a fecalith B. Acute onset of ileocolitis C. Acute onset of ischemic colitis D. Acute Yersinia infection E. Obstruction of the ileocecal valve by a mass

A. Acute appendicitis is one of the most common surgeries performed in the United States and the most common surgical emergency in every decade of life. Although it can occur at any age, many patients are teenagers and young adults. This patient's presentation is typical for acute appendicitis, with initially poorly localized pain followed by nausea and vomiting. In classic appendicitis the pain shifts to the right lower quadrant, where it becomes more localized in the area known as McBurney point. In most patients acute obstruction of the appendiceal orifice by a fecalith initiates the acute appendicitis. The acute onset of ileocolitis (choice B) will produce diarrhea or bloody stools. Patients who have Crohn's disease may have ileocolitis, and this differential may not be evident until a CT scan of the abdomen or a surgical exploration is performed. There is no evidence to suggest an etiology for ischemic colitis (choice C), which will typically present with bloody diarrhea and often with left-sided abdominal pain. This condition tends to occur in the older population. Acute Yersinia infection (choice D) will produce acute right lower quadrant findings similar to those of acute appendicitis. It is accompanied by diarrhea, however, which is not described in this case. There is no reason to suspect obstruction of the ileocecal area by any mass (choice E) in a 23-year-old man. Such an obstruction, should it occur, would typically present with abdominal distention as a result of bowel obstruction. Review Acute appendicitis is caused by obstruction of the appendiceal orifice by a fecalith. It initially presents as acute abdominal pain, which may then become localized to the right lower quadrant (classic appendicitis), accompanied by nausea and vomiting. Appendicitis is usually a clinical diagnosis but may be confirmed with a CT scan if the signs and symptoms are atypical.

A full-term infant is brought to the office on her sixth day of life because her mother noted that she looked "yellow." The mother states that the infant is strictly breast-fed and has been eating every 2-3 hours. On examination she is noted to be jaundiced over her trunk and face. There is no scleral icterus. She is otherwise healthy. Both the mother and baby are Rh-positive. Which of the following is the most likely cause of this infant's jaundice? A. Breast milk jaundice B. Glucose-6-phosphate dehydrogenase (G6PD) deficiency C. Breastfeeding failure jaundice D. Physiologic jaundice E. Rh incompatibility

A. Breast milk jaundice is common. It is caused by a glucuronyl transferase inhibitor present in breast milk of some mothers. As a result of the inhibition of the enzyme, indirect bilirubin increases in the second week of life. Glucose-6-phosphate dehydrogenase (G6PD) deficiency (choice B) is a red blood cell enzyme defect resulting in hemolysis and the overproduction of bilirubin. It is X-linked and usually occurs in those of Mediterranean, African, or Asian descent. G6PD should be suspected in male infants who have late onset of jaundice. Breastfeeding failure jaundice (choice C) is related to decreased intake and increased enterohepatic circulation. The decreased intake is usually related to failure to establish an adequate milk supply. Breastfeeding jaundice occurs within the first week of life. Breastfeeding failure jaundice is more likely to occur as a result of the mother's milk not arriving to overcome the infant's need for fluids. This relative fluid deficit may also contribute to an increase in indirect bilirubin, which results in jaundice. This condition will correct itself when the infant starts to take more breast milk and improves hydration status. Physiologic jaundice (choice D) usually appears after 24 hours of age, slowly increasing to a peak between 3-5 days of age. In this case, the infant is 6 days old and just starting to get jaundiced; thus, she is beyond the typical range for physiologic jaundice. Rh incompatibility (choice E) is not indicated in this case, because both the mother and infant are Rh-positive. Review Breast milk jaundice is caused by a glucuronyl transferase inhibitor present in breast milk of some mothers. As a result of the inhibition of the enzyme, indirect bilirubin increases in the second week of life. Treatment is the discontinuation of breast milk for 48 hours. After that, it can be resumed. For the mother to continue to be able to breastfeed, she should pump the milk so that supply is sufficient when the breastfeeding resumes.

A 32-year-old man has a 15-year history of celiac sprue. He admits to being noncompliant with the diet prescribed over the past 6 months and has lost 15 pounds during that time in association with frequent diarrhea. A microcytic, hypochromic pattern is observed on a peripheral blood smear. Serum ferritin value is 15 ug/L (normal 20-250 ug/L). Which of the following is most likely to occur in this patient? A. Iron deficiency will produce a microcytic anemia B. Vitamin A deficiency will produce nystagmus C. Vitamin B12 deficiency will produce a megaloblastic anemia D. Vitamin D deficiency will produce hypercalcemia E. Vitamin K deficiency will produce hypokalemia

A. Celiac sprue will produce signs and symptoms of malabsorption of the proximal small bowel. Iron (along with folate and calcium) is preferentially absorbed in the proximal small bowel and is not well compensated for in the distal small intestine when there is proximal small bowel malabsorption. Vitamin A, D, and K deficiencies will occur but do not produce the described symptoms. Iron-deficiency anemia is characterized by a microcytic, hypochromic pattern on the peripheral blood smear. In addition, the serum ferritin level of less than 16 ug/L is strongly suggestive of a microcytic process. It should be remembered that the red blood cells become smaller and the ferritin levels fall before the fall in the hemoglobin and hematocrit. Vitamin A deficiency (choice B) will produce night visual disturbances rather than nystagmus. Vitamin B12 deficiency (choice C) rarely occurs in celiac sprue, because the malabsorptive process is generally more severe in the proximal small bowel, whereas vitamin B12 absorption occurs in the terminal ileum. Vitamin D deficiency (choice D) will produce hypocalcemia rather than hypercalcemia. Vitamin K deficiency (choice E) produces a coagulopathy rather than hypokalemia. Review Celiac sprue is a systemic autoimmune disease that is triggered by dietary gluten ingestion. Gluten peptides may be found in wheat, rye, barley, and other related grains. Presentation of celiac sprue may include diarrhea and failure to thrive, iron-deficiency anemia, or osteoporosis. Treatment involves lifelong avoidance of gluten-containing products and nutritional supplementation to prevent iron-deficiency anemia and osteoporosis.

A 52-year-old man with peptic ulcer disease has been on drug therapy for 3 months and has noticed changes in his bowel habits, increasing headaches, dizziness, skin rashes, loss of libido, and gynecomastia. Which of the following drugs is most likely responsible for these side effects? A. Cimetidine (Tagamet) B. Famotidine (Pepcid) C. Metronidazole (Flagyl) D. Omeprazole (Prilosec) E. Sucralfate (Carafate)

A. Cimetidine (Tagamet), an H2-receptor antagonist, can produce all the side effects exhibited when taken in high doses over a long period of time. In addition, cimetidine can alter the hepatic metabolism of several drugs and can elevate liver transaminases. Famotidine (Pepcid) (choice B) is also an H2-receptor antagonist, but it does not have the side effects of cimetidine. Metronidazole (Flagyl) (choice C) is an antibacterial and antiprotozoal drug that may present with the adverse effects of headaches, dizziness, diarrhea, and rashes, but not gynecomastia. Omeprazole (Prilosec) (choice D) is a proton pump inhibitor, and sucralfate (Carafate) (choice E) is a physical barrier to gastric acid. Neither has been associated with effects on sexual function or breast development. Review Cimetidine (Tagamet), an H2 blocker, selectively antagonizes histamine H2 receptors and is used in the management of peptic ulcer disease. It has more side effects than any of the agents in this class. It activates the CYP-450 system and elevates liver transaminase levels. It is associated with headache, gynecomastia, drowsiness, and rash.

A 66-year-old man comes to the urgent care clinic with progressive jaundice, which he first noticed 6 weeks ago. He has never had any chronic medical conditions and takes no medications. He has smoked 1 pack of cigarettes a day for 30 years and drinks a glass of wine each evening. His total bilirubin is 22 mg/dL (normal <1.9 mg/dL), with a direct (conjugated) bilirubin 16 mg/dL (normal <0.3 mg/dL). Transaminases are minimally elevated, whereas his alkaline phosphatase is about 6 times the upper limit of normal. Sonogram shows dilated intrahepatic ducts, dilated extrahepatic ducts, and a very distended, thin-walled gallbladder without stones. Which of the following is the most appropriate next step in diagnosis? A. CT scan of the upper abdomen B. Endoscopic retrograde cholangiopancreatography (ERCP) C. Exploratory laparotomy D. Percutaneous transhepatic cholangiogram (PTC) E. Serologies to define the type of hepatitis

A. Obstructive jaundice is evident by the high alkaline phosphatase and the dilated biliary ducts. Malignancy is suggested by the dilated, thin-walled gallbladder. If there is a cancer of the head of the pancreas, CT scan has a good chance of showing it in a noninvasive manner. Endoscopic retrograde cholangiopancreatography (ERCP) (choice B) or percutaneous transhepatic cholangiogram (PTC) (choice D) would be the next step if the CT scan was negative. A smaller tumor at the ampulla, the common duct itself, or even the head of the pancreas, could escape detection by the CT scan and necessitate a more invasive test. Either of these could be used, though ERCP is favored by most. Exploratory laparotomy (choice C) would be premature at this point. Serologies (choice E) would have been called for if he had very high transaminases, normal or near normal alkaline phosphatase, and an unremarkable sonogram. Review Cancer of the gallbladder or head of the pancreas should be suspected in the setting of obstructive jaundice. Obstructive jaundice is suspected in the setting of elevated direct bilirubin, elevated alkaline phosphatase, and normal transaminase levels.

A 9-month-old girl is brought to the clinic by her parents because she has an abdominal mass. Physical examination shows an umbilical defect about 1 cm in diameter, with a small bulge when the girl cries. The hernial contents can be easily reduced. The hernia is not painful and the girl is otherwise asymptomatic. She is eating well and reaching all developmental milestones. Which of the following is the most appropriate next step in management? A. No therapy unless the hernia persists beyond the age of 2 years B. Repeated injections of sclerosing agents C. Elective laparoscopic surgical repair D. Elective open surgical repair E. Urgent surgical repair

A. Small umbilical hernias can close spontaneously up to the age of 2. If they are asymptomatic and pose no immediate risk for strangulation, they should be left alone. Sclerosing agents (choice B) are not appropriate to manage a hernial sac that communicates with the rest of the peritoneal cavity. If a surgical hernia repair is done when a large distal sac has to be left in place, it might be advisable to destroy the peritoneal lining with sclerosing agents so that it does not secrete fluid. Laparoscopic hernia repair (choice C) makes sense when the size of the incision or incisions can be significantly reduced by the use of laparoscopy (for instance a bilateral inguinal hernia repair). In this case, however, one would need bigger incisions to introduce a TV camera and operating instruments than one would need to directly close a 1-cm superficial defect. Because this patient is asymptomatic, surgical intervention is not needed for a condition that may spontaneously resolve. Elective open surgical repair (choice D) is what this little girl will need if she still has the hernia past her second birthday. Urgent repair (choice E) would have been indicated if the hernia was tender or if the girl had been vomiting or getting distended at the same time that the hernia became irreducible. Review Umbilical hernias occur as a result of the umbilical ring failing to close. This defect allows protrusion of the peritoneal sac, which could contain intra-abdominal contents such as the omentum or bowel. Umbilical hernias are the only type of hernia by which spontaneous healing occurs. Most umbilical hernias will close on their own by age 4-5, often correcting themselves as infants develop abdominal musculature by crawling and walking. If the umbilical hernia fails to resolve, elective surgical correction may be considered after age 5 (due to higher risk for incarceration if the hernia persists later in life).

A 44-year-old woman delivers a 3,120 g (6 lb 14 oz) newborn boy. Her pregnancy was normal except that she noted decreased fetal movement compared to her previous pregnancies. She declined an amniocentesis offered by her obstetrician. Physical examination of the newborn reveals an infant who has facial features suggestive of Down syndrome. The infant then has bilious vomiting. A radiograph showing the kidneys, ureters, and bladder (KUB) is performed, which shows a distended and gas-filled stomach and proximal duodenum and the absence of gas in the distal bowel. Which of the following is the most likely cause of the abdominal signs and symptoms? A. Duodenal atresia B. Hirschsprung's disease C. Malrotation D. Meconium ileus E. Pyloric stenosis

A. The "double bubble" sign is pathognomonic for duodenal atresia, which is a congenital anomaly associated with Down syndrome. Two large gas collections, one in the stomach and one in the proximal duodenum are the only radiographic lucencies visible in the GI tract. Duodenal atresia is associated with bilious vomiting. Children who have Down syndrome can also have esophageal atresia, imperforate anus, endocardial cushion defects, and hypotonia. Hirschsprung's disease (choice B) can cause abdominal obstruction with delayed passage of stool in the first few days of life. It is a functional obstruction caused by lack of ganglion cells in the distal portion of the colon, which causes aperistalsis in the affected segment of colon. Radiographs would show stool throughout the abdomen but little or no air in the rectum and a dilated "megacolon" just proximal to the affected segment. Malrotation (choice C) is the failure of the GI tract to properly rotate during embryogenesis. It can present with vomiting, but plain radiographs would be nonspecific. A barium enema would demonstrate an abnormally placed cecum. Malrotation can lead to a midgut volvulus, which is a surgical emergency, because the midgut can twist on its blood supply, resulting in ischemia. Meconium ileus (choice D) occurs when meconium becomes obstructed in the terminal ileum. It is the most common presentation of cystic fibrosis in the neonatal period. Mild obstruction can be treated with enemas, but complete obstruction may require operative management. Pyloric stenosis (choice E) rarely occurs in the newborn period. It presents in the first month of life with progressive vomiting without abdominal distention. It is caused by hypertrophy of the pyloric muscle at the junction of the stomach and duodenum, which interferes with passage of food and liquid further into the small bowel. Radiographs may show a large dilated gastric bubble, but there would be no "double bubble" because the obstruction is proximal to the duodenum. Review Duodenal atresia may be caused by congenital GI disorders. Patients who have Down syndrome may have duodenal or anal atresia or duodenal or anal stenosis. Patients who have duodenal atresia may have abdominal distention and bilious vomiting. Plain radiographs will show evidence of intestinal atresia with a double collection of air in both the stomach and proximal small bowel.

A 44-year-old woman has a 10-year history of progressive dysphagia without pain. The dysphagia affects all kinds of food, without preference for solids. The food sticks in the xiphoid area and can be helped along by drinking large amounts of water and sitting up straight until the whole bolus passes into the stomach. She also describes many episodes of regurgitation of foul-smelling but undigested food, either when she leans forward or when she is asleep at night. She used to be heavier and lost weight as the disease progressed. A barium swallow shows a massively dilated proximal esophagus with a narrow, beak-like appearance in the lower sphincteric area. Which of the following is the most likely original pathophysiology that explains the development of this problem? A. Failure of the lower esophageal sphincter to relax B. Reflux of acid gastric juice into the lower esophagus C. Replacement of smooth muscle by fibrous tissue D. Spasm of the lower esophageal sphincter E. Weak or nonexistent peristaltic activity in the upper esophagus

A. The clinical and radiologic picture is that of achalasia, which starts with failure of the lower esophageal sphincter to relax. The word "achalasia" in fact means "failure to relax." In addition to having inability of the lower sphincter to relax, patients also have poor tertiary stripping waves (peristalsis) in the esophagus, which causes an inability of the patient to pass the food from the esophagus into the stomach. Because this is a motor disorder, esophageal manometry could also be used to make this diagnosis. Acid reflux (choice B) patients would have a long history of heartburn and pain leading eventually to stricture. Patients who have acid reflux would have retained food in the esophagus with dysphagia, but this retained food would be the result of a stricture and this would be readily identified on the barium swallow or esophageal manometry studies. Replacement of smooth muscle by fibrous tissue (choice C) is the pathophysiologic process with scleroderma. Other manifestations of that disease would include induration of the skin, arthralgias, and Raynaud phenomenon. As noted, the initial problem is failure of the lower esophageal sphincter to relax. It is not spasm of the lower esophageal sphincter (choice D), as some might imagine, or a primary lack of propulsive motility (choice E). Review Achalasia is an esophageal motor disorder that is characterized by the failure of the lower esophageal sphincter to relax and also by the loss of tertiary stripping waves in the lower esophagus. Because it is a motor disorder, the patient will have dysphagia for both solids and liquids from the onset of the condition. This disease can slowly progress. Endoscopy, barium swallow, and esophageal manometry can all be used in the evaluation of this condition. Definitive therapy is with pneumatic dilation or laparoscopic cardiomyotomy.

A 54-year-old man has cirrhosis, with obstruction of the portal circulation within the liver. Portal blood could still be conveyed to the caval system via which of the following? A. Azygos and hemiazygos veins B. Gonadal veins C. Internal iliac veins D. Splenic vein E. Vesical venous plexus

A. The esophageal venous plexus, which drains into the azygos and hemiazygos veins within the thorax, has anastomoses with branches of the left gastric vein. Following blockage of the portal vein, portal blood thus may enter the superior vena cava via the azygos system. Other important portacaval connections include the superior rectal vein with the middle and inferior rectal veins; paraumbilical veins with epigastric veins (engorgement of these vessels results in caput medusae); and the colic and splenic veins with renal veins and veins of the posterior body wall. The gonadal veins (choice B) exclusively drain the gonads (although in the female, the ovarian vein communicates with the uterovaginal plexus). These vessels have no anastomoses with portal vessels. The internal iliac veins (choice C), which drain most of the pelvis and much of the inferior extremities, have no demonstrated portal anastomoses. The splenic vein (choice D) is incorrect, because it is itself a component of the portal venous system. The vesical venous plexus (choice E), which is situated well within the pelvis and drains the bladder and prostate (or uterus and vagina), has no association with portal vessels. Review Cirrhosis occurs as a result of blockage of the hepatic venous outflow. This results in expansion of the splenic and gastric blood volume and expands the hepatic lymph formation. This leads to intrahepatic hypertension, which results in the increase of sodium and loss of effective blood volume. There is also a decrease in the production of albumin, which also complicates the picture of ascites that occurs as a result of this portal hypertension. If there is blockage to the circulation via the portal system, alternative drainage of the liver can occur through the azygous and hemiazygous veins.

A 9-month-old girl is seen in the health care provider's office because of failure to gain weight. Her length and weight are both below the fifth percentile for her age. The patient's chart indicates that at age 6 months, her length and weight were at the fiftieth percentile. A careful history reveals that the mother returned to work when the infant was 6 months old, and the grandmother has assumed the care of the infant since then. She receives 6-8 oz of iron-fortified, cow protein-based formula every 4 hours. Which of the following is the best initial step in the management of this infant? A. Ask how the formula is mixed B. Obtain a detailed family history for lactose intolerance C. Obtain a stool specimen D. Obtain a sweat chloride test E. Obtain a urinalysis

A. This 9-month-old infant presents with failure to thrive, which is determined by the decline in her length and weight to below the fifth percentile. Failure to thrive can be either organic or inorganic (i.e., social). Although it is important to identify the causes of organic failure to thrive, most cases are inorganic. In this clinical vignette, because the reduction of weight happened after the mother returned to work, an inorganic cause is likely. One of the most common causes of failure to thrive is improper preparation of the formula, resulting either from an incorrect water-to-formula ratio or from poor mixing techniques. In addition to obtaining a detailed history of how the formula is mixed, it might be very useful to have the caretaker actually demonstrate how he or she prepares the formula. In this case, the most likely reason that the infant has failure to thrive is that the grandmother has improperly mixed the formula. Also, a 9-month-old infant should be eating some regular table foods, including meats, poultry, and vegetables. Lactose intolerance (choice B) is an uncommon cause of failure to thrive. It usually presents with abdominal pain, bloating, and diarrhea. Obtaining a stool specimen (choice C), sweat chloride test (choice D), or urinalysis (choice E) might be helpful if no identifiable inorganic causes of failure to thrive are identified. Urinalysis is helpful in screening for renal disease. A sweat chloride test is used to detect cystic fibrosis. Stool specimens can be useful in a great variety of gastrointestinal disorders, such as gastroenteritis, parasitic infection, and fat malabsorption. Review Failure to thrive refers to conditions in which a child has less than expected growth over the first 3 years of life. Undernutrition is a common reason for failure to thrive, and is in fact the most common cause of failure to thrive. Inadequate intake results in the child not having enough calories to support adequate growth. This inadequate intake may be caused by lack of knowledge of developmentally appropriate food and poor caregiver skills. Food diaries and education may be helpful to correct these deficits in knowledge to correct the inadequate nutrition.

A 19-year-old woman comes to the urgent care clinic with a 2-week history of frequent episodes of loose stools. The symptoms are accompanied by severe fecal urgency, and she has awoken with diarrhea several nights weekly. Over the past week, the stools have become increasingly bloody. A sigmoidoscopy reveals continuous, symmetric inflammation from the anal verge to the proximal sigmoid colon. Which of the following infections is most likely causing this patient's symptoms? A. Campylobacter jejuni B. Cryptosporidium C. Giardia lamblia D. Herpes simplex E. Yersinia enterocolitica

A. This patient has an acute onset of bloody diarrhea consistent with colitis, which is confirmed with the sigmoidoscopy. Of the organisms listed, Campylobacter jejuni is the one most likely to cause these symptoms. This organism can cause diarrhea in all age groups, although the peak of incidence is in young children. C. jejuni can be acquired through exposure to contaminated food (especially undercooked poultry) or water, or through exposure to infected domestic or wild animals. The diarrhea can be either watery or bloody and is often accompanied by a sometimes high fever. White blood cells are commonly seen in the fecal material. You should also be aware that there appears to be an association between summer outbreaks of diarrheal disease caused by this organism and subsequent development of the peripheral nervous system autoimmune disease Guillain-Barré syndrome. Cryptosporidium (choice B) causes a small bowel rather than colonic infection, as was demonstrated by sigmoidoscopy. Giardia lamblia (choice C) causes nausea, vomiting, eructations, and upper gastrointestinal symptoms more commonly than diarrhea. Herpes simplex (choice D) does not cause a bowel infection. Yersinia enterocolitica (choice E) affects the area of the cecum rather than the distal colon, causing right lower quadrant findings and diarrhea that can mimic the symptoms of Crohn's disease. Review Campylobacter jejuni infection is the most commonly reported bacterial cause of acute gastroenteritis in developed countries. It causes 80 to 90% of acute enteritis of all identifiable causes. Most human-related causes of Campylobacter are caused by infected poultry that is undercooked or by consuming unpasteurized milk. It is the most common bacterial pathogen causing bloody diarrhea (bacterial dysentery) in the United States. Fluid and electrolyte replacement is essential in treatment, and diarrhea typically resolves in 5 to 7 days. Only a small number of patients require antimicrobial therapy, with erythromycin or azithromycin being first-line therapy, or with ciprofloxacin, which can also be used to treat other bacterial causes of gastroenteritis.

A 5-month-old boy is brought to the emergency department by his parents because he has been having crying spells alternating with lethargy for the past several hours. He was apparently doing fine in the morning and feeding well, when he suddenly started crying painfully, drawing his legs to his chest and kicking them in the air. This would last for 10-15 minutes, and then he would act very tired and drowsy. First they thought it was colic, but after several episodes of similar attacks he started vomiting and passed a stool that contained red blood and mucus, and they immediately brought him to the hospital. He has no medical problems and has been developing well. He is breast-fed and the mother had started adding solids over the previous 3-4 weeks. On physical examination, the patient is a chubby infant in moderate distress, pale, and diaphoretic. His temperature is 36.7ºC (98.0ºF), pulse is 100/min, and respirations are 22/min. A complete blood count and leukocyte differential are within normal limits. His abdomen is mildly tender to palpation but not distended, and there is a sausage-shaped mass in the right hypochondrium. The right lower quadrant feels empty on palpation. His stool is reddish in color and with hints of gelatinous material. A plain abdominal radiograph shows absence of air in the right lower quadrant. Which of the following is the most appropriate next step in the treatment? A. Barium enema B. Immediate surgical reduction C. Laparoscopy D. Manual reduction E. Observation in expectation of spontaneous resolution

A. This patient has intussusception, which occurs when a portion of the gastrointestinal tract slips or telescopes into the portion just distal to it. A barium enema is diagnostic and therapeutic, and is the initial diagnostic and treatment of choice for symptoms of less than 48 hours' duration. Most intussusceptions are ileocecal, and most have no known cause. Intussusception is most commonly seen in children age 5-24 months (range, 3 months to 6 years) most commonly occurring in the first year of life. Acute onset of cramping, colicky abdominal pain is the hallmark of intussusception. Patients may have vomiting. As the obstruction progresses, the patient may develop fever and lethargy. The classic currant jelly stool (stool with red blood and mucus) is a late finding. Passing a stool may temporarily relieve pain. A sausage-shaped mass may be palpated in the right upper quadrant on physical examination. If the obstruction becomes complete, there is abdominal distention and shock may ensue. A coil-spring sign is seen as the barium fills the obstruction. Air enemas may also diagnose and treat with greater safety. Ultrasound may be helpful in establishing the diagnosis. Intussusception is an emergency and should be reduced as quickly as possible. Hydrostatic reduction with air or contrast barium enema is successful approximately 50% of the time for symptoms lasting longer than 48 hours, and 75-80% for symptoms lasting less than 48 hours. It should not be done in the face of prolonged intussusception, peritonitis, or perforation. Surgery is recommended in those cases or after failure of hydrostatic reduction. Untreated intussusception is almost uniformly fatal because of bowel perforation and peritonitis. Immediate surgical reduction (choice B) is not indicated in a patient who has had symptoms for less than 48 hours and who has no signs of intestinal necrosis or shock. A hydrostatic reduction (i.e., air or contrast barium enema) should be attempted first. Laparoscopy (choice C) is emerging as a treatment modality and its use will most certainly grow as familiarity with the technique increases. It is, however, not yet the first line of treatment for intussusceptions, because conservative treatment with barium enema is usually curative. Manual reduction (choice D) can be attempted during surgical intervention before resecting the affected bowel. This should be performed only if hydrostatic reduction has failed and there are no signs of ischemic necrosis of the affected bowel segment. Observation in expectation of spontaneous resolution (choice E) is not indicated, as untreated intussusception is almost universally fatal. Review Intussusception may cause bowel obstruction in young children and this occurs as a result of the prolapse of one part of the intestine into the lumen of an immediately distal adjoining part. This telescoping of one part of the intestine into another part of the intestine leads to venous obstruction and bowel wall edema and, if untreated or uncorrected, can lead to bowel necrosis, perforation, and even death. Underlying causes of intussusception in infants include hyperplasia of Peyer patches and lymphoid tissue in the intestinal wall, which may be caused by viral infection. Diagnosis and treatment can be accomplished with either air or contrast reagent barium enema.

A 42-year-old obese woman experiences episodic abdominal pain. She notes that the pain increases after the ingestion of a fatty meal. Action of which of the following hormones is responsible for the postprandial intensification of her symptoms? A. Cholecystokinin (CCK) B. Gastrin C. Pepsin D. Secretin E. Somatostatin

A. This woman has a risk profile (female, fat, forties) and symptomatology consistent with gallstones (cholelithiasis). As would be expected, contraction of the gallbladder following a fatty meal often exacerbates the pain caused by gallstones. Cholecystokinin (CCK) is the hormone responsible for stimulation of gallbladder contraction; the release of CCK is stimulated by dietary fat. It is produced in I cells of the duodenum and jejunum. In addition to gallbladder contraction, CCK also stimulates pancreatic enzyme secretion and decreases the rate of gastric emptying. Gastrin (choice B) is produced by the G cells of the antrum and duodenum. Gastrin stimulates the secretion of HCl from the parietal cells and pepsinogen from the chief cells of the stomach. Gastrin secretion is stimulated by gastric distention, digestive products (e.g., amino acids), and vagal discharge. Pepsin (choice C) is a protease produced by the chief cells of the stomach (as pepsinogen). It is involved in the digestion of proteins. Pepsinogen release is stimulated by vagal stimulation, gastrin, local acid production, secretin, CCK, and histamine. Secretin (choice D) is produced by the S cells of the duodenum. It is secreted primarily in response to acidification of the duodenal mucosa. Secretin stimulates the secretion of bicarbonate-containing fluid from the pancreas and biliary ducts. This neutralization allows pancreatic enzymes to function. Secretin also inhibits gastric acid production and gastric emptying. Somatostatin (choice E) is produced by the D cells of the pancreatic islets and in the gastric and intestinal mucosa. Somatostatin is an inhibitory hormone; it inhibits most gastrointestinal hormones, gallbladder contraction, gastric acid and pepsinogen secretion, pancreatic and small intestinal fluid secretion, and both glucagon and insulin release. Review Fatty food intake results in the release of cholecystokinin (CCK), which causes the stimulation of gallbladder contraction. To assess gallbladder function, a CCK challenge test can assess for cystic duct obstruction and functional status of the gallbladder. Intake of a high fatty meal will also cause CCK release, and the ejection fraction of the gallbladder can also be determined.

A 77-year-old woman presents to the emergency department with 12 hours of passing bright red blood from her rectum in increasingly large amounts. She becomes dizzy on standing and appears pale. Her blood pressure is 112/60 mm Hg on laying supine and 90/56 mm Hg on sitting upright with legs dangling over the side. Abdominal examination is normal. Which of the following is the most likely diagnosis? A. Bleeding diverticulum in the appendix B. Communication between an arteriole and venule in the cecum C. Laceration at the gastroesophageal junction D. Rectal polyp E. Perforation of a sigmoid diverticulum

B. A communication between an arteriole and venule in the cecum is a description of a vascular ectasia, also known as an arteriovenous (AV) malformation. This is a common cause of painless colonic bleeding in the elderly and may present with acute gastrointestinal bleeding (as in this case), chronic gastrointestinal bleeding, or iron-deficiency anemia. These lesions may be difficult to demonstrate, as the bleeding may be intermittent or the colon may be so full of blood that the site of origin is obscured. Techniques used to demonstrate bleeding AV malformations include colonoscopy, intraoperative endoscopy, and visceral angiography. Treatment of these lesions is problematic, because many patients will subsequently develop new or recurrent bleeding vessels. A bleeding diverticulum in the appendix (choice A) does not occur. Diverticular bleeding is very common in the elderly, but it is usually present in the sigmoid colon. A laceration of the gastroesophageal junction (choice C) (i.e., a Mallory-Weiss tear) may produce bright red blood per rectum if the bleeding is profound. It more often presents with a typical history of vomiting and retching preceding the bleeding, however, which is not described in this question. Rectal polyps (choice D) rarely produce large volume blood loss. Perforation of the sigmoid diverticulum (choice E) does not result in bleeding but will result in acute diverticulitis. Review AV malformation or angiodysplasia usually occurs in patients older than age 65 years. It produces intermittent, mild, or severe episodes of painless hematochezia. Significant bleeding may cause orthostatic hypotension, tachycardia, and low blood pressure. Diagnosis may be made via colonoscopy, which identifies these malformations. Treatment during colonoscopy can be with bicap therapy or injection of the vessel with a sclerosing agent.

A 43-year-old man with recurrent history of peptic ulcer disease associated with diarrhea and strong family history of duodenal ulcer disease is suspected of having Zollinger-Ellison syndrome (gastrinoma). Secretin (1 U/kg) is given as a rapid IV injection to test for gastrinoma. Which of the following results would support the existence of gastrinoma following secretin administration? A. Gastrin release from antrum B. Increased serum gastrin C. Inhibition of gastric emptying D. Inhibition of gastric secretion E. Stimulation of pancreatic HCO3- secretio

B. Gastrinomas are gastrin-secreting tumors usually present in the pancreas. Patients who have gastrinoma have high serum gastrin levels, which lead to hypersecretion of gastric acid and consequent duodenal and jejunal ulcers. Injection of secretin is the most specific and easiest test for gastrinoma. Secretin inhibits antral release of gastrin (choice A), but it stimulates release of gastrin from gastrin tumors (gastrinoma) in almost all patients. A doubling of serum gastrin 5-10 minutes after administration of secretin (1 U/kg) coupled with acid hypersecretion and increased basal serum gastrin strongly indicates the presence of gastrinoma. Secretin can inhibit gastric emptying (choice C), inhibit gastric secretion (choice D), and stimulate pancreatic HCO3- secretion (choice E), but these effects are not diagnostic for gastrinoma. Review Zollinger-Ellison syndrome is caused by a gastrin-producing neuroendocrine tumor which causes gastric acid hypersecretion and peptic ulceration resistant to therapy and multiple in number. Ulcers appear in atypical locations. It can also be associated with abdominal pain, diarrhea, and GERD. An elevated fasting serum gastrin level >1,000 is diagnostic. There is high variability among commercial labs in determining this level, which limits the utility of this test. Administration of secretin will also result in increased serum gastrin levels > 200 within 15 minutes of secretin infusion.

The health care provider becomes concerned when after 30 hours, a newborn infant has not passed meconium. He was full term with a birth weight of 3,856 g (8 lb 8 oz). The pregnancy was uncomplicated. The infant appears well, with no respiratory distress. Slight abdominal distention is noted. Rectal examination reveals a slightly tight rectum and results in a greenish gush of stool. Which of the following tests will probably confirm the likely diagnosis? A. A stool culture B. A rectal biopsy C. A barium enema D. An alpha1-antitrypsin level E. A serum TSH level

B. Hirschsprung's disease or congenital aganglionic megacolon is caused by a congenital absence of the ganglion cells of both the Meissner and Auerbach plexuses. It is the most common cause of lower intestinal obstruction in the neonatal period. In early childhood it may present as chronic constipation with intermittent fecal soiling. It occurs predominantly in males and there is an increased family incidence. Surgical treatment is indicated, but the diagnosis is confirmed by a suction biopsy that can be easily performed without general anesthesia. The biopsy specimen would reveal an absence of ganglion cells in the submucosal and myenteric plexuses. A stool culture (choice A) would be performed if one were entertaining a bacterial cause of gastroenteritis, especially in a hospitalized patient. Vomiting, diarrhea, and abdominal distention in a newborn are unlikely to be caused by gastroenteritis, however, especially in this case with a tight sphincter noted on rectal examination. A barium enema (choice C) may be indicated in suspected cases of Hirschsprung's disease, but it is the biopsy specimen that makes the diagnosis. The barium enema in this case of Hirschsprung's disease would reveal a dilated proximal bowel with evidence of a contracted distal rectum. An alpha1-antitrypsin level (choice D) would not be indicated in this case. It is obtained when one suspects an alpha1-antitrypsin deficiency. Affected infants would present with jaundice, acholic stools, and hepatomegaly. A serum TSH (choice E) would be performed if a newborn infant was thought to have hypothyroidism. In the United States, most states have mandatory newborn screening for thyroid disease. Frequently, congenital hypothyroidism is asymptomatic, but it may present with symptoms of constipation, lethargy, poor feeding, mottling, and prolonged jaundice. The typical features in this case are more suggestive of Hirschsprung's disease. Review Hirschsprung's disease is a congenital condition with partial or complete colonic obstruction associated with an absence of intramural ganglion cells. This causes the colon lumen to become tonically contracted, resulting in an obstruction. The rectum and sigmoid colon are the most common sites for this congenital absence to occur, though the entire colon may be affected. This condition may be seen with Down syndrome or other congenital anomalies. Gene mutations on chromosome 10 and 21 have been identified. Failure to pass meconium in the first 24-48 hours is seen in the majority of patients. Treatment is chronic bowel irrigation, with definitive surgery to remove the distal aganglionic segment with a pull-through of normal bowel.

Which of the following viruses produces disease or sequelae that is/are more severe if the infection occurs at a very young age? A. Epstein-Barr virus B. Hepatitis B virus C. Measles virus D. Poliovirus E. Varicella zoster virus

B. Infection with hepatitis B virus (HBV) at birth or a very young age is associated with chronic HBV infection and the development of hepatocellular carcinoma later in life. In fact, infants born to hepatitis B surface antigen (HBsAg)-positive mothers are commonly infected, and approximately 90% become chronic carriers of the virus. In chronic carriers, hepatocellular carcinoma develops at an incidence more than 200 times higher than in noncarriers. The current recommendation for infants born to HBsAg-positive mothers is administration of hepatitis B immunoglobulin (HBIg) in the delivery room, with the first dose of the hepatitis B vaccine given at the same time or within 1 week. The second and third doses of the vaccine are then given at 1 and 6 months. With this protocol, 94% protection is achieved. Epstein-Barr virus (EBV; choice A) is the agent of heterophile-positive infectious mononucleosis. In children, primary EBV infection is often asymptomatic. Measles virus (choice C) often causes a more severe disease in adults. The incidence of complications, including pneumonia, bacterial superinfection of the respiratory tract, bronchospasm, and hepatitis, is much higher in adults older than age 20 years than in children. Poliovirus (choice D) causes asymptomatic or inapparent infections 95% of the time. Frank paralysis occurs in approximately 0.1% of all poliovirus infections. The probability of paralysis increases with increasing age, however. Varicella zoster virus (choice E) is the agent of chickenpox and shingles. In immunocompetent children, it is a benign illness with a mortality of less than 2 per 100,000 cases. This risk is increased more than 15-fold in adults. Much of the increase is because of varicella pneumonitis, a complication that occurs more frequently in adults. Review The risk for becoming a chronic carrier (failing to completely clear the hepatitis B virus) is increased in patients who acquire the disease early in life. This inability to clear the hepatitis B virus from the body is one reason that newborn vaccination is performed within 12 hours of birth in infants born to mothers who are known to be positive for hepatitis B infection.

A 74-year-old man comes to the emergency department after two episodes of passing large amounts of bright red blood per rectum that occurred without pain. He has a long history of chronic constipation without a recent change in his bowel habits. A flexible sigmoidoscopy reveals multiple large diverticula in the sigmoid colon with copious amounts of fresh blood in this region. No other abnormalities are seen on examination of the splenic flexure, where formed brown stool is encountered. Which of the following is the most likely underlying cause for his bleeding? A. Diffuse descending colon ischemia B. Hemorrhage from a single diverticular arteriole C. Oozing from an inferior mesenteric vein D. Thrombosis of a branch of the inferior mesenteric artery E. Thrombosis of internal hemorrhoids

B. One of the important complications of diverticular disease is massive bleeding that can be potentially life-threatening. The mechanism involved in producing bleeding in diverticular disease is usually the disruption of an often single arteriole or small artery in a diverticulum. The diverticulum involved is often, but not always, particularly large. The small vessel can be sufficiently stretched by the diverticulum that it cannot undergo contraction, which normally stops blood flow through this size vessel after damage. In many cases, the bleeding may stop spontaneously. If it recurs within a few days, surgical resection of the involved segment of bowel (if it can be identified, which can be problematic) may be required. The patient does not have any evidence of a diffuse descending colon ischemia (choice A), which would have produced an obviously edematous bowel mucosa, often with many focal hemorrhages. Inferior mesenteric veins course primarily in the serosal fat of the bowel, and consequently do not ooze into the bowel lumen as a cause of bleeding (choice C). Thrombosis of a branch of the inferior mesenteric artery (choice D) would cause an ischemic colitis with features as described in the discussion of choice A, which are not described in this patient. Thrombosis of internal hemorrhoids (choice E) is very common but does not cause bleeding per se. Review Diverticular disease may be totally asymptomatic until the patient develops bleeding or pain when the diverticulum becomes inflamed. The most common cause of massive painless rectal bleeding is diverticulosis. The bleeding may stop spontaneously or surgical resection of the involved segment may become necessary if the bleeding does not stop. Management of these patients requires hemodynamic support until the patient stabilizes or until surgery becomes necessary. Diverticular disease is a common condition for people living in the United States, and low dietary fiber is a contributing factor for its development.

A 24-year-old student complains of midepigastric pain that she describes as a "dull ache" that is relieved by eating. She has awakened from sleep on several occasions at 2 AM because of severe exacerbation of these symptoms, which are relieved with magnesium hydroxide. She takes frequent acetaminophen for menstrual cramping. Which of the following is the most likely cause of her symptoms? A. Autonomous gastrin secretion B. Gram-negative organism C. Gram-positive organism D. Prostaglandin inhibition E. Vagal inhibition

B. Peptic ulcer disease, particularly duodenal ulceration, is strongly suggested by chronic midepigastric pain that is severe enough to awaken a patient at night and is relieved by antacids such as magnesium hydroxide. Although the differential diagnosis listed in textbooks for peptic ulcer disease is long, most patients who have ulcer symptoms are either taking nonsteroidal anti-inflammatory agents (NSAIDs) or are colonized by the gram-negative organism Helicobacter pylori. This patient is not taking NSAIDs, so H. pylori colonization is the most likely answer. This organism colonizes the mucous layer that lines the stomach and disrupts the integrity of the mucus, predisposing for both chronic gastritis and peptic ulcer disease. Autonomous gastrin secretion (choice A) by a gastrinoma, as occurs in Zollinger-Ellison syndrome, would cause multiple large complicated ulcers, often in association with a secretory diarrhea. Choice C is wrong because H. pylori is gram-negative, not gram-positive. Prostaglandin inhibition (choice D) is the mechanism whereby NSAIDs cause ulcers, but this patient is not taking these drugs. Vagal inhibition (choice E) is actually a treatment for ulcer disease when performed in order to stop acid secretion. Review The majority of duodenal ulcers are caused by H. pylori, which is a gram-negative organism. When ulceration occurs in this setting, the patient should have eradication of this organism performed. Various treatment modalities are used for eradicating this organism, with the common eradication regimens including proton pump inhibitors.

A 45-year-old chronic alcoholic presented to the emergency department 5 years ago with 24 hours of epigastric pain radiating to his back, nausea, and vomiting. He gradually recovered from that acute episode. Over the next 5 years, he is repeatedly admitted for similar symptoms. He then presents with gradual onset of weight loss, midabdominal pain radiating to his back, and steatorrhea. Which of the following conditions has most likely occurred? A. Cholangiocarcinoma B. Exocrine insufficiency of the pancreas C. Gastric outlet obstruction D. Pancreatic adenocarcinoma E. Scarring of the entire length of the common bile duct

B. Severe epigastric pain radiating to the back and accompanied by nausea and vomiting suggests pancreatitis. Pancreatitis is most likely to be encountered in alcoholics (such as this man) and patients who have biliary tract disease. This patient has a history of recurrent alcoholic pancreatitis. The development of gradual weight loss, chronic pain radiating to the back, and steatorrhea suggests that he has now developed chronic pancreatitis. This condition is often complicated by both endocrine and exocrine insufficiency of the pancreas, secondary to loss of much of the tissue of the pancreas to the disease process. Cholangiocarcinoma (choice A) produces jaundice, has a very poor prognosis, and usually occurs in patients who have a prior history of primary sclerosing cholangitis. Primary sclerosing cholangitis is a known complication of inflammatory bowel disease, particularly ulcerative colitis. Gastric outlet obstruction (choice C) would present with postprandial vomiting and early satiety. Pancreatic adenocarcinoma (choice D) may also occur in this patient but is generally present in older individuals. Pancreatic adenocarcinoma is independent of the patient having chronic pancreatitis. It typically presents with painless obstructive jaundice and weight loss. There is no evidence of scarring of the common bile duct (choice E), which would produce jaundice. Review Patients who have chronic pancreatitis can develop exocrine dysfunction as a result of inflammation and fibrosis of the gland. As a result of this tissue destruction, patients develop malabsorption with greasy stools, weight loss, and nutritional deficiencies. These patients need to be treated with pancreatic enzyme supplementation to replace the missing enzymes that the pancreas can no longer manufacture.

A 9-month-old boy is brought to the health care provider's office because his weight is persistently below the tenth percentile. His mother states that the infant seems to be hungry all the time and usually consumes 8-12 oz of formula every 2-3 hours in addition to some table food. He also has frequent, bulky, and malodorous stools. He does not take any medications. A malabsorption syndrome is suspected. The results of which of the following tests will most likely be abnormal? A. Abdominal radiography B. Fecal fat quantification C. Serum albumin D. Stool culture for Clostridium difficile toxins E. Stool smear for leukocytes and eosinophils

B. The most useful test for the 9-month-old infant in this clinical vignette is fecal fat quantification. Fat malabsorption is likely, with poor weight gain and frequent malodorous stools. A detailed family history and physical examination are also useful in evaluation of fat malabsorption syndrome. In a stool sample collected over 3-4 days, fat excretion should not exceed 15% of dietary fat in an infant, or 10% in an older child. When a malabsorption syndrome is confirmed, a sweat chloride test is indicated to evaluate for the possibility of cystic fibrosis, which is a common cause of fat malabsorption. Abdominal radiography (choice A) plays no role when a fat malabsorption syndrome is suspected. Serum albumin (choice C) is abnormal when there is malabsorption or wasting of protein. It will not be helpful in this case of fat malabsorption. Infection with Clostridium difficile can cause persistent diarrhea. It is most commonly associated with antibiotic use. A stool culture for C. difficile toxin (choice D) is used to evaluate for infection. In this case, C. difficile colitis is unlikely. Stool smear for leukocytes and eosinophils (choice E) is useful when an inflammatory or allergic disorder of the bowel is suspected. Review Patients who have cystic fibrosis often have several manifestations that would make one suspect this condition. These patients may have failure to pass meconium, failure to thrive despite a voracious appetite, wet-sounding cough, recurrent upper respiratory tract infections and pneumonia, chronic sinusitis, and stool malabsorption. Newborn screening should pick up these patients, who have an elevated immunoreactive trypsinogen at birth followed by a positive sweat chloride test. This condition is a genetic disease, making family history a big risk for acquiring this condition. Pancreatic dysfunction caused by cystic fibrosis is the most common manifestation, but death is typically caused by lung disease and infection.

A 70-year-old man comes to the emergency department complaining of abdominal pain. He describes the pain as crampy and primarily located in his left lower quadrant. He has had minimal nausea, but complains of constipation. His past medical history is significant for hypertension, hyperlipidemia, and gout. His medications include atenolol and simvastatin. He is allergic to penicillin. His temperature is 38.0º C (100.4º F), blood pressure is 140/60 mm Hg, pulse is 100/min, and respirations are 20/min. His physical examination is significant for tenderness to palpation at the left lower quadrant without rebound or guarding. His stool is negative for occult blood. His heart and lung examinations are unremarkable. Which of the following is the most likely diagnosis? A. Appendicitis B. Diverticulitis C. Diverticulosis D. Ischemic colitis E. Sigmoid volvulus

B. The presence of cramping, left lower quadrant pain associated with fever and constipation is a classic presentation of diverticulitis. Appendicitis (choice A) becomes less likely given that the patient's pain is localized in the left lower quadrant. The classic pain of appendicitis is localized to the right lower quadrant. Diverticulosis (choice C) is incorrect since this term refers only to the presence of diverticula. Ischemic colitis (choice D) is unlikely given that this condition is typically associated with abdominal pain on examination that is out of proportion to the presenting complaint, post-prandial abdominal discomfort, and rectal bleeding. Sigmoid volvulus (choice E) is unlikely since we have not been informed that the patient has a large region of hyperresonance over his abdomen, as would be expected with the bowel distension seen with volvulus. Review Acute diverticulitis consists of left lower quadrant pain, tenderness, fever, and constipation. Rebound tenderness in the left lower quadrant may occur, and CT scan can show thickening of the bowel wall, mass, abscess, and some streakiness in the mesenteric fat. Treatment is with antibiotics and analgesia.

After passing his physical examination, a 19-year-old army recruit gives urine and blood samples for further testing. Serum analysis yields elevated ALT, HBsAg, anti-HBc, HBeAg, and bilirubin. All other values are normal. What is the hepatitis B status of this recruit? A. Asymptomatic carrier B. Active carrier C. Fulminant hepatitis B D. Recovered from acute self-limited HBV E. Vaccinated against HBV

B. The presence of elevated ALT, HBsAg, anti-HBc, HBeAg, and bilirubin all point to active hepatitis B. The HBeAg is known as the envelope, and this test parallels the level of active virus replication and predicts the level of hepatitis B infectivity. An asymptomatic carrier (choice A) would not have elevated ALT and bilirubin. The absence of findings on physical examination rules out fulminant hepatitis B (choice C). Recovery from acute self-limited HBV (choice D) is associated with the presence of anti-HBs and absence of HBsAg and HBeAg. Someone who is vaccinated against HBV (choice E) has only anti-HBs in serum and the patient never develops antidotes to the core. Only people with true infection ever have positive core antibodies. Review Patients who have hepatitis B will have measurable markers in their blood that will determine the status of the patient who has regard to infectivity. Patients who are successfully immunized against hepatitis B will have +HBsAb. Patients who have recent infection will develop HBsAg. If the patient becomes infected with HBV and goes on to clear the infection (spontaneously or with treatment), the patient will undergo a change from +HBsAg to negative status with development of +HBsAb. True infection will cause +HBcAb formation, first with IgM (recent) followed by IgG becoming positive. The IgM core antibody may be the only marker while the patient is recovering from the hepatitis B infection, and this is known as the core window. Vaccination will not cause any reaction to the core. HBeAg is a marker of active viral replication and a marker for how potentially infectious the person is if someone had a blood exposure to that patient. Patients who have active carrier states will have +HBsAg, +HBcAb, and +HBeAg, with this pattern showing active viral replication.

A 53-year-old man is admitted to the hospital for fever and abdominal pain. He has a history of cirrhosis and is known to be hepatitis C positive. He was diagnosed with cirrhosis 4 years ago. He denies any alcohol or tobacco use. His only medications are spironolactone (Alactone) and propranolol (Inderal). He reports that 5 days ago he had a temperature of 38.9º C (102º F) along with a gradual onset of diffuse abdominal pain. On examination his temperature is 38.3º C (101º F), blood pressure 100/50 mm Hg, and pulse 110/min and regular. Lungs are clear. There are numerous spider angiomata on the thorax and back, and the abdomen appears distended. Laboratory studies show: Leukocytes 13,200/mm3 (normal 5,000-10,000) Hematocrit 33% (normal 41-50%) Prothrombin time 15.2 seconds (normal 11-13 seconds) Albumin 0.1 g/dL (normal 3.4-5.4 g/dL) Sodium 135 mEq/L (normal 135-145 mEq/L) Potassium 4.7 mEq/L (normal 3.5-5.3 mEq/L) Which of the following is the most appropriate next step in diagnosis? A. Abdominal CT scan B. Abdominal paracentesis C. Abdominal ultrasound D. Electrocardiogram E. Lumbar puncture

B. The prevalence of infection of ascites fluid, so called spontaneous bacterial peritonitis (SBP), is estimated at 50% in hospitalized cirrhotic patients. This patient has had long-standing cirrhosis, has an extremely low serum albumin level, and is on spironolactone (a direct aldosterone antagonist), which is prescribed to patients with ascites. All febrile patients admitted with ascites must have abdominal paracentesis performed to both determine the cause for the ascites and rule out infection of the ascites. A diagnosis of SBP is made when there is an elevated ascitic fluid absolute neutrophil count (>250 cells/mm3) without an evident intra-abdominal or surgically treatable cause for the infection. SBP most often occurs with portal hypertensive ascites. The most sensitive marker available for such ascites is a serum/ascites albumin gradient >1.1 g/dL. Abdominal CT scan (choice A) is only indicated after an abdominal paracentesis is performed and SBP is excluded. Abdominal ultrasound (choice C) is indicated only if ascites is suspected but difficult to confirm by paracentesis or physical examination. Abdominal ultrasound can detect as little as 100 mL of ascitic fluid, whereas physical examination requires the presence of at least 1,000 mL of fluid to be a sensitive indicator for the presence of ascites. The abdominal ultrasound will identify ascites but is not able to identify infection in the ascetic fluid, which is why it is not the best choice for this scenario. Electrocardiogram (choice D) is not the most appropriate initial step given that there is no clinical or laboratory indication for it at present. Lumbar puncture (choice E) would be indicated if the patient had any change in his mental status or neurologic examination coupled with laboratory evidence that more common causes for infection have been excluded. Review All febrile patients admitted with presumed ascites must undergo abdominal paracentesis to determine the cause of the ascites and rule out infection. SBP is diagnosed with an absolute neutrophil count >250 cells/mm3. The most sensitive marker available for spontaneous bacterial peritonitis is a serum/ascites albumin gradient >1.1 g/dL.

A 79-year-old man with atrial fibrillation develops an acute abdomen. When seen 2 days after the onset of the abdominal pain, he has a silent abdomen, with diffuse tenderness and mild rebound. There is a trace of blood on the rectal examination. He also has acidosis and looks quite sick. Radiographs show distended small bowel and distended right colon, up to the middle of the transverse colon. Which of the following is the most likely diagnosis? A. Acute pancreatitis B. Mesenteric ischemia C. Midgut volvulus D. Perforated viscus E. Primary peritonitis

B. The setting of an elderly patient who has atrial fibrillation (or a recent myocardial infarction) who develops an acute abdomen strongly suggests embolus to the mesenteric vessels. The combination of abdominal pain and a trace of blood in the lumen is also typical, as is the radiograph outlining the territory supplied by the superior mesenteric artery. The patient probably has a dead bowel by now, as evidenced by his acidosis and severe illness. Any one of the other options could exist, but none are the most likely. Acute pancreatitis (choice A) would be expected in an alcoholic patient or a patient who has biliary tract disease. There would have been no blood in the lumen. Pain with acute pancreatitis is typically abdominal pain that radiates straight through to the back. Midgut volvulus (choice C) would be far more likely to happen to an infant who has malrotation. A perforated viscus (choice D) is indeed possible, but the radiographs would have shown free air rather than distended bowel. Primary peritonitis (choice E) would have been a consideration in a patient who has pre-existing ascites. Review Ischemic bowel should be suspected in an elderly patient who has atrial fibrillation. Embolic phenomena to the superior mesenteric artery may occur in the setting of atrial fibrillation. Cardiovascular disease with atherosclerosis is also a major risk factor for ischemic bowel disease since blood flow to the bowel may not be sufficient, causing chronic ischemic symptoms often manifested by abdominal angina. The underlying cause of the ischemic bowel must be corrected (embolectomy or revascularization) to save the bowel. Gangrene of the bowel secondary to ischemia has a high rate of mortality.

A 62-year-old man has had gastroesophageal reflux disease diagnosed by pH monitoring, which has been present for several years. He has been less than totally compliant with medical management, which he follows only when the pain is bad but discontinues when he feels better. Endoscopy and biopsies show severe peptic esophagitis, with Barrett's esophagus and early dysplastic changes, but no overt carcinoma. Additional tests show good esophageal motility, with low pressure in the lower esophageal sphincter and normal gastric emptying. Which of the following is the most appropriate treatment at this time? A. Heller myotomy of the lower esophageal sphincter B. Laparoscopic Nissen fundoplication C. Transhiatal total esophagectomy D. Transthoracic resection of the lower esophagus E. Vagotomy, pyloroplasty, and fundic gastric wrap

B. This man has indications for surgical intervention, but all he needs is an antireflux operation. By far the most commonly used procedure is a laparoscopic Nissen fundoplication. This procedure is performed laparoscopically. Heller myotomy (choice A) is used for achalasia. Here the procedure would do the exact opposite of what is needed. Transhiatal total esophagectomy (choice C) is the most commonly used operation for esophageal cancer. It is only palliative, because it cannot provide a true "cancer operation" with wide dissection, but esophageal cancer is rarely amenable to true curative resection when it becomes symptomatic. Although this man could develop cancer if he is not treated, he does not have advanced cancer at this time. Transthoracic resection of the lower esophagus (choice D) would be the procedure if a very early cancer were to develop at the esophagogastric junction. If this man did not choose to have the antireflux surgery now but he remained under close surveillance, he might get to the point at which he became a candidate for this option if his condition later progressed to cancer. Acid reduction via vagotomy (choice E) is not part of the standard surgical treatment for gastroesophageal reflux. We can control acid medically. If we have to operate, we do it to provide a good one-way valve when the native sphincter no longer works. Review GERD that does not respond to conventional therapy may need antireflux intervention. The procedure of choice is laparoscopically performed Nissen fundoplication in which a portion of the stomach is wrapped around the lower esophageal sphincter area to reinforce this area and prevent reflux. Preventing reflux will allow healing of the lower esophageal mucosa with the hope that any dysplasia will fail to progress.

A 29-year-old carpenter comes to the emergency department with his third episode of upper gastrointestinal bleeding in the past year. On both prior occasions he was found to be bleeding from a duodenal ulcer, and the bleeding was controlled endoscopically. He also reports having ulcers in the distal duodenum, jejunum, and additional ulcers twice in the past 4 years. He does not take any nonsteroidal anti-inflammatory drugs (NSAIDs) and has tested negative for Helicobacter pylori. Which of the following is the most likely cause of his recurrent symptoms? A. Autoimmune destruction of the parietal cells B. Autonomous production of gastrin C. Autonomous production of somatostatin D. Impaired production of the mucous bicarbonate layer in the stomach E. Inhibition of parietal cells secretion

B. This patient has a history of severe peptic ulcer disease, as manifested by multiple ulcers, ulcers in unusual sites such as jejunum, and ulcers complicated by bleeding. He is not taking nonsteroidal anti-inflammatory drugs (NSAIDs), and he does not have Helicobacter pylori colonization in the stomach. This suggests that he really does have very high acid secretion by the stomach, which should raise the possibility that he has a gastrin-secreting neuroendocrine tumor that is triggering the high level of acid release. The tumors are most likely to be in either the pancreas or the proximal small bowel. The combination of a gastrin-secreting tumor and severe peptic ulcer disease is sometimes called Zollinger-Ellison syndrome, and surgical resection of the tumor (which may be small and hard to find) usually permits cure of the peptic ulcer disease. Surgery is only possible if the gastrinoma is localized and so can be controlled by surgical resection. Autoimmune destruction of the parietal cells (choice A) causes the disease known as pernicious anemia, which is associated with atrophic gastritis. It is most commonly manifested by signs and symptoms related to vitamin B12 deficiency. Somatostatin (choice C) inhibits gastrin production and acid secretion, and so excess somatostatin would not be expected to cause peptic ulcer disease. Impaired production of the mucous bicarbonate layer in the stomach (choice D) is the mechanism whereby NSAIDs cause gastric and duodenal ulcers, but this patient is not on these medications. Inhibition of parietal cell secretion (choice E) would actually reduce ulcer tendencies. Review Patients who have gastrinoma can have multiple ulcers (especially in atypical locations) along with poor response to traditional acid-suppression therapies such as proton pump inhibitors and histamine 2 blockers. Elevated serum gastrin levels are diagnostic of this condition. Treatment is with high-dose acid inhibition or surgery to remove the gastrinoma if the mass is well localized.

A 28-year-old man with a 15-year history of ulcerative colitis and primary sclerosing cholangitis has recent worsening of his jaundice. His symptoms of ulcerative colitis have been in remission for the past year. He now complains of the onset of steatorrhea approximately 3 months ago and a 12-pound weight loss during that time. Which of the following would most likely account for this patient's recent symptoms? A. Bacterial overgrowth B. Bile salt deficiency C. Celiac sprue D. Tropical sprue E. Whipple disease

B. This patient has a history of ulcerative colitis complicated by primary sclerosing cholangitis. He has developed steatorrhea as the result of poor delivery of bile salts into the small intestine. This occurs because of intrahepatic and extrahepatic duct strictures of the biliary tree, which prevents adequate delivery of bile salts to the small bowel. In the absence of adequate bile salt, maldigestion of fats occurs, because micelle formation does not occur. Bacterial overgrowth (choice A) does not occur specifically in patients who have ulcerative colitis or primary sclerosing cholangitis. It typically results from an underlying anatomic or motility disturbance, such as a blind loop, hypomotility, a fistula, or a stricture. Celiac sprue (choice C) is not associated with ulcerative colitis or primary sclerosing cholangitis. Similarly, tropical sprue (choice D) is not associated with this patient's symptoms of colitis or primary sclerosing cholangitis. Furthermore, there is no travel history to suggest exposure. Whipple disease (choice E) is a multi-systemic disease that causes characteristic small bowel biopsy findings and results in malabsorption. It is also associated with hyperpigmentation, lymphadenopathy, cardiac disease, and rheumatologic disturbances. Review Patients with inflammatory bowel disease may develop a complication known as primary sclerosing cholangitis. This leads to cholestatic liver disease, causing continued destruction of the bile ducts, cirrhosis, end-stage liver disease, and even cholangiocarcinoma. When patients develop symptoms with this condition, they will have abdominal pain (right upper quadrant or epigastric), pruritus, fatigue, weight loss, fever, or jaundice. As the disease progresses, the patient may develop steatorrhea, which may be caused by the decrease in bile acid concentrations in the small bowel. That may, in turn, lead to fat malabsorption.

A 26-year-old woman comes to the health care provider complaining of epigastric pain and chest pain for 3 weeks. She describes the pain as burning, worse with eating, and nonradiating. She denies shortness of breath, nausea, vomiting, or palpitations. Review of systems is significant for a dry nocturnal cough. She smokes approximately 10 cigarettes each day and has two to three beers each night. She denies any drug use. Physical examination reveals a moderately obese woman in no acute distress. Her heart is regular without murmurs, rubs, or gallops. Her abdomen is soft, nontender, and nondistended with normal bowel sounds. Which of the following is the most appropriate management at this time? A. H2 blockers B. Lifestyle modification C. Metoclopramide (Reglan) D. Nissen fundoplication E. Proton pump inhibitor

B. This patient has gastroesophageal reflux disease. Epigastric or chest burning is the most common presenting symptom. Nocturnal cough, hoarseness, and exacerbation of asthma symptoms are also common. Predisposing factors include factors that decrease lower esophageal sphincter tone such as alcohol, smoking, fatty foods, peppermint, chocolate, pregnancy, and certain medications. Modification of risk factors can decrease symptoms and should be attempted before medications are started. Patients should be advised to eat small, non-fatty meals, avoid carbonated beverages, and remain upright after meals. Weight loss can be advised to lessen the intra-abdominal pressure. Patients should be counseled to decrease their alcohol consumption and to quit smoking, because nicotine has been shown to affect the lower esophageal sphincter tone. H2 blockers (choice A) together with proton pump inhibitors (choice E) are the first-line medical treatment for gastroesophageal reflux disease and should be initiated if lifestyle modification or antacids are not successful. These medications are well tolerated and have relatively few side effects. These drugs can heal esophagitis in many patients. Metoclopramide (Reglan) (choice C) is a prokinetic drug. Taken before meals, it can increase gastric emptying and increase lower esophageal sphincter pressures. It may be used in the management of gastroparesis, but it does not have a role in the management of GERD. Nissen fundoplication (choice D) is the surgical procedure used for patients who have refractory gastroesophageal reflux disease symptoms. Normal esophageal function is required for this procedure, to avoid postoperative complications. Review The initial therapy for GERD is lifestyle modifications with or without antacid therapy. Lifestyle modifications include raising the head of the bed, remaining upright after meals, avoiding spicy foods, chocolate, mints, carbonated beverages, garlic, and onions, and eating small meals. Weight loss may be helpful as part of this strategy as it decreases intra-abdominal pressure. Patients who have GERD should also avoid smoking and alcohol. If the patient has treatment failure, proton pump inhibitors in standard doses may be used. If this treatment fails, the patient could undergo Nissen fundoplication to reinforce the lower esophageal sphincter. Long-term effectiveness for this procedure is unclear, especially in the setting of a patient not having a response to proton pump inhibitor therapy.

A 72-year-old woman comes to the emergency department complaining of severe left lower quadrant pain for 24 hours. She has a long history of constipation and frequently has left lower quadrant cramping after meals. Over the past 24 hours, she has had increasing discomfort in the left lower quadrant and has had a temperature to 39.4ºC (102.8ºF). On examination there is tenderness to palpation and guarding in the left lower quadrant. CT scan reveals an abscess involving and contiguous to the sigmoid colon. Which of the following is most likely causing this patient's condition? A. Bleeding at the site of the abscess B. Micro- or macroperforation of a diverticulum C. Mucosal inflammation D. Mucosal ischemia E. Pancreatitis

B. This question illustrates the classic presentation of acute diverticulitis. It is important to distinguish diverticulosis, the presence of diverticula in the colon, and acute diverticulitis, which implies acute inflammation of a diverticulum. Diverticulosis is primarily a disease of the elderly and of individuals who consume a low fiber diet and it results in the development of hundreds of diverticula. Most of the diverticula tend to develop in the very distal colon (where the stool tends to be the most dehydrated and hardest to move by peristalsis), although in severe cases even the ascending colon may be involved. This predilection for distal colonic involvement tends to clinically produce chronic constipation and left lower cramping after meals, as seen in this patient. Acute diverticulitis should be suspected in cases like this one. Most cases of acute diverticulitis occur as the result of a micro- or macroperforation of a diverticulum, which allows gut bacteria to escape from the bowel lumen. In this case, a transmural perforation has created an abscess in the region of the sigmoid colon. Patients who have acute diverticulitis do not typically develop bleeding from the inflamed area of bowel (choice A). Such bleeding can be seen in ulcerative colitis and other severe mucosal conditions. Diverticulitis is not a consequence of mucosal inflammation (choice C). Although mucosal ischemia (choice D) can affect this region of the colon, it typically presents with diarrheal bleeding and does not cause an abscess. There is no association between pancreatitis (choice E) and the symptoms described in this patient. Pancreatitis will usually present with upper abdominal pain that often radiates to the back. Review Diverticulosis results from overactivity of colonic smooth muscle, which results in herniation of mucosa and submucosa through the muscle layers of the colon. Diverticulosis typically is associated with multiple diverticuli. Low dietary fiber and high meat intake along with aging are risk factors for the development of these diverticuli, because these factors cause an increase in the intraluminal pressure within the colon. When diverticuli become inflamed secondary to infection, diverticulitis occurs. When symptoms occur with diverticular disease, constipation and left lower quadrant abdominal pain may occur. Treatment of acute diverticulitis includes bowel rest, hydration, antibiotics, and potentially surgery if conservative management is not effective.

A 7-year-old boy passes a large, bloody bowel movement. He is hemodynamically stable, with hemoglobin 14 g/dL (normal 13.8-17.0 g/dL). Nasogastric aspiration yields clear, greenish fluid. Physical examination, including anoscopy, is unremarkable. Which of the following is the most appropriate next diagnostic test? A. Celiac arteriogram B. Colonoscopy C. Radioactively labeled technetium scan D. Radioactively tagged red blood cell study E. Upper gastrointestinal endoscopy

C. In this age group, with no obvious anal pathology and negative gastric aspirate, the leading cause of gastrointestinal bleeding is Meckel diverticulum. The specific source is ulceration of the normal ileal mucosa by acid produced by gastric mucosa in the diverticulum. The technetium scan identifies that ectopic gastric mucosa. Arteriogram (choice A) as a diagnostic step is a very invasive study that is appropriate only for very large hemorrhage (>2 mL/min) with clear gastric aspirate. An arteriogram may also be indicated for therapy (embolization) in severe gastrointestinal bleeding. Colonoscopy (choice B) would not identify the Meckel diverticulum. Colonoscopy is often needed in the older patient who has lower gastrointestinal bleeding, in whom the source of the hemorrhage is likely to be polyps, cancer, diverticula, or angiodysplasia. A tagged red blood cell study (choice D) is often used as a prelude to an arteriogram in patients who have substantial lower gastrointestinal bleeding. A tagged red blood cell study is used to identify GI bleeding for which no source can be identified. For this test to be useful clinically, the patient needs to be stable and have brisk enough bleeding that can be identified via this test. Upper gastrointestinal endoscopy (choice E) would have been appropriate if the gastric aspirate had produced blood. Review Meckel diverticulum is a common cause of lower GI bleeding in the pediatric population. Patients will present with either painless melena or bright red blood per rectum. When Meckel diverticular bleeding occurs, a radionuclide imaging study will show an uptake caused by the technetium binding to ectopic gastric parietal cells. Another test to consider with suspected Meckel diverticulum is mesenteric angiography, which will show extravasation of the dye at the bleeding site in the terminal ileum.

A patient with acute hepatitis B comes to the office complaining of severe fatigue, low-grade fevers, and weight loss. He was diagnosed with hepatitis B two weeks earlier, when hepatitis B surface antigen was positive. Physical examination reveals jaundice with diffuse skin excoriations. Liver and spleen are both markedly enlarged and tender. Peripheral edema is present. Which of the following would be the worst prognostic sign? A. Albumin of 3.1 g/dL (normal 3.5-5.5 g/dL) B. Bilirubin of 9.4 mg/dL (normal <1.9 mg/dL) C. Prothrombin time of 19 (normal 11-15) seconds with an INR of 2.1 D. SGOT (AST) of 2 U/L (normal 7-40 U/L) E. SGOT (AST) of 1,200 U/L (normal 7-40 U/L)

C. Many liver function abnormalities are seen in patients with acute hepatitis B, but the signs of hepatic failure (including coagulopathy or encephalopathy) are the worst prognostic signs. Patients may require emergent liver transplantation, as more conservative measures may not be lifesaving. A minimally reduced albumin of 3.1 g/dL (choice A) is not an important prognostic factor, though it may indicate long-standing hepatic synthetic dysfunction if no other etiology of hypoalbuminemia is present. Elevated bilirubin (choice B) is not a poor prognostic sign. In fact, hepatitis B patients may become very deeply jaundiced, with bilirubin elevation as high as 30 mg/dL, and may still return to normal. Elevated bilirubin levels occur concurrently with the elevation in the liver transaminase levels. Low SGOT/AST (choice D) has no prognostic importance. Elevated SGOT/AST (choice E) may be seen with this condition but it is not a specific prognostic sign. Review Patients with acute hepatitis B will have marked abnormalities in their liver function tests. Because of the amount of inflammation that occurs with the acute hepatitis B infection, one would expect patients to have increased transaminase levels and bilirubin levels. High levels are an expected phenomenon and are related to the acute infection. As the liver fails, liver function is affected. The final sign that the liver is failing is in the area of making clotting factors. When the liver fails to make clotting factors, there is clear evidence that it has reached its end-stage of failure. Prolongation of the prothrombin time therefore gives evidence that the patient is no longer making clotting factors, which signifies end-stage disease and the worst prognosis for patients who have acute hepatitis.

A 15-year-old girl comes to the emergency department because of the sudden onset of watery diarrhea tinged with blood. The girl was previously healthy. Her only medications are topical benzoyl peroxide and oral clindamycin for acne vulgaris. Physical examination reveals a slightly distended abdomen that is diffusely tender. Her temperature is 38.1ºC (100.5ºF). She has not been exposed to any uncooked meat or eaten unusual foods. Which of the following is the most likely diagnosis? A. Gastroenteritis B. Irritable bowel syndrome C. Pseudomembranous enterocolitis D. Salmonella infection E. Ulcerative colitis

C. Pseudomembranous enterocolitis is caused by the toxins produced by Clostridium difficile. It occurs in some patients after treatment with antibiotics (especially clindamycin, cephalosporins, and amoxicillin). Patients develop fever and abdominal pain with diarrhea containing leukocytes and blood. Gastroenteritis (choice A) is an acute condition, usually caused by a virus which presents with vomiting and diarrhea. There is typically no blood in diarrhea associated with gastroenteritis. Irritable bowel syndrome (choice B) is a chronic condition that produces episodic diarrhea with periods of constipation. There often is blood in the diarrhea but it is not associated with a fever. Salmonella infection (choice D) is a common cause of food poisoning. Findings include nausea, vomiting, abdominal pain, and diarrhea, often with blood. Ulcerative colitis (choice E) is a chronic condition that has features of bloody diarrhea, abdominal pain, and weight loss. Review C. difficile colitis is most likely to occur as a complication of antibiotic use. Although any antibiotic use puts a patient at risk for acquiring this condition, clindamycin is infamous for causing this condition. The other main risk factor for C. difficile colitis is recent hospitalization. Patients who have this condition present with recurrent episodes of foul-smelling diarrhea and abdominal pain. There may also be associated fever and abdominal tenderness.

A 56-year-old man has been having bloody bowel movements on and off for the past several weeks. He reports that the blood is bright red, coats the outside of the stools, and appears in the toilet bowl even before he wipes himself. There is also blood on the toilet paper after he wipes. After further questioning it is ascertained that he has been constipated for the past 2 months and that the caliber of the stools has changed. They are now very thin compared to the usual diameter of an inch or so that was customary for him. He has some minor discomfort. Which of the following is the most likely diagnosis? A. Anal fissure B. Cancer of the cecum C. Cancer of the rectum D. External hemorrhoids E. Internal hemorrhoids

C. The combination of red blood coating the stools and a change in bowel habit and stool caliber spells out cancer of the rectum in someone in this age group. Anal fissure (choice A) is typically seen in young women who have very painful bowel movements with streaks of blood. Pain is the dominant symptom in this condition. Although constipated people can develop anal fissures, this condition would not cause a change in the caliber of the stool. Cancer of the cecum (choice B) leads to anemia and occult blood in the stools, but the blood is rarely seen. If it is, the entire stool is bloody. Furthermore, there is no change in bowel habit or stool caliber when the tumor is so proximal in the colon. External hemorrhoids (choice D) hurt and itch, but they rarely bleed. They do not change the caliber of the stool. Internal hemorrhoids (choice E) do indeed bleed, but they do so without changing the pattern of bowel movements or the caliber of the stools. Review The following are the combination of symptoms in the presentation of rectal cancer: hematochezia (bright red blood in the stools) and/or enterorrhagia (intestinal bleeding), a change in bowel habit (constipation), and a change in stool caliber (pencil-like feces).

A 12-month-old boy is brought to the emergency department with bilious vomiting and abdominal distention for 10 hours. His mother states that the infant has been constipated since birth and failed to pass meconium during the first 48 hours of life. When specifically asked, she says that he often has brown, speckled vomitus. On examination he is very irritable. His length and weight are both below the fifth percentile for his age. The abdomen is moderately distended. After a digital rectal examination, a fair amount of stool ejects out from the anus. There is no stool in the ampulla. Which of the following is the most likely diagnosis? A. Duodenal atresia B. Intussusception C. Hirschsprung's disease D. Malrotation E. Pyloric stenosis

C. This infant has Hirschsprung's disease, or congenital aganglionic bowel disease. It is 5 times more common in boys than in girls. It results from congenital absence of ganglion cells in either part of or the entire wall of the colon, resulting in a state of chronic contraction. In most cases, the aganglionic segment is limited to the rectosigmoid colon. In very rare cases, part of or the entire small bowel can be aganglionic as well. Bilious or feculent vomiting, abdominal distention, and constipation are the classic clinical signs. There might also be a history of failure to pass meconium in the first 48 hours of life. If only a short segment of the colon is involved, Hirschsprung's disease might not be evident until childhood or adolescence. Megacolon proximal to the aganglionic segment might be visible on barium enema. The diagnosis is confirmed with the demonstration of an aganglionic segment of the bowel on punch biopsy. Duodenal atresia (choice A) usually presents with vomiting. A "double bubble" sign is seen on abdominal radiography. Thirty to forty percent of cases are associated with Down syndrome. Intussusception (choice B) is certainly in the differential diagnosis of vomiting and abdominal distention. In this case, however, the history of failure to pass meconium in the newborn period and failure to thrive is much more suggestive of Hirschsprung's disease. Intussusception occurs when one segment of the bowel telescopes into another segment just distal to it. The most common site of intussusception is the ileocolic junction. Malrotation (choice D) is usually caused by the presence of a volvulus, which presents with sudden onset of bilious vomiting, abdominal distention, rectal hemorrhage, peritonitis, and shock. It is a surgical emergency. Pyloric stenosis (choice E) usually presents with projectile vomiting in the first 2 or 3 weeks of life. On examination, an olive-shaped mass is usually palpable in the epigastric area. It is caused by hypertrophy and hyperplasia of the antrum of the stomach, resulting in obstruction. It occurs in 1 of 150 boys and in 1 of 750 girls. Review Hirschsprung's disease: Neurocrest migration defect that leads to absence of ganglion cells in segments of the colon or the entire colon Presents with symptoms of distal intestinal obstruction in the neonatal period Diagnosis is suspected when there is failure to pass the first meconium. Patients who have less severe cases can be diagnosed later in childhood and have a history of chronic constipation.

A 44-year-old obese woman comes to the emergency department complaining of 3 hours of severe abdominal pain. She has also had multiple episodes of vomiting during this time. She describes the pain as "worse than labor," and it radiates to the interscapular region. Her temperature is 38.9ºC (102ºF), and there is severe tenderness in the right upper quadrant. She reports that she has had multiple similar episodes in the past which have lasted approximately 30 minutes and then resolved spontaneously. Which of the following is most likely being obstructed by a gallstone? A. Common bile duct B. Common hepatic duct C. Cystic duct D. Pancreatic duct E. Right hepatic duct

C. This patient has acute cholecystitis. Risk factors include female gender, obesity, and a classic history of prolonged biliary colic in association with fevers. The presentation illustrated here is typical and results from obstruction of the cystic duct, which drains the gallbladder. Obstruction of the common bile duct (choice A) or the pancreatic duct (choice D) will produce acute ascending bacterial cholangitis, which would be demonstrated by Charcot triad (i.e., right upper quadrant pain, fever, and jaundice). Obstruction of either the common hepatic duct (choice B) or the right hepatic duct (choice E) may give a limited episode of cholangitis but will not cause cholecystitis, because the obstruction occurs in the biliary tree above the level of the entry of the cystic duct. Review Cystic duct obstruction is the most common cause of acute cholecystitis. The site of gallstone impaction is either at the neck of the gallbladder or in the cystic duct, which leads to inflammation in the gallbladder wall. Ultrasound of the gallbladder can diagnose this condition. Surgery in the form of laparoscopic cholecystectomy is curative.

A 44-year-old woman comes to the clinic after being treated in the emergency department 2 weeks ago for her first episode of diverticulitis. Her condition was diagnosed by CT scan of the abdomen and treated with an outpatient course of ciprofloxacin (Cipro) and metronidazole (Flagyl). She has no previous medical history and takes no medication on a regular basis. The patient's recovery was uneventful, and she is currently without any new complaints. She is now concerned about multiple small gallstones that were incidentally noted on the CT scan. Of note, the gallbladder appeared otherwise normal, and there was an absence of dilated ducts. She denies any problems with cholelithiasis in the past but is worried because her mother had a "gallstone infection" in the past that required surgery and intravenous antibiotics. Her temperature is 37.0ºC (98.6ºF), blood pressure 140/80 mm Hg, pulse 93/min, and respirations 18/min. She is 60 inches tall and weighs 77 kg (169 lb). Her abdomen is nontender and nondistended with normal bowel sounds. There is no rebound or guarding. Murphy sign is negative. Her liver is normal sized. Laboratory studies show normal levels for all of the following: Hemoglobin 11 mg/dL Leukocyte count 5700/mm3 Platelets 270,000/mm3 Hematocrit 31% Sodium 145 mEq/L Potassium 4.5 mEq/L Chloride 100 mEq/L Bicarbonate 24 mEq/L Glucose 116 mg/dL Creatinine 1.0 mg/dL BUN 10 mg/dL Total protein 6.0 g/dL Albuimin 3.5 g/dL Total bilirubin 0.9 mg/dL Direct bilirubin 0.4 mg/dL Alkaline phosphatase 59 U/L AST 20 U/L ALT 31 U/L Which of the following is the most appropriate management of this patient? A. Elective laparoscopic cholecystectomy B. Endoscopic sphincterotomy C. Exercise and diet modification D. Right upper quadrant ultrasound E. Treatment with ursodeoxycholic acid

C. This patient has asymptomatic gallstones that were incidentally found. Gallstones are frequently asymptomatic and when found do not require any further intervention or workup. There is some evidence that a low-carbohydrate diet together with increased physical activity may help prevent gallstone formation. In our patient, lifestyle modification would be useful mainly to prevent the sequela of obesity. Rapid weight loss should be discouraged in patients who have gallstones, because it may actually increase the risk for symptomatic gallstone formation caused by the biliary sludge that might occur in the setting of a patient eating fewer fat calories. Laparoscopic cholecystectomy (choice A) is the treatment of choice in patients who have symptomatic gallstones, porcelain gallbladder, chronic cholecystitis, and acute cholecystitis or cholangitis. It is also indicated in patients who have asymptomatic gallstones and sickle cell disease or for people who are traveling to remote areas of the world who lack appropriate medical care if their condition deteriorates. Because our patient has no symptoms from her incidentally discovered gallstones, treatment is unnecessary. Endoscopic sphincterotomy (choice B) is the recommended treatment for choledocholithiasis. In this condition symptoms can be variable, but imaging reveals dilated gallbladder ducts. Antibiotics are necessary in this condition, because patients may progress to ascending cholangitis. Right upper quadrant ultrasound (choice D) is the first test ordered in the workup of gallbladder disease, but our patient has already been adequately imaged with her CT scan. As long as she remains asymptomatic, no further imaging is necessary. Treatment with ursodeoxycholic acid (choice E) is reserved for patients who refuse surgery or who are at too high a risk for surgery. This medication might help dissolve gallstones in those patients when taken for prolonged periods. Gallstones tend to recur whenever this medication is stopped. This is not a suitable alternative for surgery in patients who have symptomatic disease. Again, our patient does not need any intervention at this time. Review Asymptomatic gallstones discovered incidentally in an otherwise healthy patient require no intervention. Weight loss is indicated to prevent complications associated with obesity. An exception is those who have asymptomatic gallstones and coexisting sickle cell disease who will require elective cholecystectomy. Patients with sickle cell disease tend to have chronic hemolysis, which leads to the formation of gallstones. Because this is a lifetime disease, those who have sickle cell disease (not just the trait) are encouraged to undergo elective cholecystectomy whenever gallstones are identified.

An 8-year-old girl is brought to the office because she has been complaining of frequent abdominal pain. The episodes occur every several days, show no particular pattern, and resolve without treatment within 10-15 minutes. During the episodes, the child has to lie down and bring her legs up to her chest for relief. The parents state that she started having this problem approximately a year ago, and they believed it was related to her diet. After trying to restrict various kinds of food with no effect, they decided to have her examined. Her bowel movements are regular. She has no significant past medical history and takes no medication on a regular basis. She has an older brother who is in excellent health. The mother tells you that she loves school and is a very good student. She is very popular among her friends and has no problems in social relationships. Of interest, they immigrated to the United States 3 years earlier and have moved 3 times since then for work. She did not seem to have any trouble adjusting to that situation. They will be moving to another city again in 6 months. On physical examination the girl is in no acute distress. She is in the 90th percentile for height and weight. Vital signs are within normal limits, as is a complete blood count with white blood cell differential. Her abdomen is not distended and is soft and nontender to palpation. Plain abdominal films show no abnormalities. Which of the following patient intervention/patient education scenarios should be pursued? A. A computerized tomography scan of the abdomen is indicated B. A trial of oral antibiotics is warranted C. She is likely to continue having episodes of abdominal pain even as an adult D. She may have Hirschsprung's disease and a biopsy of the colon is indicated E. A barium enema is indicated at this time

C. This patient has functional recurrent abdominal pain and it is highly likely that she will continue to have episodes of abdominal pain even as an adult. Patients who have functional recurrent abdominal pain present with nonspecific symptoms. It is difficult to obtain a pattern of occurrence of the pain. Stressors such as school or examinations may produce a predictable pattern. Social factors, such as relocating, family illness, and sibling rivalry may account for abdominal pain. Medically unexplained headache or abdominal pain occurs at least once a week in 10 to 30% of children and adolescents in the United States. Seventy percent of children will continue to experience abdominal pain into adulthood, although the symptoms may not impair their physical activity to the same degree. Up to 30% will develop additional complaints, such as headaches. Parents may have suffered from abdominal pain. Treatment may include counseling, relaxation training, behavioral methods, biofeedback, and family therapy, together with treatment of possible comorbid conditions, such as anxiety or depression. After a thorough history and physical examination, most patients who have recurrent abdominal pain do not require further evaluation. Useful studies, however, include complete blood count, sedimentation rate, urine analysis, stool studies, ultrasound, radiographic studies, breath hydrogen, and endoscopy. Patients who have functional recurrent abdominal pain do not require imaging studies beyond a plain abdominal radiograph, unless a specific medical condition is suspected. It is therefore incorrect that a computerized tomography scan of the abdomen is indicated (choice A). Treatment of functional recurrent abdominal pain involves counseling, relaxation training, behavioral methods, biofeedback, and family therapy. There is no indication for a trial of oral antibiotics (choice B) in this patient, as she is not exhibiting any signs of an infectious process or an inflammatory condition that would benefit from that. Hirschsprung's disease most commonly presents at birth as delayed passage of meconium for 36 hours or more and frequent episodes of vomiting in the neonate. There is abdominal distention, normal anal sphincter tonus, and an empty rectal vault. An abdominal radiograph with barium contrast shows the megacolon and a transition zone. Biopsy of the colon confirms the diagnosis. Most cases of Hirschsprung's disease are diagnosed in infancy. It is therefore highly unlikely that this patient may have Hirschsprung's disease and that a biopsy of the colon is indicated (choice D). A barium enema (choice E) is not indicated in this patient, as her history and findings are more consistent with functional recurrent abdominal pain than with intussusception. Review Functional abdominal pain may be acute, chronic, or cyclic. Girls between ages 8-12 are likely to have this condition. Presentation is with vague, persistent, central abdominal pain, and this pain may be associated with nausea and vomiting, especially when this is a chronic complaint. There may be a familial pattern with others in the family having irritable bowel, anxiety, fibromyalgia, migraine, or other psychiatric conditions. Examination of the abdomen may reveal periumbilical tenderness, but the abdomen is otherwise soft, undistended, and without rebound or guarding. The remaining examination is normal. This diagnosis is clinical after elimination of organic causes. Treatment involves lifestyle changes, which are intended to lessen stress. There may be a role for probiotics in improving this condition.

Four weeks after a camping trip, a 16-year-old boy begins to pass foul-smelling stools. He also develops anorexia and flatulence. None of his friends from the trip are ill. He is generally healthy and takes no medications. His temperature is 37.3ºC (99.1ºF). Physical examination shows diffuse abdominal pain and distention and guaiac-negative stool. The remainder of the examination is unremarkable. A microscopic image of his stool is shown. Which of the following is the most correct statement about his condition? A. Asymptomatic infection is rare B. Boiling of water during the camping trip would not have killed the infective organism C. A 5- to 7-day course of metronidazole (Flagyl) has a cure rate of 80 to 95% D. Infection is limited to the large intestine E. The trophozoite is the infective form of this organism

C. This patient has giardiasis, which is caused by Giardia lamblia. G. lamblia is a flagellated protozoan that exists in a cyst and trophozoite form. The cyst, not the trophozoite, is the infective form (choice E). Infection is limited to the biliary tract and small intestine, not the large intestine (choice D). Contaminated water supplies are the common source of infection, and boiling water can kill the infective cysts (choice B). Many individuals infected with Giardia remain asymptomatic (choice A). A 5- to 7-day course of metronidazole has a cure rate of 80 to 95%. Review Giardiasis is an enteric infection caused by a flagellated protozoan parasite, Giardia lamblia. It is one of the most commonly identified intestinal parasites in the United States, though it is more common in the developing countries. Campers and those who drink fresh water sources are most likely to acquire this disease. Infection requires the oral ingestion of cysts, with as few as 10 cysts resulting in disease. Incubation is from 3-25 days. After the cysts are ingested, trophozoites are released into the small bowel. These trophozoites are converted to the infectious cysts in the large intestine. First-line therapy is metronidazole (Flagyl) or tinidazole (Tindamax), which has the advantage of a single 2-g curative dose. If treatment fails, a different class of medication should be tried, such as albendazole (Albenza). Parasites are classically cleared from the stool in 3-5 days and symptoms clear in 5-7 days. Health care providers need to stress the necessity of good hand washing after toileting or before preparing food to prevent spread of this disease. Patients need to avoid drinking untreated water, and any raw food that is going to be consumed should be washed thoroughly with uncontaminated water. When traveling to endemic areas, raw food should be avoided and all water boiled for at least 1 minute before consuming.

A 44-year-old school bus driver comes to the emergency department complaining of severe abdominal pain. She reports that the pain began approximately 8 hours ago, after eating lunch at a fast-food restaurant. The pain has become increasingly severe and radiates to her back. She recalls a similar episode, lasting 3 hours, 2 months earlier and another episode, lasting 12 hours, during her last pregnancy. She is afebrile, and without jaundice, and has right upper quadrant tenderness with deep palpation of this area. A rectal examination reveals brown stool that is negative for occult blood. Her leukocyte count is 12,900/mm3, (normal 5,000-10,000) and her hematocrit is 39% (normal 41-50%). Her total bilirubin is 2.1 mg/dL (normal <1.9). An ultrasound is performed at the bedside revealing a positive ultrasound Murphy sign but there were no stones or sludge noted in the gallbladder and the cystic duct is not dilated. Which of the following is the most appropriate next step in her diagnostic evaluation? A. CT scan of the abdomen and pelvis B. Endoscopic retrograde cholangiopancreatography (ERCP) C. HIDA scan D. Percutaneous transhepatic cholangiogram (PTC) E. Upper gastrointestinal barium study

C. This patient has the classic presentation of acute cholecystitis. The episodes she had several months ago and during pregnancy suggest a prior history of biliary colic. A HIDA scan is a noninvasive nuclear medicine test that will reveal obstruction of the cystic duct, which is caused by an impacted gallstone and is the cause of acute cholecystitis. The patient's elevated total bilirubin provides evidence that the bile duct is blocked. The HIDA scan can also be used to identify gallbladder dysfunction for patients whose gallbladders do not function but who may not have stones. A CT scan (choice A) may show a distended gallbladder, but it is not as accurate as a HIDA scan for evaluating the cystic duct. Endoscopic retrograde cholangiopancreatography (ERCP) (choice B) is useful for evaluation of the common bile duct but is of a less value in evaluation of the cystic duct and, furthermore, is a far more invasive test than a HIDA scan. Percutaneous transhepatic cholangiography (PTC) (choice D) is an examination that is performed by the interventional radiologist by injecting the intrahepatic biliary tree percutaneously. This is rarely done since ERCP is a more accurate evaluation of the biliary of tree. PTC is of limited value in evaluating the cystic duct and it is an invasive procedure. An upper gastrointestinal barium study (choice E) may be useful for the evaluations of peptic ulcer disease but the symptoms here are far more suggestive of acute cholecystitis. Furthermore, if the patient has been vomiting she is unlikely to tolerate this examination. Review Acute cholecystitis is typically caused by an impacted stone in the gallbladder neck or cystic duct. This impacted stone leads to inflammation within the gallbladder wall. Ultrasound is the initial test of choice for evaluation of suspected cholecystitis, with HIDA scan or MRI scan being done when the diagnosis is in question or when gallbladder function is being assessed.

A 56-year-old man presents with complaints of gnawing pain in the mid-epigastrium, with occasional radiation to the back. He also notes a 15-pound weight loss over the past 3 months. The clinician suspects pancreatic carcinoma. Which of the following tumor markers would aid in confirming this diagnosis? A. Alpha-fetoprotein (AFP) B. CA-125 C. Cancer Antigen (CA 19-9) D. Human chorionic gonadotropin (hCG) E. Prostate-specific antigen (PSA)

C. Tumor markers can be very helpful in narrowing the possible primary sources for metastatic lesions. CA 19-9 is present in about 80% of patients who have pancreatic cancer. Tumor markers should not be used as the primary tool for cancer screening, but they have considerable utility in the confirmation of the diagnosis, as well as for monitoring recurrence or response to therapy. Alpha-fetoprotein (AFP; choice A) is seen in hepatocellular carcinoma, embryonal cell tumor of the testis, and malignant teratoma. CA-125 (choice B) is produced by ovarian cancer. The beta subunit of human chorionic gonadotropin (hCG; choice D) is seen in choriocarcinoma, hydatidiform mole, and germinoma. Prostate-specific antigen (PSA; choice E) is seen in prostatic carcinoma. Reviews CA 19-9 is used preoperatively in pancreatic cancer as a predictor for invasive disease and postoperatively after tumor resection to assess that the tumor was completely eradicated and to monitor the patient postoperatively to assess for recurrence of the tumor. Because of its inadequate sensitivity and specificity, it is not used as a general screen for pancreatic cancer in asymptomatic patients.

A 48-year-old woman with a long history of hepatitis C is admitted for hematemesis. She is stabilized in the emergency department, and an upper endoscopy reveals bleeding esophageal varices. On examination, she is also noted to have a moderate amount of ascites. Which of the following is the most appropriate next step in the management of her bleeding? A. Alpha interferon B. Vasopressin C. Endoscopic variceal banding D. TIPS (transjugular intrahepatic portosystemic shunt) E. Liver transplant

C. When varices are seen, the immediate appropriate management is endoscopic banding of these varices, which will usually immediately lead to cessation of bleeding. Esophageal sclerotherapy is also effective but is associated with a higher instance of post-procedure complications, such as esophageal ulceration, stricture, and bacteremia. Preoperative interferon (choice A) is not effective at all in the management of variceal bleeding. It is a component of therapy for hepatitis B and hepatitis C. Vasopressin (choice B) is a vasoconstrictor that reduces portal pressures, but it is not the first line of therapy in control of variceal bleeding. TIPS (transjugular intrahepatic portosystemic shunt) (choice D) is used as an intrahepatic shunt to divert blood flow from the portal venous system to the systemic venous system. It may be used to decrease esophageal venous pressure, but it is not used acutely in the management of bleeding esophageal varices. Liver transplantation (choice E) is used only in patients who have variceal bleeding that cannot be controlled by any other means. This procedure is not readily available in emergency situations and there is a high mortality associated with bleeding esophageal varices. Review Esophageal varices develop as a direct consequence of portal hypertension. Portal hypertension can occur as a result of cirrhosis of the liver, which may be caused by chronic alcohol use or infection such as hepatitis B or C. Acute esophageal bleeding can be managed with resuscitation and endoscopic ligation. Options for therapy include octreotide (Sandostatin) and vasopressin. Intrahepatic portocaval shunts may also be used to lower portal pressures or can be used in the setting of failed endoscopic band ligation. Balloon tamponade may provide adjunctive therapy to the intrahepatic portocaval shunt to control the bleeding until the shunt is able to drop the portal pressures.

A 57-year-old woman with a 30-year history of alcoholism and liver disease visits her health care provider complaining of abdominal swelling and shortness of breath. The health care provider determines that she has severe ascites. Which of the following factors contributes to the accumulation of fluid in the abdominal cavity? A. Decreased plasma epinephrine and norepinephrine B. Decreased plasma volume C. Increased hepatic lymph flow D. Increased hydrostatic pressure in splanchnic capillary beds E. Increased natriuresis

D. Ascites often occurs in patients who have cirrhosis and other forms of severe liver disease and is usually noticed by the patient because of abdominal swelling. Shortness of breath may occur because the diaphragm is elevated when the accumulation of fluid becomes more pronounced. A number of factors contribute to accumulation of fluid in the abdominal cavity. Portal hypertension plays an important role in the production of ascites by raising capillary hydrostatic pressure within the splanchnic bed. Elevated, not decreased serum levels of epinephrine and norepinephrine (choice A) occur as a result of increased central sympathetic outflow, in patients who have cirrhosis and ascites. The increased sympathetic output leads to decreased, rather than increased natriuresis (choice E) by activation of the renin-angiotensin system and diminished sensitivity to atrial natriuretic peptide. Interstitial fluid often weeps freely from the surface of the cirrhotic liver because of distortion and obstruction of hepatic lymphatics with resultant decreased rather than increased hepatic flow (choice C). This interstitial fluid has a high protein concentration because the endothelial lining of the hepatic sinusoids is discontinuous. The entry of protein-rich interstitial fluid into the peritoneal cavity may account for the high protein concentration present in the ascitic fluid of some patients. The high protein content of the interstitial fluid results in fluid retention and an increase, not a decrease in plasma volume (choice B). Review Ascites is the accumulation of fluid in the peritoneal cavity; the causes are usually related to cirrhosis. With cirrhosis the liver undergoes loss of functional cells, causing increased portal pressures and increased pressures in the gastric and splenic venous systems. Other causes of ascites include hypoalbuminemia (generalized swelling caused by poor oncotic pressure in the cell with resultant cellular leakage) and neoplasms such as ovarian cancer with malignant accumulation of fluid in the peritoneal cavity.

A previously healthy 7-year-old girl comes to the office with complaints of episodic abdominal pain over the past several months. The pain is periumbilical and sharp but does not wake her from sleep and rarely interferes with play. She has no fever, joint complaints, or constipation or diarrhea. Growth and development have been normal. Physical examination is within normal limits. Which of the following is the most likely diagnosis? A. Acute appendicitis B. Acute cholecystitis C. Crohn's disease D. Functional abdominal pain E. Irritable bowel syndrome

D. Functional abdominal pain is pain that lasts for more than 3 months and rarely interferes with normal activity. The pain is periumbilical and often hard to describe. The pain typically does not awaken patients from sleep or interfere with pleasant activities. The pain is real and is the result of the regulation of gastrointestinal motility in response to either psychologic or physical stress. Acute appendicitis (choice A) usually occurs with right lower quadrant pain, fever, and anorexia. The chronic nature described in the question would rule out acute appendicitis. Acute cholecystitis (choice B) presents with right upper quadrant pain and vomiting, and again the history of several months of symptoms would rule out this diagnosis. Crohn's disease (choice C) would usually present with abdominal pain, diarrhea (usually loose with blood), and anorexia. Although the abdominal pain lasts several months, as in this case, it usually causes weight loss and delayed growth. Patients who have this condition may also have weight loss, aphthous ulcers, or perianal problems such as fistulas. Irritable bowel syndrome (choice E) would cause abdominal pain that could last several months. This condition would also typically cause bouts of diarrhea alternating with constipation. Patients who have functional abdominal pain during childhood may have an increased risk for the development of irritable bowel syndrome. Review Functional abdominal pain is seen most frequently in girls 8-12 years of age, and patients who have this disorder typically complain of vague, persistent, central abdominal pain that can be associated with nausea and/or vomiting. Examination will reveal periumbilical tenderness with a soft abdomen and no rebound or guarding. The remainder of the physical examination is normal. This is a diagnosis of exclusion and no red flag symptoms are present (weight loss, failure to thrive, nocturnal diarrhea, signs of bleeding).

A previously healthy 11-year-old boy is brought to the health care provider with a fever and persistent vomiting for 4-5 days. Initially the emesis was clear, but now it contains streaks of bright red blood. Physical examination, complete blood count, and serum electrolytes are all normal. Which of the following is the most likely diagnosis? A. Esophageal varices B. Esophagitis C. Gastritis D. Mallory-Weiss tear E. Peptic ulcer disease

D. Mallory-Weiss tears occur in the lower esophagus after forceful or protracted vomiting and involve only the superficial mucosa in the lower esophagus. In most cases, bleeding is painless and resolves once the insulting incident (i.e., vomiting) resolves. Esophageal varices (choice A) cause bright red bleeding in vomit, which is the most common presenting symptom of portal hypertension. Other signs of portal hypertension (e.g., splenomegaly, hepatomegaly, ascites) should be present. Esophagitis (choice B) in children is most often associated with gastroesophageal reflux disease. The patient will usually have a complaint of chest and abdominal pain, with a history of frequent regurgitation. Gastritis (choice C) in children is usually associated with abdominal pain. It rarely is associated with a fever and sometimes causes vomiting with bright red blood. Gastritis occurs as a complication of a substance irritating the gastric mucosa, such as alcohol, NSAIDs, or corticosteroids. Peptic ulcer disease (choice E) has symptoms of mainly nocturnal abdominal pain. Vomiting is present in only 10% of cases, and only 5% have hematemesis. Review Mallory-Weiss syndrome is a tear in the mucosa of the gastroesophageal junction. It occurs as a result of a sudden rise in abdominal pressure that results in the tear and subsequent GI bleeding. This syndrome occurs following forceful or long-term retching, vomiting, coughing, or straining. Treatment is supportive, because most cases are self-limited.

A 70-year-old man complains of chronic heartburn. It is painful for him to bend over, and he sleeps on a wedge-shaped pillow to try to reduce the burning sensation. Which of the following agents would be most efficacious in reducing his symptoms? A. Bisacodyl (Dulcolax) B. Cimetidine (Tagamet) C. Magnesium hydroxide (Milk of Magnesia) D. Omeprazole (Prilosec) E. Promethazine (Phenergan)

D. Omeprazole (Prilosec) is an irreversible inhibitor of H+/K+/ATPase of the parietal cell (also known as the proton pump). It is used in the treatment of peptic ulcer disease, reflux esophagitis, and Zollinger-Ellison syndrome. Omeprazole can reduce daily gastric acid secretion by more than 95%. This drug is of particular value to patients who do not respond sufficiently to H2 antagonists. Because of the profound reduction in gastric acid secretion, patients can develop modest hypergastrinemia. It is the most effective way to treat GERD burning, because it maximally suppresses gastric acid production. Bisacodyl (Dulcolax) (choice A) is a stimulant laxative. It does not have any effect on GERD symptoms. Cimetidine (Tagamet) (choice B) is an H2-receptor antagonist that would block histamine-induced acid secretion. It is not as efficacious as omeprazole, as it does not suppress acid secretion as its mechanism of action. Magnesium hydroxide (Milk of Magnesia) (choice C) is an antacid and is also used as an osmotic laxative for relieving constipation. It would help relieve the heartburn but would not be as efficacious as omeprazole. Promethazine (Phenergan) (choice E), a dopamine-receptor antagonist, is an antiemetic. Review The most effective class of medications for treating GERD-related symptoms is a proton pump inhibitor. If symptoms only partially resolve with these agents, the dosage can be increased to 2x per day, before breakfast and dinner. If symptoms still fail to resolve, further testing should be done to rule out potential for carcinoma.

An 82-year-old woman is brought to the emergency department from the local nursing home because of complaints of severe constipation. She has not moved her bowels in 5 days and today began complaining of lower abdominal discomfort and distention. She takes hydrochlorothiazide and acetaminophen with codeine for severe arthritic pain in both hips. Over the past week she has been bedridden because of the severe pain. She is afebrile. Abdominal examination shows mid- and lower-abdominal distention with mild tenderness. Bowel sounds are normoactive. Rectal examination demonstrates that the rectal vault is filled with hard stool. Which of the following is the most appropriate next step in management? A. Barium enema B. Colonoscopy C. Flexible sigmoidoscopy D. Manual disimpaction E. Passage of a nasogastric tube

D. This elderly woman, who has been bedridden and on a narcotic analgesic, has become severely obstipated. This multifactorial condition is caused by her immobilization, bowel hypomotility secondary to the codeine, and possible mild dehydration from her diuretic. Treatment is manual disimpaction, which will promptly "open the flood gates" and yield immediate relief for the patient. No other imaging studies, such as barium enema (choice A), colonoscopy (choice B), or flexible sigmoidoscopy (choice C) are necessary at present. The patient should be disimpacted to promptly resolve her abdominal distention and discomfort. Passage of a nasogastric (choice E) tube would be indicated if the patient showed signs of upper gastrointestinal bleeding, but it would not be appropriate for this woman who has obvious constipation. Review Immobilized patients who are simultaneously taking narcotics for pain relief are at high likelihood of developing constipation or obstipation. Manual disimpaction, performed by the health care provider, is a safe method of treatment and will provide immediate relief. Patients taking short-term narcotics may need laxatives in addition to stool softeners to prevent constipation.

A 37-year-old accountant comes to the health care provider to ask for advice regarding the future management of his ulcerative colitis. He has had pancolitis for the past 19 years and has been told that he is at an increased risk for developing colorectal cancer. He asks for the health care provider's recommendation regarding appropriate surveillance. Which of the following is the most appropriate response? A. Annual stool guaiac testing B. Barium enema C. Colonoscopy D. Colonoscopy and multiple biopsies E. Flexible sigmoidoscopy with multiple biopsies

D. Patients who have long-standing extensive ulcerative colitis for at least 10 years' duration are at increased colon cancer risk. Appropriate surveillance involves annual or biannual colonoscopy with multiple biopsies at regular intervals, even of normal appearing mucosa, to check for dysplasia. None of the other choices encourage sampling of the entire colonic mucosa for histologic examination for the precancerous lesion of low- or high-grade dysplasia. Review Patients who have inflammatory bowel disease (ulcerative colitis and Crohn's disease) are at increased risk for colon cancer. Inflammatory pseudopolyps develop as a result of mucosal ulceration and regeneration. These polyps are numerous in number and scattered throughout the colonic region of the colon. The risk for development of colorectal cancer increases with the duration of the disease along with the level of disease activity. The risk for development of colon cancer for those with ulcerative colitis is 5-10 times higher than in those without inflammatory bowel disease.

A patient undergoes a total gastrectomy because of a proximally located gastric cancer. After the surgery, which of the following digestive enzymes will be produced in inadequate amounts? A. Amylase B. Chymotrypsin C. Lipase D. Pepsin E. Trypsin

D. Pepsin is secreted by the chief cells of the stomach. Pepsinogen is activated by contact with stomach acid. Amylases (choice A) are produced by the pancreas and salivary glands. Chymotrypsin (choice B) is a proteolytic enzyme released by the pancreas as the inactive proenzyme, chymotrypsinogen. Lipases (choice C) are mostly released by the pancreas and serve to digest various lipids, including triacylglycerols. Trypsin (choice E) is a proteolytic enzyme released by the pancreas as the inactive proenzyme trypsinogen. Review Gastric cancer is treated with surgical resection when it is regionally confined. Total gastrectomy is recommended for proximal tumors. Patients who have distal gastric cancers are able to have similar outcomes with subtotal gastrectomy. Adjunctive perioperative chemoradiation or perioperative chemotherapy is done for later stage disease. The stomach secretes pepsin (chief cells), gastrin (G-cell stimulation), pepsinogen (G-cell stimulation), and hydrochloric acid as initial means for digestion of food substances.

A 23-year-old woman seeks help for exquisite pain with defecation and blood streaks on the outside of her stools, which she has been having for several weeks. Because of the pain, she has avoided having bowel movements, and when she finally did, the stools were hard and even more painful. When seen, she has no fever or leukocytosis. Physical examination has to be done under spinal anesthesia, because the patient was so afraid of the pain that she initially refused even inspection of the area. The examination confirms the suspected diagnosis, and she is placed on stool softeners and appropriate topical agents, but without success. She is now willing to undergo more aggressive treatment. Which of the following is the most appropriate next step? A. Excision of the lesion B. Fistulotomy C. Incision and drainage D. Lateral internal sphincterotomy E. Rubber band ligation

D. The clinical picture is classic for anal fissure, which is a split in the skin of the anatl canal. Patients with this condition present with pain with defecation along with rectal bleeding from the area of the split. It commonly occurs in young to middle age persons. Pain is described as severe and this can adversely affect quality of life. If spontaneous healing does not occur, surgical intervention with lateral sphincterotomy can be performed. Excision (choice A) used to be done for this condition, before the role of the "too tight sphincter" was identified; it is no longer first-line therapy for anal fissures. Fistulotomy (choice B) is not the answer. She has a fissure, not a fistula. Incision and drainage (choice C) is another option that addresses a wrong diagnosis. We do that for perirectal abscess, which produces severe pain with fever and leukocytosis, but without blood streaks, and drains spontaneously after several days if not diagnosed and treated. Rubber band ligation (choice E) is the treatment for internal hemorrhoids. Internal hemorrhoids can bleed but typically do not hurt. Thrombosed external hemorrhoids can hurt tremendously, but those are not amenable to rubber band ligation, for which surgical excision is recommended. Review Anal fissure patients will complain of severe pain with defecation and following defecation with the pain lasting up to several hours following the bowel movement. On physical examination of the anus, there may be marked spasm of the sphincter muscles seen or a split in the skin of the distal anal canal. Conservative therapy is with stool softeners, sitz baths, high fiber diet, adequate fluid intake, and topical anesthetics.

A 51-year-old man undergoes a barium enema as a colon cancer screening examination. A 3-mm polyp is found in the ascending colon. It is rounded and smooth and meets all the radiographic criteria of a benign sessile polyp. No other lesions are seen within the colon. Which of the following is the most appropriate management? A. Repeat the barium enema in 1 year B. Schedule a colonoscopy in 1 year C. Schedule a sigmoidoscopy now D. Schedule a colonoscopy now E. Schedule a CT scan to rule out nodal involvement

D. The finding of a sessile mass in the colon, even though it is small and has a benign appearance, mandates a colonoscopy for its removal. This would allow a complete excisional biopsy and histologic review. Although frank carcinoma is unlikely with a mass this small, either a tubular adenoma or a villous adenoma is conceivably present and should be removed both to establish the diagnosis and to prevent the lesion from eventually progressing to cancer. In addition, the structure of the bowel is sufficiently complex that it is not uncommon for other small polyps to be present that were not picked up radiologically. These should also be excised, and their histology reviewed. The need for another diagnostic test to be performed highlights the advantage of performing the colonoscopy as the initial screen rather than barium enema. It would be inappropriate to wait 1 year with a known polyp, even with a benign appearance, so follow-up barium enema (choice A) or colonoscopy (choice B) in 1 year would be inappropriate. A sigmoidoscopy (choice C) would be an inadequate examination to reach the ascending colon to remove this polyp. Extraluminal involvement with such a benign-appearing polyp is highly unlikely, so CT scan (choice E) is unnecessary. Review Colon cancer typically has a precursor lesion that begins as a colonic polyp. The majority of colorectal cancers arise in adenomas. All types of adenomas may be dysplastic and premalignant, with adenomas having a villous component being the most likely to become malignant. When polyps are identified, they should be removed to prevent them from becoming malignant. Colonoscopy is the procedure of choice to remove these polyps.

A 5-year-old boy is brought to the emergency department with 2-day history of fever, anorexia, loose stools, and yellow skin color. He attends a large daycare center. On physical examination his temperature is 38.1ºC (100.7ºF), blood pressure 88/56 mm Hg, pulse 74/min, and respirations 15/min. Initial laboratory evaluation reveals a total bilirubin of 1.8 mg/dL (normal <1.9 mg/dL) and alanine aminotransferase 764 U/L (normal <25 U/L in this age group). Nobody else in his family is sick. Which of the following is the most appropriate diagnostic test? A. Hepatitis B surface antigen in serum B. IgG for hepatitis A in serum C. IgG for hepatitis B surface antigen in serum D. IgM for hepatitis A in serum E. Stool culture for hepatitis A

D. This boy most likely has hepatitis A. While it is more common in developing countries, it is still a common infection in the developed world. It is transmitted by the fecal-oral route when contaminated food or water is ingested. The incubation period is about 15-40 days. Ninety percent of children acutely infected with hepatitis A are asymptomatic. The virus is shed in the feces 2-3 weeks before the onset of jaundice until approximately 1 week after onset. Most infected children are infectious for a long time before they are symptomatic. Hepatitis A is therefore very difficult to control. Large outbreaks frequently occur in daycare centers. Diagnosis is best made by determination of IgM levels against hepatitis A virus. The presence of IgM for hepatitis A suggests acute infection; this antibody peaks at 4-6 weeks and does not persist beyond 6 months. IgG is produced in the primary infection, but for most viral infections, including hepatitis A, it persists for a lifetime. The presence of IgG against hepatitis A (choice B) therefore could mean a previous infection and is not diagnostic of a current infection. Stool culture for hepatitis A (choice E) cannot be used to confirm the diagnosis. Stool HAV RNA detection can be performed, but it is very expensive and rarely used in routine settings. Prophylaxis with immunoglobulin is recommended for the household and close contacts of the infected person within the first 2 weeks of exposure. Strict hand washing is also very important. There is a killed-virus vaccine available that provides immunity to travelers to developing countries if given in two doses 6-12 months apart. The presence of hepatitis B surface antigen in the serum (choice A) signifies an active/current infection with hepatitis B. IgG against hepatitis B surface antigen (choice C) comes from an immunologic response to either the hepatitis B virus surface antigen from an infection, or from the hepatitis B virus vaccine. Review Risk factors for hepatitis A infection include travel outside the United States, contact with other children in day care, and poor sanitation. Hep A is acquired via a fecal-oral route of transmission. Determination of anti-hepatitis A virus IgM levels is the best confirmatory test, which suggests the presence of an acute infection. Hepatitis A infection will result in increased transaminase levels, with ALT typically becoming more greatly elevated than AST. Transaminase levels will increase before elevation in bilirubin. Alkaline phosphatase levels will also increase but to a minimal degree. IgM anti-HAV has a high sensitivity and specificity and can be detected even before the onset of symptoms and can persist for 4-6 months. Acute infections will have IgM elevation as part of antibody response before IgG elevations against HAV. IgG elevation will show previous infection and sufficient antibody protection. IgG elevation will rise after IgM elevation and will stay elevated throughout a person's lifetime.

A 69-year-old woman comes to her health care provider's office complaining of 1 week of crampy lower abdominal pain and bloody diarrhea. She had previously been followed for symptoms of stable exertional angina and hypertension. She had an uncomplicated myocardial infarction 3 years earlier. Her symptoms began 1 week ago with mild postprandial abdominal cramping followed by diarrhea, which became bloody after 2 days. She has not traveled recently and is a retired librarian. Her temperature is 38.6ºC (101.4ºF), blood pressure 120/84 mm Hg, and pulse 96/min. She has moderate tenderness to palpation of the left lower quadrant. Rectal examination reveals bloody stool and no masses. Which of the following is the most likely diagnosis? A. Arteriovenous malformation B. Diverticulitis C. Diverticulosis D. Ischemic colitis E. Ulcerative colitis

D. This elderly woman who has a history of atherosclerotic vascular disease as demonstrated by a history of a myocardial infarction and exertional angina has developed colitis symptoms, as demonstrated by the left lower quadrant pain and bloody diarrhea. This is typically caused by ischemia of small branches of the inferior mesenteric artery. The bleeding is caused by poor blood flow to the bowel, which leads to mucosal sloughing that can cause blood loss into the bowel lumen. The diagnosis is suspected clinically and generally confirmed with a flexible sigmoidoscopy, because many cases involve the rectosigmoid region. Arteriovenous malformation (choice A), most commonly located in the cecum and may be associated with aortic stenosis, produces painless bleeding in the elderly. Diverticulitis (choice B) does present with postprandial crampy left lower quadrant abdominal pain and fever, but does not produce bloody diarrhea. Often these patients have symptoms of constipation. Diverticulitis causes pain with cramping which can lead to abdominal distention and obstruction with the potential for perforation. Diverticulosis (choice C) also presents with postprandial crampy left lower quadrant pain and bright red blood per rectum or it may be asymptomatic, but it does not present with diarrhea. Diverticulosis is the most common cause of massive painless GI bleeding. Ulcerative colitis (choice E) typically presents in a subacute or chronic fashion in a younger patient population. Although it can occur in this patient's age group, it rarely presents as an acute illness. It typically presents with recurrent bouts of bloody diarrhea. Review The risk factors for ischemic colitis are aging and cardiovascular disease. Risk factors include atrial fibrillation, myocardial infarction, atherosclerosis, and heart failure. The presentation of patients who have ischemic colitis is typically abdominal pain that is variable, depending on the portion of the bowel that is involved. Patients will present with hematochezia or melena along with diarrhea and abdominal tenderness. Treatment depends on the underlying cause. Patients will need fluid resuscitation and possible embolectomy to restore blood flow to the colon.

A 24-year-old dress designer complains of a crampy periumbilical pain over the past 9 months since she began her first job. During that time she has experienced several episodes of constipation lasting 4-5 days, typically followed by 3-4 days of frequent loose bowel movements. She denies any bloody stools, fever, weight loss, or change in appetite. Her symptoms are generally milder on weekends. Physical examination is normal. White blood cell count is 6,700/mm3 (normal 5,000-10,000/mm3), hematocrit 38%, (normal 36-44%), and erythrocyte sedimentation rate 4 mm/h (normal <25 mm/h in women). Serum albumin and liver function tests are normal. Which of the following is the most likely diagnosis? A. Crohn's disease B. Diverticulosis C. Giardia infection D. Irritable bowel syndrome E. Ulcerative colitis

D. This patient has typical crampy abdominal symptoms with alternating diarrhea and constipation. She is a young individual who has chronic symptoms, which is very typical of irritable bowel syndrome. Patients who have this diagnosis will generally have symptoms of pain related to meals or stress and an alteration in bowel habits. This change in bowel pattern may take the form of frequent loose stools versus constipation, or a combination of both. These patients have no physical findings or laboratory results to suggest an inflammatory process. Dietary changes and stress reduction are usually the goals of therapy, with antispasmodic drug therapy for symptomatic relief. Crohn's disease (choice A) is associated with a number of inflammatory findings, including tenderness on abdominal examination, mucopurulent or bloody stools, weight loss, and/or leukocytosis, and/or elevated erythrocyte sedimentation rate (ESR). There is a great deal of pain associated with flares of this disease and the entire GI tract may be involved. Diverticulosis (choice B) rarely happens in patients this young and typically presents with constipation and intermittent pain without diarrheal symptoms. This disease tends to be silent until infection (diverticulitis) occurs. It is also the most common cause of massive, painless bleeding. Giardia infection (choice C) may cause chronic symptoms with abdominal cramping, but typically with symptoms of upper gastrointestinal involvement, such as nausea, vomiting, and eructations, because Giardia tends to live in the upper small bowel. Ulcerative colitis (choice E) will present with bloody stools and evidence of an inflammatory condition (e.g., elevated ESR). There is typically a pattern of increased pain and cramping along with bloody diarrhea. Review Irritable bowel disease typically has recurrent abdominal pain or discomfort that is associated with a change in stool frequency or pattern of stool. The pain or discomfort is typically relieved by defecation. Irritable bowel syndrome is more common in females and usually begins by early adulthood. Symptom onset may occur following enteric infection. Stress seems to worsen symptoms. Pattern of irritable bowel may be recurrent constipation, recurrent diarrhea, or an alternating pattern of diarrhea and constipation.

A 62-year-old social worker with a past history of recurrent GERD, complains of increasing difficulty with swallowing. She has had trouble with solid foods and senses a discomfort in the mid-lower chest after eating meats or dry bread. She has not lost any weight and denies any other medical problems. There is no family history of gastrointestinal malignancy. Physical examination is unremarkable. Which of the following would be the most appropriate next step in the evaluation of this patient's symptoms? A. Barium esophagram B. Chest radiograph C. CT scan of the chest D. Esophagoscopy E. 24-hour pH monitor

D. This patient is presenting with symptoms of a peptic stricture after many years of gastroesophageal reflux disease (GERD) symptoms. An endoscopy will allow evaluation of the stricture and biopsy to ensure that the condition is benign. Furthermore, it will allow dilatation of the stricture using an endoscopic-guided balloon to relieve the patient's symptoms. A barium esophagram (choice A) would demonstrate the typical smooth tapering nature of a peptic stricture, distinguishing it from the irregular, ulcerated, mass-like stricture seen in patients who have esophageal carcinoma. It would not conclusively make this distinction, however, and would require a follow-up endoscopy with biopsy anyway. A chest radiograph (choice B) is not of specific diagnostic value in patients who are undergoing evaluation for dysphagia. A CT scan of the chest (choice C) is not very useful in determining intraluminal esophageal lesions. CT scan is used for patients who have suspected malignant strictures to evaluate the paraesophageal regions and assess for the possibility of local spread. A 24-hour pH monitor (choice E) is useful to demonstrate that there is acid reflux, which is not in question with this patient who has the long-standing history of typical symptoms. Review Progressive dysphagia in an elderly patient requires evaluation. Ideally the patient who has dysphagia should have direct visualization with upper endoscopy, in which biopsy of any suspicious lesions can be performed. Esophageal cancer causes progressive dysphagia as the tumor encroaches on the lumen, whereas motor-related dysphagia has dysphagia for both solids and liquids from the outset.

A 25-year-old man presents with weight loss, abdominal pain, and bloody diarrhea. Sigmoidoscopy/colonoscopy reveals mucosal erythema and ulceration extending in a continuous fashion proximally from the rectum. Which of the following pathologic findings would also be characteristic of this patient's illness? A. Bowel wall thickening B. Cobblestone appearance of mucosa C. Fistulas D. Pseudopolyps E. Transmural lesions

D. You are being tested here on your ability to distinguish between ulcerative colitis (UC) and Crohn's disease. First, you need to figure out which one this patient has. Key clues are the bloody diarrhea (much more common in UC), the rectal involvement, and especially the continuous nature of the mucosal damage. Once you have identified UC, you need to identify the answer choice that is characteristic of UC. The correct answer is pseudopolyps, which are inflammatory polyps found in ulcerative colitis and not Crohn's disease. These post-inflammatory polyps (also known as pseudopolyps) are considered to be high risk for the development of colon cancer, because they are markers of significant inflammatory bowel disease. All of the other choices (choices A, B, C, and E) are features of Crohn's disease. Especially diagnostic is the transmural nature of the inflammation, which can lead to the development of fissures and fistulas. Remember also that although Crohn's disease can involve any part of the gastrointestinal tract, it typically does not involve the rectum and is usually found in the terminal ileum and/or colon. In contrast to UC, the lesions are discontinuous (skip lesions). Review Ulcerative colitis is more likely than Crohn's disease to have rectal involvement, mucosal involvement (rather than transmural involvement), continuous proximal spread of inflammation from the rectum, and bloody diarrhea. Because of significant inflammation, patients will have pseudopolyp formation, which is a feature that is much more likely to occur with ulcerative colitis than with Crohn's disease. Surgical intervention in ulcerative colitis is considered to be curative, whereas the disease will recur in a patient who has Crohn's disease, which is more likely to have GI involvement anywhere from the mouth to the anus, with perianal involvement frequently being present. Extraintestinal manifestations are part of both types of inflammatory bowel disease.

A 35-year-old woman complains of increasing weakness and shortness of breath. A complete blood count with differential indicates a megaloblastic anemia. Her vitamin B12 level is found to be low. She is also found to have hypothyroidism and diabetes and is given the diagnosis of chronic type A gastritis. Which of the following is associated with this illness? A. Antral involvement B. Decreased serum gastrin level C. Helicobacter pylori infection D. Non-steroidal anti-inflammatory drugs (NSAIDs) E. Parietal cell antibody

E. Chronic type A gastritis is immunologically mediated and is associated with an elevated level of parietal cell antibody. Acid secretion is thus reduced. The gastritis primarily involves the stomach body as well as the antrum (choice A). Due to the reduced acid level (choice B), the gastrin level would be high. Type B gastritis is more commonly associated with Helicobacter pylori infection (choice C). This pathogen is the most common cause of both duodenal and gastric ulcers. Chronic NSAID use (choice D) will lead to type B gastritis as well, involving the antrum of the stomach. Review Patients who have autoimmune disease will often have more than one abnormality associated with these conditions. Vitamin B12 deficiency can be caused by pernicious anemia, which has antibodies to parietal cells located in the stomach. Pernicious anemia results in achlorhydria with decreased stomach acid secretion. Chronic vitamin B12 deficiency caused by pernicious anemia will result in megalolastic anemia.

A 64-year-old man with a long history of "heart burn" comes to the emergency department with 6 episodes of hematemesis. He denies alcohol use, smoking, or drug use. He also denies significant nonsteroidal anti-inflammatory drug use. He has no known liver disease. His laboratory results are normal with the exception of hemoglobin 7.1 g/dL (normal male 13.8-17.2 g/dL). He is taken for an emergent esophageal gastroduodenoscopy, which reveals a gastric ulcer with a bleeding visible vessel. Compared with duodenal ulcers, gastric ulcers have which of the following? A. Better response to medication B. Less commonly associated with significant gastrointestinal bleeding C. Lower gastric pH D. More common association with H. pylori E. More common association with malignancy

E. Gastric ulcers may be associated with malignancy, and therefore biopsy should be performed when they are discovered. Repeat esophageal gastroduodenoscopy (EGD) is necessary to assure that these ulcers have healed. This association with malignancy is not found with duodenal ulcers. Gastric ulcers have a variable response to medications (choice A). In contrast, duodenal ulcers have a better and more reliable response to treatment with H2 blockers and proton pump inhibitors. As in our patient, gastric ulcers often present with significant gastrointestinal bleeding. These bleeds can be serious and life threatening and sometimes require operative interventions. Although duodenal ulcers can perforate and bleed, they do so less commonly than gastric ulcers (choice B). The gastric pH in patients who have gastric ulcers is higher (less acidic) than the gastric pH in patients who have duodenal ulcers (choice C). This might explain the discrepancy in response to acid suppressing medication in the two conditions. Initially it was believed that duodenal ulcers are more commonly associated with Helicobacter pylori than are gastric ulcers. Duodenal ulcers are almost all associated with H. pylori infection, whereas gastric ulceration occurs much more commonly with the use of NSAIDs, which makes choice D incorrect. Review Patients who have gastric ulceration should have a repeat EGD 6-8 weeks after treatment to ensure that healing has taken place and to rule out gastric cancer, because a small number of gastric cancers can present as gastric ulcers. Patients who have uncomplicated duodenal ulcers do not need follow-up endoscopy. If the patient was treated for H. pylori eradication, follow-up should be done to assess for cure, because resistance to treatment has been documented.

A young man is shot with a .45-caliber revolver point blank in the lower abdomen, just above the pubis. The entrance wound is at the midline, and there is no exit wound. Radiographs show the bullet embedded in the sacral promontory, to the right of the midline. Digital rectal examination and proctoscopic examination are unremarkable, but he has gross hematuria. He is hemodynamically stable. Which of the following is the most appropriate next step in management? A. CT scan of the abdomen B. Intravenous pyelogram C. Retrograde cystogram D. Diagnostic peritoneal lavage E. Exploratory laparotomy

E. He has an obvious indication for exploratory laparotomy: a gunshot wound to the abdomen. He also has evidence of injury to the urinary bladder, but that will be dealt with at the same time that other intra-abdominal injuries are found and repaired. CT scan (choice A) would not change the surgical approach and the surgical indication. CT scan is called for in cases of blunt trauma to diagnose intra-abdominal bleeding and to identify intra-abdominal injuries. Intravenous pyelogram (choice B) would indeed show the bladder injury, as would a retrograde cystogram (choice C). We already know clinically that there is a bladder injury, however; we know the trajectory of the bullet and we have blood in the urine. Diagnostic peritoneal lavage (choice D) is used to diagnose intra-abdominal bleeding in blunt trauma, when the patient is not stable enough to be taken to the CT scanner. In many centers the diagnostic peritoneal lavage has been replaced by sonogram done in the emergency department by the trauma team. Surgery should not be delayed while waiting for the results of these studies. Review Gunshot wounds to the abdomen may produce dramatic injuries caused by the bullet pattern on entry and exit. Even when a patient is stable, exploratory laparotomy should be performed to assess the extent of injuries, especially in the setting of a patient manifesting intra-abdominal injuries such as hematuria. The extent of injuries can be determined and surgical correction of those injuries can be undertaken during the surgery. Gross contamination caused by bowel disruption may result in the patient not having primary closure of the abdomen, and wound care will be a priority for these patients after stabilization and resuscitation is performed.

A 17-year-old boy is brought to the health care provider by his mother because "he has been sick for some time, and the health care providers do not seem to know what is wrong with him." The precipitating event for their visit today was the fact that the boy's sclera turned yellow a few days ago. On further questioning a history of protracted diarrhea is elicited. For about 3 years the patient has had intermittent episodes of crampy abdominal pain and diarrhea. On one occasion he almost had an appendectomy because of right lower quadrant abdominal pain; surgery was canceled when he improved with observation and a sonogram had been read as negative for acute appendicitis. At this time he has a bilirubin of 3.6 mg/dL (normal <1.9 mg/dL) with 2.9 mg/dL conjugated (normal <0.3 mg/dL), alkaline phosphatase 625 U/L (normal 44-147 u/L), and near-normal transaminases (AST, ALT levels). Sonogram of the right upper quadrant shows normal caliber extrahepatic ducts, no gallstones, and no evidence of liver abscess. Stool cultures are negative. He has never been out of the country. Which of the following is the most likely diagnosis? A. Acute hepatitis B. Amebic abscess of the liver C. Cancer of the head of the pancreas D. Ischemic colitis E. Sclerosing cholangitis associated with inflammatory bowel disease

E. Jaundice with very high alkaline phosphatase is consistent with cholangitis. Cholangitis is typically from obstruction or cholestasis, but sonogram does not show dilated extrahepatic ducts. Normal caliber extrahepatic ducts without gallstones or gallbladder sludge findings are most consistent with sclerosing cholangitis. Furthermore, the current liver findings must be in some way related to his intermittent diarrhea ("always seek a single diagnosis to explain all the findings"), and sclerosing cholangitis is seen in conjunction with inflammatory bowel disease, which in turn is a perfect bet for intermittent, crampy diarrhea in a 17-year-old. Acute hepatitis (choice A) should show high transaminases with near-normal alkaline phosphatase, and acute hepatitis would not explain the diarrhea. Additionally, this disease pattern would not occur on a chronic basis. Amebic abscess (choice B) could indeed tie together a history of diarrhea with current minimal jaundice and high alkaline phosphatase. The problem is that the sonogram should easily show the abscess, and that amebic abscess would be rare in someone who has never left the United States. Cancer of the head of the pancreas (choice C) would be rare at age 17, and it would produce dilatation of the extrahepatic ducts. Furthermore, such cancer would not be preceded by a history of intermittent diarrhea. Ischemic colitis (choice D) could explain diarrhea in a much older person, but it would not lead to liver involvement. Review The major risk factors for sclerosing cholangitis are male sex plus history of inflammatory bowel disease. There is also an increased risk for first-degree relatives of patients who have primary sclerosing cholangitis. Serum alkaline phosphatase levels can be elevated up to 3 times normal. The transaminase levels show only mild to moderate elevation in patients who have sclerosing cholangitis. Conjugated bilirubin levels will also be elevated, which pushes the total bilirubin levels to also be elevated. MRCP or ERCP may show intrahepatic and/or extrahepatic strictures and dilation with or without a prominent biliary stricture.

A 55-year-old HIV-positive man has a fungating mass growing out of the anus. He can feel it when he wipes himself after having a bowel movement, though it is not painful. For the past 6 months he has noticed blood on the toilet paper and from time to time there has also been blood coating the outside of the stools. He has lost weight, and he looks emaciated and ill. On physical examination the mass is easily visible. It measures 3.5 cm in diameter, is fixed to surrounding tissues, and appears to grow out of the anal canal. He also has rock-hard, enlarged lymph nodes on both groins, some of them as large as 2 cm in diameter. Which of the following is the most likely diagnosis? A. Adenocarcinoma of the rectum B. Condyloma acuminata of the anus C. External hemorrhoids D. Rectal prolapse E. Squamous cell carcinoma of the anus

E. The entire description is classic for anal cancer, but the best evidence is the presence of metastasis in the inguinal nodes. The mass that is visible is consistent with an anal mass. Adenocarcinoma of the rectum (choice A) could look like this if it arose very low in the rectum, but it would not metastasize to inguinal nodes. It is more likely that the visible mass began in the anal area and not in the rectum. Condyloma acuminata (choice B) could give fungating masses, but it would not lead to cachexia and would not produce the rock-hard inguinal nodes. Viral infections may precede the development of this kind of tumor (and could coexist with it), but it would be wrong to assume that all the patient has is the benign viral process. External hemorrhoids (choice C) are not fungating masses, and they are usually very painful. They do not lead to inguinal adenopathy or cachexia. Rectal prolapse (choice D) would produce a protruding mass with concentric mucosal folds, would come in and out with straining, and would be a nuisance--but it would not produce adenopathy and cachexia. Review The fungating anal mass with positive inguinal lymph nodes in a patient who has HIV should elevate your suspicions for anal cancer. The skin of the anus is drained by the inguinal lymphatic nodes, and the rectal mucosa is drained by internal lymph nodes. An anal mass with palpable inguinal lymph nodes should raise the suspicion for metastatic cancer.

A 5-day-old full-term boy born at home is being evaluated in the urgent care clinic for bruising and gastrointestinal bleeding. Laboratory findings include: Partial thromboplastin time and prothrombin time >2 minutes (normal for newborns ≤60 seconds) Serum bilirubin 4.7 mg/dL (normal for newborns) Prothrombin time 15.2 seconds (normal 11-13 seconds) Alanine aminotransferase 18 mg/dL (normal <45 mg/dL) Platelet count 330,000/mm3 (normal 150,000-300,000/mm3) Hemoglobin 16.3 g/dL (normal 13.2-21.0 g/dL) The boy's mother has factor V Leiden deficiency. Which of the following is the most likely cause of the boy's bleeding? A. Factor VIII deficiency B. Factor IX deficiency C. Idiopathic thrombocytopenic purpura D. Liver disease E. Vitamin K deficiency

E. The infant in this clinical vignette has hemorrhagic disease of the newborn as a result of vitamin K deficiency. It was a major cause of bleeding in neonates in the past, but it is now uncommon because of the routine administration of vitamin K at birth. It is still encountered in situations in which infants are born outside the hospital, however. The normal newborn has a moderate deficiency of the vitamin K-dependent coagulation factors. The plasma levels of these factors fall even further during the first 2-5 days of life, rise again when the infant is 7-14 days old, and attain normal adult levels at about 3 months of age. This variation usually does not produce any bleeding or bruises. In hemorrhagic disease of the newborn, however, the initial fall is accentuated, and the restoration is delayed and incomplete. As a result, coagulation abnormalities become severe and bleeding may occur. All newborns should receive 0.5-1.0 mg of vitamin K intramuscularly within the first hour after birth. Prematurity has been associated with hemorrhagic disease of the newborn. Delayed feeding, breast-feeding (low vitamin K concentrations in breast milk), vomiting, severe diarrhea, and antibiotics also delay the colonization of the gut by bacteria. Bleeding is usually severe and occurs most commonly on the second or third day of life. The most common manifestations are melena, large cephalohematomas, and bleeding from the umbilical stump and after circumcision. Generalized ecchymoses, often without petechiae, intracranial bleeding, and large intramuscular hemorrhages also may develop in severe cases. In infants who have hemorrhagic disease of the newborn, the prothrombin time (PT) is always prolonged. The partial thromboplastin time (PTT) and the thrombin time are also prolonged. Specific factor assays reveal deficiencies of prothrombin; factors VII, IX, and X; and proteins C and S. The bleeding time and the platelet count usually are within normal limits. In the differential diagnosis of hemorrhagic disease of the newborn, virtually all causes of bleeding, particularly thrombocytopenia and disseminated intravascular coagulation (DIC) must be considered. Factor VIII deficiency (choice A), or hemophilia A, is caused by factor VIII deficiency. Factor VIII is the critical cofactor for generation of factor Xa by factor IXa. Significant prolongation of the PT is not found in hemophilia A. Factor IX deficiency (choice B), or hemophilia B (also known as Christmas disease), is not associated with significant prolongation of the PT. Idiopathic thrombocytopenic purpura (choice C) is usually characterized by a platelet count of <20,000/mm3. It often follows an acute infection and has spontaneous resolution within 2 months. Liver disease (choice D) would cause prolongation of PT and PTT, as well as liver enzymes. Review Hemorrhagic disease of the newborn is a consequence of vitamin K deficiency. Children born outside the hospital are prone to vitamin K deficiency, while those in the hospital setting are routinely given intramuscular vitamin K. Susceptibility period is 2 to 7 days after birth, with the most common manifestations bleeding from the umbilical stump, melena, large cephalohematoma, and bleeding after circumcision. This condition can occur idiopathically or can be from delayed or inadequate intake. It is prevented by the routine administration of vitamin K at birth.

A 4-month-old boy has gained only 10 ounces since birth. He has failed to gain weight with multiple different formula preparations. His stools have been loose and fatty. An older sister had similar symptoms and has been repeatedly hospitalized for failure to thrive and recurrent pulmonary infections. Which of the following is the most likely cause of this patient's gastrointestinal symptoms? A. Achlorhydria B. Bacterial overgrowth C. Colonic inertia D. Gastric hypersecretion E. Pancreatic exocrine insufficiency

E. The patient described here has cystic fibrosis. This inherited disease primarily affects the gastrointestinal and respiratory systems. Cystic fibrosis can present in early infancy with meconium ileus, in the first year of life with steatorrhea and failure to thrive, or in older childhood with recurrent pulmonary infections. The pancreatic secretions are characteristically very thick and tend to occlude the pancreatic duct system. Steatorrhea has developed in this infant because of pancreatic duct inspissation of secretions and the resultant decrease in delivery of pancreatic enzymes to the small bowel. Achlorhydria (choice A) is not a feature of cystic fibrosis, but it can complicate gastric disorders, including pernicious anemia. Bacterial overgrowth (choice B) is seen in conditions with poor intestinal motility and can cause vitamin deficiencies and malabsorption secondary to degradation of biliary and pancreatic secretions. Although meconium can cause ileus in newborns who have cystic fibrosis, this process usually involves the small intestine rather than the colon (choice C). Gastric hypersecretion (choice D) is not a feature of cystic fibrosis, but it can be seen in patients who have gastrin-secreting pancreatic tumors (Zollinger-Ellison syndrome). Review Cystic fibrosis is a genetically inherited condition that affects multiple organs, especially the GI and respiratory tracts. There is a defect in the salt and water transport across the epithelial surfaces that results in pancreatic dysfunction resulting in calorie malabsorption. There is also lung disease from mucus retention, infection, and inflammation. Pancreatic enzymes are given to support the patient's growth and nutrition.

A 21-year-old college senior comes to the health care provider with a 2-month history of frequent episodes of loose stool, preceded by lower abdominal cramping. Over the past 4 weeks, the stools have become increasingly bloody. On a number of occasions he has had the sensation of rectal fullness but is unable to pass any fecal matter. A sigmoidoscopy reveals inflammation in a circumferential pattern from the anal verge to the mid-sigmoid colon, where a transition to normal mucosa is seen. Which of the following is the most appropriate treatment for this patient? A. IV hydrocortisone B. IV infliximab (Remicade) C. Oral azathioprine (Imuran) D. Oral prednisone E. Topical mesalamine (Asacol, Pentasa)

E. The patient described here has ulcerative colitis confined to the distal colon, also known as ulcerative proctosigmoiditis. Because the disease is limited to the distal colon, topical agents such as mesalamine (or alternatively hydrocortisone) would be effective in reducing inflammation. Mesalamine is an anti-inflammatory drug used principally to control ulcerative colitis. Its active ingredient is also known as 5-aminosalicylic acid, which is available in the forms of rectal suspension, suppositories, delayed release oral tablets, and controlled release oral capsules. The mode of action is unknown but is thought to involve topical (because mesalamine is poorly absorbed), rather than systemic, modulation of arachidonic acid metabolites, including prostaglandins, leukotrienes, and hydroxyeicosatetraenoic acids. It is usually well tolerated, but it can cause significant allergic reactions related to sulfite sensitivity. IV hydrocortisone (choice A) is reserved for patients who do not respond to high doses of oral prednisone. It can also be used in fulminant disease to try to obtain control acutely. IV infliximab (Remicade) (choice B) is used for patients who have severe refractory Crohn's disease. It can be used as an adjunctive agent with cyclosporin in the management of fulminant disease if there is suboptimal response to IV corticosteroids after 72 hours of treatment. Oral azathioprine (Imuran) (choice C) is used in Crohn's disease and ulcerative colitis in patients already refractory or dependent on steroids to control symptoms or maintain remission. Azathioprine is considered to be steroid-sparing and is primarily used in refractory or corticosteroid-dependent disease. Oral prednisone (choice D) is not warranted for those who have not been previously tried safer medications, such as topical mesalamine or hydrocortisone or oral mesalamine or sulfasalazine. Review Ulcerative colitis treatment is dependent on the activity of the disease and the extent of colon involvement. The goal of therapy is to induce and to maintain remission. For mild to moderate disease affecting the distal colon, topical mesalamine is clearly a first-line choice. This is also the treatment of choice for distal colitis that is in remission. Oral mesalamine may be needed for patients who have extensive disease of the colon that has extensive colitis.

A 31-year-old woman smashes her car against a bridge abutment. She sustains multiple injuries, including upper and lower extremity fractures. She is fully awake and alert, and she reports that she was not wearing a seat belt and distinctly remembers hitting her abdomen against the steering wheel. Her blood pressure is 135/75 mm Hg and pulse88/min. Physical examination reveals a rigid and tender abdomen. There is severe tenderness when external pressure is applied to her abdomen and then suddenly released. She has no bowel sounds. Which of the following would be the most appropriate step in evaluating potential intra-abdominal injuries? A. Continued clinical observation B. CT scan of the abdomen C. Sonogram of the abdomen D. Diagnostic peritoneal lavage E. Exploratory laparotomy

E. The presence of an "acute abdomen," which this woman has, is an indication for exploratory surgery and prompt repair of the injuries (probably affecting hollow viscera) that have produced the signs of peritoneal irritation. Continued clinical observation (choice A) would be irresponsible when there is sufficient clinical evidence of an acute abdomen. CT scan (choice B) is ideal when the issue is potential intra-abdominal bleeding in a hemodynamically stable patient who can be safely sent to the radiology department. CT scan might even be a good idea if the picture of acute abdomen were equivocal. It is not needed here, however. Diagnostic peritoneal lavage (choice D) and sonogram done in the emergency department (choice C) are our options when we suspect intra-abdominal bleeding and the patient is too unstable to be sent anywhere. When an acute abdomen has clearly developed, it is time to operate. Review Remember how to approach abdominal blunt trauma: IV fluid resuscitation is mandatory; locating the bleeding is also a priority that will determine the treatment approach. If there are signs of peritoneal irritation that confirm the abdominal origin of the bleeding, go straight to the OR. If there is doubt of internal bleeding or its location, investigate further; if unstable after resuscitation, do ultrasound at the emergency department or OR. If the patient is stable, perform a CT scan.

A 25-year-old man sustains multiple stab wounds to the abdomen when he is mugged while jogging. The assault takes place in the evening. He is dumped by the attackers behind thick vegetation and is not found until the next morning, at which time exploratory surgery reveals multiple small bowel and colonic lacerations. All of the lacerations are repaired. In the post-op period, the patient has persistent hypotension, even though he received adequate fluid infusion and his central venous pressure is 12 mm Hg. Further studies done with the help of a pulmonary artery catheter reveal high cardiac output and low peripheral resistance. Which of the following is the most likely diagnosis? A. Adrenal insufficiency B. Cardiogenic shock C. Hypovolemic shock D. Neurogenic shock E. Septic shock

E. The signature of septic shock in the normovolemic patient is high cardiac output and low peripheral resistance. The findings are classic, and the circumstances (massive peritoneal bacterial contamination) easily explain its development. Adrenal insufficiency (choice A) is always part of the differential to consider when adequate fluid resuscitation treatment for shock fails to improve a patient who, unknown to us, may have adrenal suppression. In clinical practice, most adrenal insufficiency states are from exogenous steroid administration. The key to recognizing this problem is the history, with the sequestration of bacteria from the gut. High cardiac output and low peripheral resistance are not the typical findings in a patient with adrenal crisis who has problems maintaining salt and vascular tone. Patients who have adrenal insufficiency will have a low central venous pressure caused by the body's impaired ability to retain salt and to maintain vascular tone. Patients who have cardiogenic shock (choice B) would have high CVP, higher than the 12 quoted, as well as low cardiac output from pump failure. These patients would also have increased venous distention caused by the inability to maintain the cardiac output with resultant fluid backup into the pulmonary and systemic systems. Acute myocardial infarction with pump failure is a common precipitating factor for cardiogenic shock. A patient may develop cardiogenic shock from pericardial tamponade or tension pneumothorax, but there is no history of chest trauma that would precipitate these conditions. Hypovolemic shock (choice C) would have a CVP near zero and a low cardiac output, because the patient would be significantly behind in fluid resuscitation. Neurogenic shock (choice D) in the trauma patient would result from high spinal cord transection, an injury that we were not told this man had. The peripheral resistance would be low, as would be the CVP and the cardiac output. Patients who have neurogenic shock classically have a disconnection between the brain and the heart, with these patients classically manifesting hypotension and an inappropriate bradycardia despite the hypotension. Review The signatures of shock in the different causal conditions are: Septic shock: high cardiac output and low peripheral resistance leading to hypotension Hypovolemic shock: low CVP and low cardiac output Neurogenic shock: low CVP and low cardiac output Cardiogenic shock: high CVP and low cardiac output caused by pump failure activity of the heart

A 2-month-old boy is brought to the office because of chronic constipation and failure to thrive. He has had infrequent bowel movements since birth, usually no more than once a week and with great difficulty. The mother tried changing to formula instead of breast-feeding, hoping that it would help, but that seemed to have made it worse. In addition to that, he has not been gaining weight as she had expected. He weighed 2,900 grams at birth and now weighs only 3,500 grams. Her pregnancy was uneventful and she delivered vaginally at full term without complications. The infant did not pass meconium until the third day, however, and had several episodes of vomiting in the first few weeks of life. On physical examination the patient seems in no apparent distress. He is in the 30th percentile for length and 5th percentile for weight. His abdomen is moderately distended and nontender to palpation. Anal sphincter tone is normal and rectal vault is empty of stool. Barium enema shows a transition zone in the colon. A biopsy of the rectosigmoid confirms absence of ganglion cells. Which of the following is the most appropriate treatment for this patient? A. Counseling B. Daily enema C. Dietary manipulation D. Stool softeners E. Surgery

E. This patient has Hirschsprung's disease, the most common cause of constipation in neonates and infants. Treatment of choice is surgical resection of the aganglionic segment. Constipation is the infrequent passage of hard, dry stools. Obstipation is the absence of bowel movements. Functional constipation (voluntary withholding) is the most common cause of constipation outside of infancy. Constipation occurs secondary to defects in filling or emptying the rectal vault. Other causes of constipation include imperforate anus, cystic fibrosis with meconium ileus at birth, an anteriorly displaced anus, and Hirschsprung's disease. Infantile botulism may also cause constipation. Hard stools are passed infrequently, sometimes after several days, and with difficulty. Occasionally liquid stool can pass around the obstruction and cause the false impression of diarrhea and encopresis. Constipation in the neonate should be considered Hirschsprung's disease until proven otherwise. In the United States, the incidence of Hirschsprung's disease is approximately 1 in 500 live births, and is 4 times more common in boys than in girls. The aganglionic segment is located in the colon, usually rectosigmoid. Infants may have failure to thrive and abdominal distention. Diagnosis is typically made during the first year of life. Hirschsprung's disease is diagnosed by biopsy specimen showing areas devoid of ganglion cells. Barium enema shows a megacolon with a transition zone between the normal and aganglionic segment of the colon. Newborns who have this condition receive colonic irrigation to manage colonic distention before surgical intervention. Treatment of Hirschsprung's disease is surgical. Treatment of functional constipation includes initial cleaning out and may involve dietary manipulation, stool softeners, and counseling. Irrigation of the colon is not effective in patients who have Hirschsprung's disease that affects the entire colon. An ileostomy is initially required in these patients, with definitive surgery needed later in life. Counseling (choice A) is not the appropriate treatment for this patient. For one, he is only 2 months old and counseling needs adequate cooperation of the child to be useful. Also, Hirschsprung's disease is amenable only to surgical treatment, as the aganglionic segment cannot be made functional with any other therapeutic modality. Daily enema (choice B) is used to accomplish bowel cleaning in patients who have Hirschsprung's disease awaiting surgery, but that is not the most appropriate treatment, as it does not lead to any lasting improvement in the patient's bowel habits. Dietary manipulation (choice C) and stool softeners (choice D) have no effect in patients who have Hirschsprung's disease, because a section of their bowel is immobile from the lack of ganglion cells. They may be helpful in patients who have functional constipation in whom the gastrointestinal system is functionally intact. Review Hirschsprung's disease is a congenital condition characterized by partial or complete obstruction of the bowel caused by the absence of intramural ganglion cells. This aganglionic colon results in the lumen being contracted, which leads to the bowel obstruction. The aganglionic segment is located distally in the colon, and the length of colon that is affected is dependent upon how much of the colon lacks these ganglionic cells. Surgical correction involves removal of the distal aganglionic segment with pull-through of the proximal normal ganglionic bowel.

A 39-year-old woman comes to the health care provider with complaints of difficulty swallowing. She has a history of scleroderma for the past 15 years, during which time she has required a variety of medications to reduce the symptoms of heartburn. Over the past 6 months, she has also noted difficulty swallowing food, such as steak, and has felt as if food "sticks" in her lower chest. She is able to tolerate liquids without difficulty. She denies any weight loss. Which of the following is the underlying cause for these symptoms? A. Development of a squamous carcinoma in the upper third of the esophagus B. Recent return of peristaltic activity in the body of the esophagus C. Reverse peristalsis in the body of the esophagus D. Reverse peristalsis of the lower esophagus E. Scarring at the lower esophagus because of chronic acid reflux

E. This patient has a history of systemic sclerosis, also known as scleroderma. This condition can cause esophageal dysfunction secondary to fibrosis, which is a common manifestation of visceral disease. Patients who have scleroderma are very vulnerable to acid reflux, because neither peristalsis nor the lower esophageal sphincter functions normally after much of the muscle tissue has been replaced by fibrosis. The situation is complicated by the fact that persistent acid reflux will eventually induce scarring, with formation of a peptic stricture in the distal esophagus. Care should be taken to carefully manage even mild reflux symptoms in patients who have scleroderma to prevent (or at least to slow) the development of this troubling complication. Although chronic acid reflux can predispose for Barrett's esophagus with the risk for progression to adenocarcinoma of the distal esophagus, squamous carcinoma of the upper third of the esophagus (choice A) would not be an expected complication. Once peristalsis is lost, there is almost never a return of peristaltic activity (choice B). Reversed peristalsis (choices C and D) is not usually seen clinically. Disordered peristalsis with increased esophageal tone can be seen in symptomatic diffuse esophageal spasm. Review Chronic transmural inflammation from systemic sclerosis may result in stricture formation within the esophagus, leading to esophageal stricture and symptoms of dysphagia. Patients who have scleroderma have fibrosis of the esophagus caused by lower esophageal dysmotility with either hypomotility or aperistalsis along with a decrease in the lower esophageal sphincter tone, which allows further reflux into the lower portion of the esophagus.

A 50-year-old chronic alcoholic presents with dementia, paralysis of lateral gaze, and difficulty walking. Which of the following vitamin deficiencies is most likely responsible for this clinical picture? A. Vitamin B12 B. Vitamin B6 C. Folate D. Vitamin D E. Vitamin B1 (thiamine)

E. This patient has all the signs of Wernicke's encephalopathy, a disease characterized by dementia, ataxia, and ophthalmoplegia. This disease typically occurs in alcoholic patients and is the result of thiamine deficiency. Risk factors for thiamine deficiency include alcohol dependence, malabsorption, and a diet low in thiamine, such as the consumption of polished rice or an alcoholic who obtains the majority of his calories via alcohol rather than food. Wet beriberi can also be a complication of thiamine deficiency, which can present with high-output heart failure with edema and orthopnea. Dry beriberi, another complication, occurs with chronic deficiency states and is associated with distal peripheral polyneuropathy. Vitamin B12 deficiency (choice A) is associated with peripheral neuropathy, memory loss, paresthesias, gait disturbance, and glossitis. Cobalamine levels will be decreased. There is associated macrocytosis, and homocysteine levels are elevated. Patients who have gastric bypass or autoimmune disease, poor intake (long-term vegans), and those who have atrophic gastritis are at risk for this condition. Vitamin B6 or pyridoxine (choice B) -deficient patients typically present with peripheral neuropathy. Patients who are most at risk for this condition are those who take isoniazid on a long-term basis if they are not given pyridoxine supplementation. Folate deficiency (choice C) is a common complication of alcoholism, because alcoholic patients tend to get all of their calories from alcohol and not from dietary intake of foods containing folate. Folate deficiency is associated with macrocytosis, but there are no associated neurologic deficits. Vitamin D deficiency (choice D) states are typically asymptomatic. They are associated with growth retardation and rickets (children), and osteomalacia, osteopenia, and osteoporosis (adults). Review Wernicke's encephalopathy is a complication of acute deficiency of thiamine. It is clinically manifested by alterations in consciousness, eye movement abnormalities, and gait and balance disorders. It needs to be recognized and treated emergently with thiamine replacement; if not, permanent neurologic deficits will persist.

A 44-year-old woman with a history of recurrent biliary colic presents with 18 hours of very severe right upper quadrant pain, fever, and jaundice. An abdominal ultrasound reveals a markedly dilated common bile duct. Multiple gallstones are seen in the gallbladder. Which of the following would best determine whether there is cystic duct obstruction? A. Abdominal CT scan B. Abdominal MRI C. Abdominal ultrasound with Doppler flow studies D. Endoscopic retrograde cholangiopancreatograph (ERCP) E. HIDA scan

E. This patient has evidence of a prolonged episode of biliary colic, which may progress to acute cholecystitis. An abdominal ultrasound has revealed a markedly dilated common bile duct, as well as gallstones within the gallbladder. Acute cholecystitis is the result of cystic duct obstruction, and this would be best demonstrated by a HIDA scan. A HIDA scan shows visualizes the iminodiacetic acid, which follows the flow of bile (i.e., from the bloodstream into the liver, into the gallbladder, and then through the cystic duct into the common bile duct). An abdominal CT scan (choice A) may reveal gallstones and pericholecystic changes but does not demonstrate good visualization of the cystic duct. It also does not assess the functional capability of the gallbladder. The abdominal CT is definitely inferior to the ultrasound when used in the assessment of a patient for possible gallbladder disease. Abdominal MRI scan (choice B) is not effective in the demonstration of cystic duct obstructions. Although this test is inferior to the ultrasound study, it can be done during pregnancy when a pregnant patient is being evaluated for abdominal pain. An abdominal ultrasound (choice C) is not as effective in revealing cystic duct obstructions, and Doppler flow studies are useful for blood flow patterns but would not assist in the diagnosis of acute cholecystitis. ERCP (choice D) is more effective in demonstrating the intra- and extrahepatic biliary trees but not as effective in demonstrating the cystic duct. This test may be done as part of the diagnostic and therapeutic intervention for a patient who has choledocholithiasis. Review The HIDA scan or cholescintigraphy scan will directly show cystic duct obstruction by showing a failure of gallbladder filling with normal hepatic uptake of iminodiacetic acid. For a patient who has no blockage of the cystic duct, the test is normal. This test will also show the function of the gallbladder; some patients do not have cystic duct blockage but rather a failure of the gallbladder to contract. This would make the gallbladder dysfunctional, indicating a need for cholecystectomy.

A 51-year-old welder comes to the health care provider complaining of severe fatigue and the onset of jaundice. He has a known history of hepatitis C, which he acquired after intravenous drug use 20 years earlier. Over the past 6 months he has developed ascites and has had two admissions to the hospital for esophageal variceal bleeding. On physical examination he is icteric with bitemporal wasting and multiple stigmata of chronic liver disease. On abdominal examination, the liver is percussed to be 7 cm in the midclavicular line and splenomegaly is present. There is near-tense ascites and moderately severe lower extremity edema that extends to the mid-calf. Laboratory results reveal an albumin of 2.1 g/dL (normal 3.4-5.4 g/dL), total bilirubin 12.1 mg/dL (normal <1.9 mg/dL), and prothrombin time 19 seconds (normal 11-13 seconds). Which of the following is the most appropriate therapy? A. Interferon B. Ribavirin C. Interferon plus ribavirin D. Mesocaval shunt E. Evaluation for liver transplantation

E. This patient has known hepatitis C. Features indicating that he has advanced signs of cirrhosis and portal hypertension include ascites, splenomegaly, jaundice with elevated bilirubin, hypoalbuminemia with leg edema, increased prothrombin time probably secondary to inadequate synthesis of clotting factors by the liver, and esophageal variceal bleeding. In the setting of disease this advanced, antiviral treatment for hepatitis C is without value and the patient should undergo evaluation for a liver transplant. As discussed above, all anti-viral therapy (choices A, B, and C) would be futile given the advanced stage of his cirrhosis at this point. A mesocaval shunt (choice D) is a surgical procedure whereby portal flow is diverted from the superior mesenteric vein into the inferior vena cava to reduce portal pressures. It is often complicated by encephalopathy and does not improve the underlying liver dysfunction. Review Patients who have hepatitis C may progress to end-stage liver disease, which is clinically apparent by tense ascites, jaundice, increased bilirubin, and prolongation of the INR and prothrombin time. The only effective therapy for end-stage liver disease is liver transplantation. Five-year survival after liver transplantation is limited because of hepatitis C infection, which will subsequently occur in the new liver and cause the same progressive liver damage pattern that the original liver acquired.

A 45-year-old woman has been experiencing generalized weakness and a sensation of "pins and needles" for the past 3 week. She exercises daily, rarely drinks alcohol, and is a strict vegetarian. Her temperature is 37ºC (98.6ºF), blood pressure 110/70 mm Hg, pulse 60/min, and respirations 18/min. Examination shows weakness of the proximal and distal muscles of the lower extremities. Deep tendon reflexes are increased. The gait is ataxic. Which of the following is the most likely cause of these symptoms? A. Guillain-Barré syndrome B. Lambert-Eaton syndrome C. Myasthenia gravis D. Polymyositis E. Vitamin B12 deficiency

E. This patient has subacute combined degeneration of the spinal cord, which is caused by a vitamin B12 deficiency. It is most often caused by pernicious anemia, but it may be acquired by patients who have strict vegetarian diets or small bowel disease. The clinical manifestations include weakness, paresthesias, loss of vibratory sensation, increased deep tendon reflexes, and extensor plantar responses. The gait is ataxic. Mental changes may also occur. The diagnosis is made by measuring serum vitamin B12 levels. Vitamin B12 deficiency occurs only after years of being a vegetarian, because vitamin B12 stores are present for years before depletion occurs. Neurologic changes caused by pernicious anemia are considered to be irreversible, so early treatment is warranted. The treatment is vitamin B12 replacement. Guillain-Barré syndrome (choice A) is an acquired demyelinating neuropathy that usually follows a viral respiratory infection. It is characterized by ascending weakness, whereas sensation is largely intact. A main diagnostic clue is absent deep tendon reflexes, which occurs with this disorder. Lambert-Eaton syndrome (choice B) is a neuromuscular disorder that causes proximal muscle weakness, ptosis, and diplopia. Deep tendon reflexes are depressed or absent. Repetitive nerve stimulation shows increased responses. Small-cell carcinoma of the lung has been associated with this disorder. Myasthenia gravis (choice C) is a neuromuscular disorder caused by autoantibodies to acetylcholine receptors. It is characterized by weakness and easy muscle fatigability. The extraocular and eyelid muscles are often affected, leading to diplopia and ptosis, but deep tendon reflexes are preserved. Polymyositis (choice D) is a skeletal muscle disorder that is characterized by progressive proximal muscle weakness. Patients often complain of difficulty climbing stairs and brushing hair. Ocular muscles are generally not affected. Creatine kinase is elevated, but muscle biopsy and electromyography confirm the diagnosis. Review Vitamin B12 deficiency is a condition that clinicians need to diagnose and treat. Typical symptoms which may occur with long-term vitamin B12 deficiency are: Pallor Weakness Paresthesia Ataxic gait Loss of vibratory sensation Increased deep tendon reflex Extensor plantar responses Dementia

A 54-year-old woman complains of severe lower abdominal pain and distention. The symptoms began approximately 24 hours ago, when her abdomen became visibly swollen and she developed nausea and vomiting. She has not moved her bowels over the past 24 hours. Over the past 4 months, she has lost 14 pounds and has noted progressive symptoms of constipation. On several occasions she has noted blood mixed in with her bowel movements, which have become thinner in caliber. She denies any recent travel, use of antibiotics, or fevers. On physical examination she appears acutely uncomfortable and has a temperature of 38.3ºC (100.9ºF). The abdomen is diffusely distended and tender to palpation in the left lower quadrant. There are hyperactive rushing bowel sounds. On rectal examination the stool is brown and guaiac positive. An obstructive series reveals multiple small bowel air fluid levels and a dilated colon proximal to the sigmoid colon. Which of the following is the most likely diagnosis? A. Amebic abscess B. Colon polyp C. Diverticulitis D. Diverticulosis E. Sigmoid carcinoma

E. This patient has symptoms of a chronic gastrointestinal process as demonstrated by her weight loss, change in bowel habits, and thinner caliber stools with bleeding. The thinner caliber stools specifically suggest that a mass lesion or luminal narrowing is present. Of the lesions listed, only colonic cancer would be likely to produce this pattern. In other settings, Crohn's disease and tuberculosis of the colon could produce similar clinical patterns. This patient's change in symptoms over the past 24 hours suggests that she may have developed an acute large bowel obstruction. An amebic abscess (choice A) would be expected to give more focal findings and symptoms of high spiking fevers. Furthermore, there would usually be a history of travel diarrhea to suggest this diagnosis. A colon polyp (choice B) will very rarely produce obstructive symptoms and does not produce the symptoms of weight loss and persistent thin caliber stool. Diverticulitis (choice C) is a result of a micro- or macroperforation of a diverticulum and results with an acute onset of local inflammatory signs, usually in the left lower quadrant, corresponding to the sigmoid colon diverticulosis. It would not affect the stool pattern between episodes of flare-up of the diverticulitis. Diverticulosis (choice D) may give her crampy lower abdominal pain and, on rare occasions, may produce obstructive symptoms. This patient's symptoms of weight loss and bloody bowel movements with thinner caliber stools are more suggestive of colon carcinoma, however. Review Left-sided colon cancer can present with ongoing pain in the left lower quadrant (later sign of progressive disease), change in bowel habits with increased frequency or loosening of stools, and rectal bleeding. As the tumor encroaches on the bowel wall, the stools change in caliber with progressively thinner stools. Ongoing untreated sigmoid colon cancer will cause anemia because of the rectal bleeding that occurs. Right-sided colon cancer typically presents with anemia caused by ongoing bleeding and not with obstructive-type symptoms, because the stool is very liquid in the right colon, which makes obstruction much less likely.

A 4-month-old girl is brought to the office because of frequent crying, sleep disturbance, and decreased appetite. She has been irritable and crying a great deal since birth, especially after feeding when she might spit up and even vomit. She also seems to get hiccups frequently. She has not been gaining weight as expected and her parents are concerned that something may be wrong. Last night they thought she had stopped breathing for approximately 20 seconds. She has been on a diet of cow's milk and breast milk. On physical examination the infant seems in no acute distress. Her vital signs are within normal limits and her weight and length are in the tenth percentile. Otherwise the physical examination is within normal limits. Which of the following is the most appropriate next step in management? A. Inserting a nasogastric feeding tube B. Nissen fundoplication C. Prokinetics D. Proton pump inhibitors E. Thickening of feeds with cereal and elevating the head of the bed

E. This patient has the typical presentation of gastroesophageal reflux. Gastroesophageal reflux occurs with a reduction in lower esophageal sphincter pressure, inappropriate lower esophageal sphincter relaxation, hiatal hernia, or delayed gastric emptying. This condition is common in the pediatric population, particularly in infants with developmental delay or cerebral palsy. It is a minor condition and of no consequence (so-called "functional gastroesophageal reflux"); patients present with a wide array of symptoms. Some form of spitting up and even forceful vomiting are common. Apnea can be a presenting sign. Reflux into the hypopharynx triggers laryngospasm and subsequent obstructive apnea. Chronic cough and wheezing may signal aspiration. Some patients exhibit poor weight gain and failure to thrive. Sandifer syndrome presents with gastroesophageal reflux and opistotonus, presumably to avoid aspiration or decrease pain. A pH probe is the standard method that is used for diagnosing gastroesophageal reflux. Other imaging studies include technetium scanning and barium swallow. Initial therapy consists of antireflux measures, such as elevating the head of the bed and thickening of feeds. If this fails to provide relief of symptoms, medical management with antacids, prokinetics, H2-receptor blockers, and proton pump inhibitors should be attempted. Failure of medical management may require surgical correction with a Nissen fundoplication. Most patients, however, have resolution of symptoms without any treatment. A nasogastric feeding tube (choice A) would not alleviate a problem caused by the inappropriate relaxation of the lower esophageal sphincter. It is arguable that a continuous low-volume feeding rate would prevent reflux, but the patient is in no acute distress and has only minor signs of gastroesophageal reflux. Nissen fundoplication (choice B) is the last resort for patients who have intractable gastroesophageal reflux resistant to medical management. Prokinetics (choice C) and proton pump inhibitors (choice D) are used in the medical management of gastroesophageal reflux. They should not be the initial step in the management of this patient, however. Measures such as thickening of the feeds, giving smaller and more frequent meals, elevating the head of the bed, and keeping the child in an upright position for a while after every feeding should be attempted first. Review Gastroesophageal reflux is a common entity encountered in clinical practice, with 10 to 20% of the population having GERD. All age groups are affected and this condition is a common occurrence in infants. Infants who have developmental delays and cerebral palsy are especially affected by GERD. First-line management consists of thickening the feeding and keeping the infant in a more upright position during and following feeding. If these modifications are ineffective, proton pump inhibitors may be added as first-line management.

A 22-year-old sociologist returns from 6 months in Jamaica and notes a 6-pound weight loss during her first month back home. She reports that her appetite is normal. She has developed symptoms of mild abdominal cramping and bloating after meals and frequent greasy bowel movements. Physical examination is consistent with evidence of recent weight loss and abdominal examination is normal. A stool specimen is guaiac-negative. There are scattered ecchymoses on all 4 extremities. Laboratory results reveal an albumin of 2.9 g/dL (normal >3.8 g/dL), INR 1.9 (normal 1.0 in patients who are not taking warfarin), and normal liver function tests. A trial of a gluten-free diet is attempted. There is no change in symptoms over the subsequent 3 weeks, and she has an additional 4-pound weight loss. Stool cultures for enteric pathogens and ova and parasites are negative on 3 occasions. Which of the following is the most likely diagnosis? A. Celiac sprue B. Enterohemorrhagic Escherichia coli C. Enterotoxigenic Escherichia coli D. Pancreatic insufficiency E. Tropical sprue

E. This patient presents with signs of malabsorption as described by her loss of weight, frequent loose stools, and evidence of multiple vitamin deficiency. She is also hypoalbuminemic and has just returned from a tropical region. There are no risk factors or evidence, by history, for liver disease. The hypoalbuminemia, ecchymosis, and INR are explained by the malabsorptive process instead, which is consistent with a vitamin K deficiency. The findings are consistent with a tropical sprue. These patients often develop a vitamin B12 deficiency, as the terminal ileum is also affected more severely in the proximal small bowel. The refractory response to a gluten-free diet and the abrupt onset makes the diagnosis of celiac sprue (choice A) less likely. Enterohemorrhagic strains of E. coli (choice B) can cause a hemorrhagic colitis, which would have guaiac-positive stool. Enterotoxigenic strains of E. coli (choice C) can cause traveler's diarrhea and cholera-like disease with profuse watery diarrhea. There is no evidence here for chronic pancreatitis (choice D), which usually will occur in patients who have a long history of chronic alcoholism. Review Tropical sprue causes progressive villus atrophy in the small intestine that is similar to celiac sprue. This condition is common in travelers to endemic tropical areas and is believed to be caused by an infection. After treatment with folate and antibiotics, patients rapidly improve. Patients are at risk for recurrent disease if they return to the endemic tropical region. Vitamin B12 treatment is also indicated.

A 36-year-old man with ulcerative colitis develops pruritus and fatigue. Alkaline phosphatase is elevated. The biliary tree appears beaded on barium radiograph. Which of the following is the most likely diagnosis? A. Acute cholecystitis B. Cholesterolosis C. Chronic cholelithiasis D. Gallstone ileus E. Sclerosing cholangitis

E. Young men who have ulcerative colitis are at increased risk for developing primary sclerosing cholangitis (PSC), a chronic cholestatic condition that leads to fibrosis of the bile ducts. This cholestasis leads to an elevation in the alkaline phosphatase level. A classic clue to the diagnosis is a beaded appearance of the biliary tree on barium radiograph. Patients who have acute cholecystitis (choice A) present with acute onset of right upper quadrant pain, fever, tenderness, and leukocytosis. Cholesterolosis (strawberry gallbladder; choice B) refers to lipid foci deposited in the gallbladder wall. It is asymptomatic and unrelated to cholelithiasis. Cholelithiasis (gallstones; choice C) is commonly asymptomatic but can cause biliary colic and may progress to acute cholecystitis. Gallstone ileus (choice D), caused by obstruction of the small bowel by gallstones, is associated with air in the biliary tree on abdominal radiograph. Review Sclerosing cholangitis is a condition in which there is scarring in the bile ducts, leading to cholestatic liver disease, cirrhosis, and end-stage liver disease. It is most common in young and middle-aged men, especially those with underlying inflammatory bowel disease. Pruritus and jaundice are presenting symptoms. Serum alkaline phosphatase levels are elevated along with serum gamma-GT levels. When hepatocellular damage occurs, there is elevation of aminotransferase levels and conjugated bilirubin. ERCP and MRCP studies will show tight narrowing in the extrahepatic biliary tree caused by progressive structuring. Typically there is both intrahepatic and extrahepatic duct involvement.

A 40-year-old obese white woman, mother of 5 children, gives a history of repeated episodes of right upper quadrant abdominal pain. The pain is triggered with the ingestion of fatty foods and relieved with anticholinergic medications. The pain is spasmodic, radiates to the right shoulder and around to the back, and is accompanied by nausea and occasional vomiting. The patient is in no pain at the moment but is anxious to avoid further episodes. She is afebrile, and physical examination is unremarkable. Which of the following is the most appropriate next step in management? A. Sonogram of the biliary tract and gallbladder B. Upper gastrointestinal series with barium C. Antibiotics, intravenous fluids, and nothing by mouth D. Endoscopic retrograde cholangiopancreatogram (ERCP) E. Exploratory surgery

The correct answer is A. The clinical description is classic for biliary colic caused by gallstones that are intermittently impacted at the cystic duct. The diagnostic study of choice to confirm the presence of gallstones is a sonogram. An upper gastrointestinal series (choice B) will miss the diagnosis. This woman does not have a problem in her stomach or duodenum. She has to be suspected of having gallstones, and the study has to target that area. Antibiotics and intravenous fluids (choice C) are required to "cool down" an episode of acute cholecystitis. This woman, however, does not have fever, leukocytosis, and a tender right upper quadrant. She does not have acute cholecystitis. Endoscopic retrograde cholangiopancreatogram (ERCP) (choice D) would be an expensive, invasive, and totally unjustifiable way to take a look at the gallbladder. This procedure is indicated for patients who have choledocholithiasis or pancreatitis caused by retained stones, because stones can be removed via this technique. Exploratory surgery (choice E) would be even worse. This woman will need surgery, but it should be directed at the gallbladder and done laparoscopically as an elective procedure (i.e., at a convenient time) after a diagnosis has been confirmed. Review The clinical picture is typical for gallbladder stones. Follow the 4 F's in cases of a colicky abdominal pain exacerbated by fatty meals: Fat Forty Fertile Female The diagnostic study of choice is an upper-abdominal ultrasound.

A 61-year-old man is found to have a malignant polyp in the cecum. He undergoes a right hemicolectomy, and the specimen reveals a 2-cm adenocarcinoma extending into, but not through, the muscularis propria. Eleven lymph nodes are negative and there is no evidence of distant metastatic spread. He recovers uneventfully and is discharged home on the postoperative day 6. Four weeks later he develops sudden onset of abdominal distention with vomiting and the inability to pass flatus. Rectal examination reveals no masses and brown stool that is negative for occult blood. An abdominal radiograph is pending. Which of the following is the most appropriate next step in management? A. Placement of a nasogastric tube B. Small bowel series C. Barium enema D. Abdomen and pelvic CT scan E. Surgical exploration

The correct answer is A. This patient has signs and symptoms of a small bowel obstruction. Given the histologic findings of the surgical specimen of an early (Duke B1) lesion, it is highly unlikely that there is recurrent tumor, especially given the prompt occurrence of this bowel obstruction. The most likely etiology is therefore adhesions, which have caused this obstruction. Placement of a nasogastric tube would be the most appropriate measure to decompress the bowel. In most of these patients, this approach will lead to gradual and complete resolution of the bowel obstruction. Should there be any symptoms of clinical deterioration with the placement of a nasogastric or small intestinal tube, then surgery (choice E) should be considered. Although a small bowel series (choice B) or a CT scan (choice D) may be helpful in delineating the site of the adhesion and small bowel obstruction, they would be performed after bowel decompression with a nasogastric tube. Barium enema (choice C), if thought necessary, would be performed after bowel decompression, but it is unlikely to demonstrate an obstructive point. That is because the obstruction is unlikely to be caused by anastomotic tumor recurrence, given the early lesion and recent resection. Surgical exploration (choice E) would not be indicated until a trial of conservative therapy with small bowel decompression and a period of NPO have been attempted. Review In the setting of a bowel obstruction, IV fluids, a nasogastric tube, and bowel decompression are indicated. Most patients will improve with conservative management; if the patient fails to do so, then surgery must be considered.

A 28-year-old woman has significantly symptomatic peptic ulcer disease. Extensive medical management, including eradication of Helicobacter pylori, fails to heal her ulcers. Endoscopy shows several duodenal ulcers in the first and second portions of the duodenum. She also complains of watery diarrhea. Which of the following is the most appropriate next step in management? A. Biopsy of the duodenal ulcers B. Culture of the watery stools C. Measurement of serum gastrin D. Repetition of the H. pylori eradication every 2 months E. Replenishment of the normal gut flora

The correct answer is C. Virulent and extensive peptic ulcer disease should trigger a workup for gastrinoma. The presence of watery diarrhea actually adds to our suspicion that a gastrinoma must be present. Biopsy (choice A) is not the answer, because a serum gastrin level will make this diagnosis. A biopsy may prove to be needed in gastric but not duodenal ulcers given the low risk for malignancy in duodenal ulcers. . Culture of the stool (choice B) would also miss the point. The diarrhea is secretory, not infectious. The diarrhea occurs as a result of a patient having excessive gastrin secretion, not infection. Continued medical therapy (choice D) will continue to fail as long as the very high levels of gastrin are present. If the patient has not responded to treatment via eradication of H. pylori, further attempts will also be likely to fail. The gut flora does not need to be replenished (choice E). The diarrhea is not caused by overgrowth of pathogens. The underlying cause of this patient's condition is a gastrinoma and not an alteration in the gut flora, which is more likely to occur in the setting of C. difficile infection. Review Zollinger-Ellison syndrome should be suspected when patients present with either ulcers that are not responsive to appropriately given therapy (even to the point at which H. pylori infection is eradicated) or whenever multiple ulcers or ulcers located in atypical areas are found during endoscopy. The evaluation method of choice is a serum gastrin level or secretin infusion test, which will both be elevated in this condition as a result of the patient having a gastrinoma. The main goal of treatment is control of gastric hypersecretion with proton pump inhibitors. Surgery may be used for those who have localized disease or in those who fail to respond to therapy.


Related study sets

Computer Science Chapter 4 and 5 True or False

View Set

Philosophy of Critical thinking part 2: Psychological Impediments and Language

View Set

Lesson 69 Test 7 Ch 8 abeka 5th grade history

View Set

AP Statistics Semester 2 Quiz/Checkpoint Questions

View Set

CISM - Information Security Governance, Strategy, Objectives & Metrics

View Set

Chapter 12 - Psychological Disorders

View Set